+ All Categories
Home > Documents > Indirect Taxation CA Workbook

Indirect Taxation CA Workbook

Date post: 03-Oct-2014
Category:
Upload: devapriya11
View: 192 times
Download: 11 times
Share this document with a friend
Popular Tags:
189
CA. Raj Kumar Practice Booklet CA FINAL INDIRECT TAXATION PRACTICE BOOKLET- PART (1) (UPDATED UPTO 31 ST OCTOBER 2010)
Transcript
Page 1: Indirect Taxation CA Workbook

CA. Raj Kumar Practice Booklet

CA FINAL INDIRECT TAXATION

PRACTICE BOOKLET- PART (1) (UPDATED UPTO 31ST OCTOBER 2010)

Page 2: Indirect Taxation CA Workbook

CA. Raj Kumar Practice Booklet

AUTHOR ALL RIGHTS ARE RESERVED. NO PART OF THIS BOOK SHALL BE REPRODUCED, STORED IN A RETRIEVAL SYSTEM, OR TRANSMITTED BY ANY MEANS WITHOUT WRITTEN PERMISSION OF AUTHOR.

““EEvveerryy eeffffoorrtt hhaass bbeeeenn mmaaddee ttoo aavvooiidd eerrrroorrss oorr oommiissssiioonnss iinn tthhiiss ppuubblliiccaattiioonn.. IInnssppiittee ooff tthhiiss,, eerrrroorr mmaayy ccrreeeepp iinn.. AAnnyy mmiissttaakkee,, eerrrroorr oorr ddiissccrreeppaannccyy nnootteedd mmiigghhtt bbee bbrroouugghhtt ttoo oouurr nnoottiiccee,, wwhhiicchh sshhaallll bbee ttaakkeenn ccaarree ooff iinn tthhee nneexxtt eeddiittiioonn..””

S.N CONTENT PAGE NO.

Page 3: Indirect Taxation CA Workbook

CA. Raj Kumar Practice Booklet

1 Problem solution-

→ Excise valuation → Central Excise Rules, 2002 → Cenvat credit Rules, 2004 → Small scale industries → Customs valuation → Duty Drawback → VAT → Service Tax

5

2 Past Examination Questions 101

3 Excise Audit 2000 185

4 Examination Tips 190

Page 4: Indirect Taxation CA Workbook

CA. Raj Kumar Practice Booklet

NOTE –

1. In the Numerical questions Education cess @ 3% shall mean -Education Cess @2% and Secondary and Higher Education Cess @1%

2. If nothing specified regarding the status of assessee then always assume NON SSI.

Problems in- Excise Valuation

Q.1:- An assessee has factory in Kolkata. As a sales policy, he has fixed uniform price of Rs. 2,000 per piece (excluding taxes) for anywhere in India. Freight is not shown separately in his invoice. During the FY 2 3 s: -- (i) Sale at factory gate in Kolkata-- 1,200 pieces – no transport charges; 00 -04, he made following sale

Page 5: Indirect Taxation CA Workbook

CA. Raj Kumar Practice Booklet (ii) S charges incurred – Rs. 28,000; (iii) Sale to buyers in Bihar—400 pieces—actual transport Charges incurred Rs.18, 000; (iv) Sale to buyers in Kerala—1

ANS.1

ale to buyers in Gujarat-- 600 pieces—actual transport

,000 pieces—actual transport charges—Rs. 54,000. Find the assessable value per piece.

Assessable value shall be determined as per sec. 4(1)(b) read with Rule 5 which provides that,

Where any excisable goods are sold

actory,

The transaction value, excluding the cost of transportation

At a place other than f

Then the value of such excisable goods shall be deemed to be

from the factory up to the place of delivery of such

As per explanation 1- “Cost of transportation” includes-

i. The actual cost of transportation; and

ii. In case where freight is averaged, the cost of transportation calculated in accordance with generally accepted

appears that assessee is following average freight, therefore deduction shall be allowed on equalized basis and actual cost shall be ignored.

Calculation of Equalized Freight

excisable goods.

principles of costing.

And because assessee is having uniform price all over India, it

o. of pieces sol ctual T

,000

Place of delivery N d A ransportation charges

Kolkata 1,200 NIL

Gujarat 600 28,000

Bihar 400 18,000

Kerala 1 54,000

Total 3,200 1,00,000

Equalized Freight = Total transportation cost

units sold Total no. of

0 = 1,00,00

3,200

Sales price – equ

2,000-31.25 75

Fo lete computer systems. g quotation:-

Amount (Rs)

= 31.25

Assessable Value = alized freight

= = Rs. 1,968.

Q.2:- Sigma Ltd. Asked for a quotation from Omega Ltd. r the supply of 100 compOmega Ltd. Furnished the followin

Particulars

Page 6: Indirect Taxation CA Workbook

CA. Raj Kumar Practice Booklet

Total price per unit 50,000

Advance to be paid p

Terms: Delivery one month from the date of receipt of the firm order and advance. Sigma Ltd. accepts the ollowing alterations which are agr

oard would be supplied free of cost by Si gma Ltd. Is able chase the keyboard for Rs. 3,000 per unitcharged by Omega Ltd. is to reduced to Would make an

dvance of Rs. 20,000. However, no interest is pay ble on advDetermine the assessable value u/s 4 and the Excise Duty liability @ 15% ad valorem.

ANS.2-

Components CPU 20,000

Monitor 10,000

Keyboard 5,000

35,000

Labour and overheads 10,000

45,000

Profit 5,000

er unit 20,000

quotation subject to the f eed to by Omega Ltd.

(i) Keyb gma Ltd. to omega Ltd. Since Sito pur .

(ii) Profit Rs. 4,000 since Sigma Ltd. a a the ance.

Computation of Assessable Value

Particular Amount (Rs)

CPU 20,000

Monitor 10,000

Keyboard (i) 3,000

TOTAL 33,000

Labour and overheads 10,000

TOTAL 43,000

Profit (Actual charged) 4,000

Add- consideration (ii) 1,000

47,000

Additional

seAs ssable Value 48,000

e Duty @ 15.45% advaloreExcis m (48,000 * 15.45/100) 7,416

(Inclusive of 2% education cess, &1% SHE CESS)

Notes:-

Page 7: Indirect Taxation CA Workbook

CA. Raj Kumar Practice Booklet lanation 1 to rule 6 provides where any material component part etc. are supplied by buyer free of charges, its

apportioned cost shall be added to the price paid or payable if not already included in it. Thus, Rs. 3,000 shall be

i of advance deposit profit has been reduced by Rs. 1,000. Thus, i ill be included in on advance deposit is includible in the assessable value where has in d the price.

2 to Rule 6].

How would you arrive at the assessable value for the purposes of levy of excise duty the following articulars-cum-duty selling price exclusive of sales-tax Rs. 10,000 - Rate of excise d applicable product: 15%

nt allowed - Rs. 1,200 - Freight Rs. 750.

i. Exp

added.

i. In the given case because t wAV. Interest [Explanation

it fluence

Q. 3 - from p- Trade discou

uty to the

Ans:3-

Cum Duty Price 10000

Less: Permissible deductions

Trade discount 1200

Freight 0 50 75 19

8050

Assessable Value

8050 x100/115.45 6973

Excise duty including Education Cess

8050 x15.45/115.45 1077

Excise duty 15% on AV 1046

Education Cess- 2% 21

Higher Education Cess- 1% 10

Note - It is assumed that cum duty price is given before tra d freight.

eces of a product ‘A’ were manufactured du 2008-09. Its list price (i.e. retail price) is Rs. 250 per exclusive of taxes. The manufacturer offers 20% disc o wholesalers on th price. During thewere sold in wholesale, 510 pieces were sold in retail, and 35 pieces were distributed as free sa alance

f 115 pieces was in stock at the end of the year. T ate of duty is 15%. What is the total duty e during Assume that the manufacture is not eligible for SSI concession

-

ue

de discount an

Q.4-1,500 pi ring piece, pieces

ount t e list year, 840 mples. B

quantity othe year 08-09

he r payabl

Ans:4

Calculation of Sales Val

Particulars Qty. In units Rs. Rate Value Rs.

Page 8: Indirect Taxation CA Workbook

CA. Raj Kumar Practice Booklet 40 00 00 Whole Sale 8 2 1680

Retail 510 50 00 2 1275

Samples 35 00 2 7000

Total Assessable Value 302500

Excise duty @ 15% 45375

Edu 908 cation Cess2%

Secondary Higher Education Cess 1% 454

Total Excise duty payable 46736

Note:-Samples are to be valued -As per Rule 4 Read with Rule 2, of valuation rules, 2000,

The value of the excisable goods(samples) shall be based on the value of “such goods” (Identical goods) sold by the assessee,

For delivery at any other time nearest to the time of the removal of goods under assessment, ormal transaction Value” means the transaction value at which the greatest aggregate quantities of goods are

sold.

ointed brokers for obtaining orders from wholesalers. The brokers procure orders for kerage of 5% on selling price. Manufacturer sells goods to buyers at Rs. 250 per piece ce is inclusive

of sales tax and Central excise duty. Sales tax rate is 6% and excise duty rate is 20%. What is the AV, and what is duty

ns: 5-

t (In Rs.)

N

Q.5-A manufacturer has app which they get bro . The pri

payable per piece?

A

particulars Amoun

Sale Price 250.00

(-) sales tax [250*6/106] 14.15

Cum –duty price 235.84

Excise duty [235.84*20.6/120.6] 40.29

Page 9: Indirect Taxation CA Workbook

CA. Raj Kumar Practice Booklet xcise duty 20% 39.11 E

Education Cess 2% 0.78

Higher Secondary Education Cess 1% 0.39

Total Duty payable 40.29

Note: Sales man Commission is includible in A.V.

Q.6-A manufacturer has to supply a machinery on following terms and conditions: (a) Price of machinery: 3,40,000 (net Packing (normally done by him for all machinery) : 4,000 (c) Design and drawing charges

re of machinery : 30,000 (Net of taxes and duties) (d) Central Sales Tax @ 2% (e) Central Excise ash discount of Rs. 5,000 will be offered if full payment is received before dispatch of goods. (g) The

ed along with bought out accessories @ Rs. 8,500. The accessories were optional and assessee on. You are informed that (a) the buyer made all payment before delivery. (b) The manufacturer

incurred cost of Rs. 1,200 in loading the machinery in the truck in his factory. These are not charged separately to buyer. Value’ and the duty payable

ns:6-

of taxes and duties) (b) relating to manufactuDuty @ 20% (f) Cmachine will be suppliexercised the opti

- Find the ‘Assessable

A Calculation of Assessable Value

Price of Machinery 340000

Add: Inclusions

Packing Charges 4000

Design and drawing

Accessories

30000

500

8

42500

382500

Less: Cash Discount 5000

Assessable Value 377500

Excis 75500 e Duty @20%

Education Cess 2% 1510

Higher Secondary Education Cess 1% 760

Total Duty Payable 77770

Page 10: Indirect Taxation CA Workbook

CA. Raj Kumar Practice Booklet

2. Loading charges of machinery Rs. 1200 is includible in A. V. However it is specified that they are not charged separately, hence it is assumed that it is included in the price of machinery.

and duty payable - if the product covered under MRP Provisions?

e: Rs. 1,100/- per unit. - Sales Tax, Surcharge, Octroi and other Local Taxes: - Cash e Discount: 8% - Primary and Secondary packing cost include in the above MRP: e

duty rate: 8% advalorem. Abatement 40%.

Ans:7-

Maximum Retail Price 1,100

Note:

1. CST is not includible in Valuation.

Q.7-Find Assessable Value -

Maximum Retail Trade Pric 10% Discount: 2% - Trad d Rs. 100 - Excis

Less: Abatement 40% 440

Assessable Value 660

Excise Duty @8% 52.80

Education Cess 2% 1.06

Higher Secondary Education Cess 1% 0.53

Total Duty Payable 54

Note-When the product covered under MRP except notified abatement no other deduction will be available.

/s. Karan & Co., Ghaziabad sold 3000 emergency lamps at a uniform duty price of Rs. 1000 per piece for delivery at any place .

The details of sales are as follows:

-1000 lamps were sold at the Ghaziabad factory gate and hence no transport charges were incurred on them.

-1000 lamps were delivered to a buyer at kanpur by incurring freight charges of Rs.14,000 and

-1000 to a buyer at chandigarh at a freight cost of Rs.10,000. What is the assessable value per emergency lamp?

Ans: 8

Q. 8 -M

Calculation of Equalized freight per lamp

Page 11: Indirect Taxation CA Workbook

CA. Raj Kumar Practice Booklet ight = Total freight/total no of units sold Equalized fre

(0+14000+10000/1000+1000+1000)= 24000/3000 per lamp Rs. 8

A V per lamp = Rs. 1000- Rs. 8 992

Note: Value as per Valuation Rule 5 (FOR Contract)

Q. 9-Having regard to the provision of section 4, compute/derive the assessable value of excisab ods for levy of duty of excise, given the following information:

Particulars Rs.

le go

Cum-duty wholesale price including sales tax of Rs. 2,000 15,000

Normal Secondary Packing cost 1,000

Cost of Special secondary packing 1,500

Cost of durable and returnable packing 1,500

Freight 750

Insurance on freight 200

Trade discount (norm 1,000 al practice)

Rate of C.E. duty as per C.E. Tariff d-valorem 15% A

State in your answer, reasons for the admissibility or otherwise of the deductions.

ANS. 9

STATEMENT SHOWING COMPUTATION OF ASSESSABLE VALUE

Particulars Rs.

Cum-duty wholesale price

Less:

⇒ Sales Tax (WN-2) 2,000

⇒ Normal Secondary Packing Cost (WN-3) Not deductable

⇒ Cost of special secondary packing (WN-3) Not deductable

⇒ Cost of durable and returnable packing (WN-4) 1,500

0

15,00

Page 12: Indirect Taxation CA Workbook

CA. Raj Kumar Practice Booklet

⇒ Trade discount (normal practice) (WN-5) 1,000

Freight (including transit insurance) (WN-6) 950

5,450

Cum e value

s

-duty assessabl

Le s: Excise duty (9,550 * 15.45/115.45) (1,278)

9,550

ASSESSABLE VALUE 8,272

Wo

n done in accordance with Rule 5 of Central Excise Valuation

2.

4. he transaction value as charges thereof can’t be

5. o. 354/81/2000). Assuming that given trade discount has actually been passed

on, it is also deductible.

ctible as cost of transportation. (It has been assumed

s (on excisable goods) @ 2% and Secondary & Higher Education Cess (on excisable goods) @ 1% is leviable additionally. Thus, total effective rate is 15.45%

epots of Company in different States. The price is inclusive of excise duty. Local sales tax is charged extra. During the year, 3,000 Tons of ‘P’

:

Qty. sold Freight Charge paid (Rs.)

Delhi 1,400 11,30,000

Rajasthan 500 7,40,000

rking Notes:

1. Freight and transit insurance are appearing in the question indicating thereby that transaction is one for delivery of goods at customer’s premises and not of ex-factory delivery. Further assessing that said transaction satisfies all the other requirements of Sec 4(1)(a), valuation has beeRules, 2000.

Sales tax is deductable from the price to arrive at TV as definition of TV specifically provides for exclusion of sales tax.

3. CBEC has clarified that charges for packing are also includible in the transaction value as these charges are also by reason of, or in connection with sale of goods. It is immaterial whether the packing is normal, secondary or special secondary (Circular No. 354/81/2000).

Cost of durable and returnable packing shall not be included in tsaid to be reason of, or in connection with the sale.

CBEC has clarified that discount of any type or description is deductible if it is established that it has actually been passed on to the buyer (Circular N

6. As per Rule 5 of Central Excise Valuation Rules, 2000 cost of transport from the place of removal to the place of delivery shall be deducted while arriving at assessable value. Thus, cost of freight is deductible. Also, SC in case of BOMBAY TYRES INTERNATIONAL has held that cost of transportation will include the cost of transit insurance. For that reason, cost of transit insurance is also deduthat freight and insurance have been charged on actual basis).

7. The given rate is 15% -- the rate of basic excise duty. Education Ces

(15%+2% of 15% + 1% of 15%).

Q.10-Product ‘P’ is sold by the Company at uniform price of Rs. 15,000 per Ton at various d

was sold in Haryana, Delhi and Rajasthan as per following details

State

Haryana 1,100 9,50,000

Page 13: Indirect Taxation CA Workbook

CA. Raj Kumar Practice Booklet ight charge’ is from fa pot. Excise duty rate is 16.00%. What is the ‘Va

nder section 4 of Central Excise and total excise duty payabl

er ton is given depot o deductions for fre the price is inclusi duty 16% the Assessable Value will be calculated using backwards = 15000 x 100/116.48 12877.75

Calculation of Total Assessable Value

The ‘fre ctory to the de lue’

u e?

Ans:10-

Price p price. N ight Since ve of excise

Depot Qty in tons AV per ton Assessable

Rs. Value Rs.

Haryana 1100 12877.75 1,41,65,525

Delhi 1400 12877.75 1,80,28,850

Rajasthan 500 12877.75 64,38,875

3000 3,86,33,250

Excise Duty @ 16% 61,81,320

Education Cess 2% 1,23,626

Higher Secondary Education Cess 1% 61,813

Total Duty Payable 63,66,760

Q. 11- A trader is owner of a brand name ‘J-17’. He supplies materials to a job-worker. The job worker goods with brand name ‘J-17’ and supplies the goods to the trader. Cost of i

manufactures nputs is Rs. 360 per piece, inclusive of

per piece to manufacture the product. The trader sells the goods in market at Rs. 630 per piece. The rate of duty is 16%. Find the Assessable Value. What is the uty payable per piece?

ns: 11

ll taxes.

raw material and Job worker charges are not relevant

transport cost up to the factory of job worker. Job worker charges Rs. 130

d

A

Assessable value will be Rs. 630 Per Piece, As per Valuation rule 10A, the price at which the Principal manufacturer sells the goods.

Assumption-it is assumed that market price of Rs. 630 is exclusive of a

The value of Cost of

Q. 12- M/s. Ashok Leyland, sent their chassis (value Rs. 5,00,000) to M/s. Raj Nandini & Sons, for bus body building. M/s. Raj Nandini & Sons built the body and sent the bus to the former with their bill for Rs.5,00,000. Wh

o will pay E.D.

on the bus? . Ashok Leyland will sell the bus for Rs.15,00000/- What is the assessable value? Work out the total E.D.

Page 14: Indirect Taxation CA Workbook

CA. Raj Kumar Practice Booklet ayable assuming Cenvat @ 16 % and taking the values given above.

ns: 12

/s T V Raj Nandini & Sons has to pay excise duty, because the body building of vehicle is deemed manufacture as per section/chapter Notes of CETA.

Valuation can be done using Job work valuation rule 10A

p

A

M

. Assessable value will be the price at which Ashok Leyland khs. Assumption-it is assumed that the price of Rs. 15 lakh is exclusive of all taxes. sells bus i.e. Rs. 15 la

Total Duty payable

ssessable value 15,00,000 A

Excise Duty @ 16% 2,40,000

Education Cess 2% 4,800

Higher Secondary Education Cess 1% 2,400

Total Duty Payable 2,47,200

Q.13:- (i) Discuss briefly how ‘Value’ is to be determined in this situation?

X Ltd. Does not effect any sales of its goods ex-factory. Goods are sold only from Depots/branches situated to depots. X Ltd, the

assessee and its customers are not related persons and price is the sole consideration for the sale. What is the basis for determining the value of clearances

( d. Manufacture TV sets. They had sent the TV sets from their plant to their depot at Jammu. The depot sold them at Rs. 12,000 on 1.8.2002 and at depot at Rs. 12,500 per set on 10.8.2002 Please mention wh ry on 3.8.2002 and 10.8.2002.

ANS. 13

away from the factory and costs are incurred for transport of goods from factory

ex-Depot/branches?

ii) M/s OTV Lt

at would be the value of the TV sets removed from the facto

(i) Rule 7 of V

Valuation of Depot Sales

aluation (DPEG) Rules, 2000 is applicable in case of stock transfer.

It provides that

If the excisable goods are transferred to a depot, or agent etc.

From where they are to be sold,

Page 15: Indirect Taxation CA Workbook

CA. Raj Kumar Practice Booklet

⇒ Where such goods are not sold at or about the same time, at the time nearest to the time of removal of goods under

“Normal transaction value” means the value at which the greatest aggregate quantity of goods from the depots etc. are s ov r the f

Rule 7 w any deduction from such NTV. Also, Rule 5 of the said rules can not be applied for valuation of stock transfer as rule 7 is a specific provision for valuation of stock transfer. Hence, transportation charges from factory to dep lu d.

Therefore n

Applying the provisions of section 4(1)(a) read with Rule 7 of Valuation (Determination of Price of Excisable Goods) Rules, 2000, as discussed above.

b) In 2 case, the NTV shall be Rs. 12,500. As on the date of removal itself NTV is available at the branch. And it een clarified by the board that first NTV should be taken of the day on which goods are removed from the

factory. And if not available on that day only then NTV of the nearest past can considered.

Q.14-How would you arrive at the assessable value for the purpose of levy of excise duty from the following particulars: g price exclusive of sales tax Rs 20,000 * Rate of excise du licable to the pr duct 16% * Trade

owed Rs. 2,400 * Freight Rs. 1,500

ns: 14-

The value shall be the normal transaction value of such goods sold from such other place at or about the time of removal from factory/warehouse and,

assessment.

old at or about the time of rem al of the goods f om actory/warehouse.

does not allo

ot can not be exc de

when goods are transferred from factory to depot, value of the goods shall be the price at which goods are ormally being sold at the branch, at the time of removal from factory.

(ii) Date of removal (a) 03.08.2002 (b) 10.08.2002

from factory

Price at Branch As on 01.08.02- Rs. 12,000 As on 10.08.02- Rs. 12,500

a) In 1st case, the NTV shall be Rs. 12,000. As the nearest to 3.8.2002 shall be taken as the nearest past which shall be 1.8.2002.

nd

has b

* Cum-duty sellin ty app odiscount all

A

Cum duty selling price 20,000

Less: Permissible deductions

Trade Discount 2400

Freight 1500

16,100

Assessable Value = 16100 x 100/116.48 13,822.12

Excise duty@ 16% 2,211.54

Page 16: Indirect Taxation CA Workbook

CA. Raj Kumar Practice Booklet Education Cess 2% 44.23

Higher and Seco 22.12 ndary education cess 1%

Total Duty Paya 2278 ble

Q. 15- Determin the following information:

s. 18,000

c) Octroi Rs. 100

d) Insurance from Factory to depot Rs. 100

e) Freight from factory to depot Rs. 700

ate of Basic Excise duty 16% ad valorem

) Rate of Special Excise duty 24% ad valorem

otal Invoice price

e the transaction value and the Excise duty payable from

i) Total Invoice Price R

ii) The Invoice Price includes the following:

a) Sales-tax Rs. 1000

b) Surcharge on ST Rs. 100

f) R

g

Ans: 15

T 18,000

Less: Exclusions

Sales tax 1000

Surcharge on sales tax 100

Octroi 100 1200

16800

Assessable Value – 16800 x 100/141.2 1898.02 1

Excise duty

Basic – 16% 1903.68

Spe cial – 24% 2855.52

Page 17: Indirect Taxation CA Workbook

CA. Raj Kumar Practice Booklet Tota l 4759.21

Ec 2% 95.18

HS Ec 1% 47.59

Total Duty Payable 4901.98

It is assumed-The given price is sale price at depot.

No deduction for freight and insurance from factory to Depot shall be allowed.

nder TV Ltd. is engaged in the manufacture of colour television sets having its factories at Bangalore and Pune factory where it is

tively consumed picture tubes from the formation: - * Direct material cost (per unit) Rs. 600 * bour Rs. 100 * Indirect Labour Rs. 50 *

es Rs. 100 * Indirect Expenses Rs. 50 * Administrative Overheads Rs. 50 * Selling and Distribution al Info rmation: - (1) Profit Margin as p Annual Report of the company for 1999-2000

was 15% before Income Tax. (2) Material Cost includes Excise Duty paid Rs. 100 (3) Excise Duty Rate applicable is

oduction- as per Rule 8 of Valuation Ru 2000.

Particulars unt

Q.16-ThuPune. At Bangalore the company manufactures picture tubes which are stock transferred toconsumed to produce television sets. Determine the Excise duty liability of capfollowing inDirect Expens

Direct La

Overheads Rs. 100. Addition er the

16%.

Ans: 16

Calculation of Cost of Pr les,

Amo

Direct Material (600-100) 500

Direct material 100

Direct labour 100

Administrative overhead 50

Total 750

A 75 dd 10% on Cost

Assemble Value 825

Excise duty @ 16 135.96 .48%

Q.17:- Determi ion in the followin . Quote sectio al Excise Lane the valuat g instances n/rules of Centr w.

Price at Depot as on Actual Sale Price

31/01/2001 at 1/02/2001

Amritsar D Rs. 105/unit Rs.115/unit

Bho ep

epot Rs. 130/unit Rs. 125/unit Rs.135/unit

Place of Removal

01/01/2001 Depot on 0

epot Rs. 110/unit

pal D ot Rs. 120/unit Rs. 115/unit Rs.125/unit

Cuttack D

Page 18: Indirect Taxation CA Workbook

CA. Raj Kumar Practice Booklet

r Depot 100 units (ii) (ii) Quantity cleared to Bhopal Depot 200 units

pot 200 units oods were cleared to respective Depots on 01/01/2001 and actually sold at the depots on

1.

Additional information:-

(i) (i) Quantity cleared to Amritsa

(iii) (iii) Quantity cleared to Cuttack De(iv) The g

01/02/200ANS. 17

Rule 7 and as per section 4(3)(Under c)(iii) and section 4(3)(cc) of Central Excise Act, 1944

T

- Will be the relevant value to pay Excise duty.

And accordingly:-

on the price as on 01/01/2001 Transaction value Rs. 110 * 100 units = Rs. 11,000

- he price prevailing at the Depot

- On the date of clearance from the factory

(i) Clearance to Amritsar depot will attract duty based

(ii) Clearance to Bhopal depot. Depot price at the price Transaction value Rs. 120 * 200 units = 24,000 on 01/01/2001.

(iii) Clearance to Cuttack Depot price on 01/01/2001. Transaction value Rs. 130 * 200 units = Rs. 26,000.

N vant date is 01/01/2001, since on thatpay goods are later sold from de

ote: The rele the goods were cleared to the depots date. No additional duty is able even if pot at higher price.

Determine duction of the under mentioned purpose of captive onsumption Valuation (DPE) Rules, 2

11,600

Quality Control costs 3,500

Research and Development Costs 2,400

1,600

Rea

Q.18:- the cost of Pro product forc under Rule 8 of the central Excise 000.

Rs.

Direct Material

Direct Wager & Salaries 8,400

Works Overheads 6,200

Administrative Overheads 4,100

Selling and Distribution Costs

lizable Value of Scrap 1,200

Page 19: Indirect Taxation CA Workbook

CA. Raj Kumar Practice Booklet Administrative overheads are in relati activities. Material cost includes excise 1,600

on to production duty Rs.

ANS. 18

Cost of Production/Manufacturing for purposes of Captive Consumption shall be determined as per ‘Cost Accounting Standard (CAS) – 4: Cost of Production for Captive Consumption’ issued by ICWAI [CBEC Circular]. Different

on of cost of production/manufacturing for the purpose of Rule 8

PARTICULARS Amount In (RS.)

elements of cost have been treated as per CAS-4.

Computati

Direct Material Consumed

Purchase Price 11,600

dit (WN-i) 1,600Less: Excise duty available as cre

salaries

control costs

in

Total

ess: Realised value of Scrap

AI )

3,500

Direct Wages and

Direct Expenses

Works overheads

Quality

Research & Development Costs 2,400

Adm istrative Overheads

L

COST OF PRODUCTION ( in accordance with the CAS-4 issued by ICW

10,000

8,400

Nil

6,200

4,100

34,600

1,200

33,400

Notes:

(i) AS per CAS-4 excise duty of which credit is available, shall be deducted from the cost of material consumed.

(ii) Selling and distribution Overheads doesn’t form part of Cost of Production/Manufacturing and accordingly, ignored in the question.

Q.19-A manufacturer has agreed to supply a machine on following terms: -

ine at Rs. 4,50,000 (Exclusive of taxes and duties) (ii) Packing for transportation chine Rs. port charges of machinery Rs. 25,000 (iv) Development and tooling charges Rs. 40,000 (exclusive of

on machine supplied Rs. 2,000 (not recovered fro party separately) 6%, (viii) Interest will be charged @ 16% on delaye ond 30 days, (ix) Special

discount of Rs. 5,000. if advance of Rs. 2,00,000 is paid with order. Work out the excise duty liability based on following additional information - (i) Actual transportation cost is Rs. 26,000 (ii) Interest of Rs. 5,000 was charged as

(i) Price of the mach of the ma15,000 (iii) Transtaxes and duties), (v) C.S.T. @ 3% (vi) Octroi paid m(vii) Excise duty @ 1 d payment bey

Page 20: Indirect Taxation CA Workbook

CA. Raj Kumar Practice Booklet yment within 30 days, (iii) The buyer paid advance order.

ns: 19-

inery

party has failed to make pa with the

A

Price of Mach 4,50,000

Add: Inclusions

Packing for transport 5,000 1

Development and tooling 0,000 4 55,000

5,05,000

Less: Octroi 2,000

Assessable Value 5,03,000

Exc 80,480 ise duty @ 16%

Edu 1,610 cation Cess 2%

Hig aher nd Secondary education cess 1% 805

Total Duty Payable 82,894

Note:

Q.20-Cost of production of a product 'X' calculated as per CAS-4 standard is Rs 350 per piece. 500 pieces of a product 0 pieces were sold at Rs. 700 per piece to Industrial Consumers, 70 pieces were sold to a Central

Government department @ Rs. 690 per piece; 210 pieces were sold to wholesalers at Rs. 720 per piece; 70 pieces were l @ Rs. 800 per piece and 20 pieces were given as free samples. Ou e 70 pieces sold to Government

jected, which were subseque old to other cu rs @ Rs. 300 ce, without alance pieces were in stock, ec re so damage ey became

rices are exclusive of excise and sales tax. The rate of duty on the product is 16%. What is total duty payable? Advise Management about steps to be taken in respect of 25 pieces, which have been damaged in

Transport charges not includible in AV, Actual transportation charges are not relevant.

CST, Interest for delayed payment not includible in Valuation.

It is assumed- that Octroi of Rs. 2000 already included in the value of Machine. And as per the provisions of valuation such charges are not includible in A.V

Special discount Rs. 5000 is not deductible, as it is a condition for advance Payment, as the price is not sole consideration. (Rule 6 of valuation rules, 2000.)

were manufactured. 12

sold in retai t of thdepartment, 25 pieces were rebringing them in the factory. B

ntly s out of which 25 pi

stomees we

per pied that th

unsalable. [Note that all the p

storage.

Ans: 20

Particulars Qty Rate Value

Sale to Industrial Consumers 20 00 1 7 84000

Page 21: Indirect Taxation CA Workbook

CA. Raj Kumar Practice Booklet rnment 0 90 Sale to Central Gove 7 6 48300

Sale to Wholesalers 210 20 7 151200

Sale in Retail 70 00 8 56000

Removed as Samples 0 20 2 7 14400

353900

Excise duty @16% 56624.00

Edu 132.48 cation Cess 2% 1

Higher and Secondary education cess 1% 566.24

Total Duty Payable 58323

Note:-Samples are to be valued -As per Rule 4Read with Rule 2, of valuation rules, 2000,

The value of the excisable goods(samples) shall be based on the value of “such goods” (Identical goods) sold by the assessee,

Normal

For delivery at any other time nearest to the time of the removal of goods under assessment, transaction Value” means the transaction value at which the greatest aggregate quantities of goods are

supplies fabrics to independent processor. Cost of fabrics is Rs. 1,150. The processor charges Rs. 450,

Valuation Rule 10 A for Job work value will Rs. 1800. Cost of material, Job worker chargers and transport nt.

factured product under his brand name. Find the assessable value for the purpose of levy of excise duty bringing raw

400. (v)

w valuation Rule 10 A for Job work value will Rs. 15000. Cost of material, Job worker charges and transport nt.

he purpose of captive consumption based upon the follow

als purchased (includes excise duty Rs. 2,000) - Rs. 22,000 Realizable value of scrap – Rs. 0. Wages 00. Manufacturing expenses – 8,000. Administrative expenses – 8,500. Selling and Distribu enses – Rs.

3,400. Expenses of quality inspection department – Rs 4,000. Expenses of research and development department – Rs

sold.

Q.21-A traderwhich includes Rs. 350 as processing charges and Rs. 100 as his profit. After processing goods are sent back to the trader, who sells them at Rs. 1,800. Transport charges for receiving goods at the premises of the processor is Rs. 50 and the transport charges for sending goods after processing is Rs. 60. Please determine the assessable value of the goods under Section 4 of the Central Excise Act.

Ans: 21

As per newnot releva

Q. 22-An assessee manufactures certain goods on job-work basis. The trader supplies the raw material to job-worker and sells the manufrom the following particulars - (i) Cost of raw material supplied by trader – Rs. 10,000. (ii) Cost ofmaterial to factory – Rs. 500. (iii) Value of job work done – Rs. 2,500. (iv) Job worker’s profit – Rs. Transportation charges incurred for returning the manufactured product to the trader – Rs. 600. (vi) Trader’s sales price of finished product – Rs. 15,000

Ans: 22

As per nenot releva

Q. 23- Calculate the cost of production for t ing

details: Materi 2,00– Rs. 12,0 tion exp

Page 22: Indirect Taxation CA Workbook

CA. Raj Kumar Practice Booklet

duction

6,000.

Ans: 23

Calculation of Cost of Pro

Particulars Amount

Material 20,000

Wages 12,000

Manufacturing expenses

Administrative Expenses ,500

8,000

8

Quality inspection department exp 4,000

Expenses of Research and development 6,000

58,500

Less: Realization of Scrap 2,000

Cost of Production 56,500

Q. 24- A manufacturer having a factory at Jaipur has uniform price of Rs. 1,000 per unit (exanywhere in India. During the financial year 2008-09, he made the f

cluding taxes) for sale ollowing sales: (i) Sale at factory gate in Jaipur:

i: 500 pieces – actual transport charges incurred Rs. 12,000. (iii) Sale to buyers in Chennai: 600 pieces – actual transport charges incurred Rs. 48,000. (iv) Sale to buyers in Mumbai:

Ans: 24

Calculation freight per unit

0+12 +48

Assessable v ight = 1000 – 30 = 970

Q.25:- Dete

unrelated (ii) A Ltd. and B Ltd. are interconnected undertakings under section 2(g) of MRTP Act. A Ltd. sells

goods to B Ltd at value of Rs. 100 per unit and to C Ltd. at Rs. 110 per unit, who is an independent buyer.

a value of Rs. 100 per unit. The said goods are captively in its factory. A Ltd. and B Ltd. are unrelated. The cost of production of the

0 per unit.

1,000 units – no transport charges. (ii) Sale to buyers in Delh

900 pieces – actual transport charges incurred Rs. 30,000. Find assessable value per unit under the central excise.

of Equalized

000 000+30000/1000+500+600+900= 90000/3000 = 30

alue = FOR Price – fre

rmine the valuation in following instances. Quote section/rules of Central Excise Law.

(i) A Ltd. Sold goods to B Ltd. at a value of Rs. 100 per unit. In turn, B Ltd sold the same to C Ltd. at value of Rs. 110 per unit. A Ltd. and B Ltd. are related, whereas B Ltd. and C Ltd. are

(iii) A Ltd. sells goods to B Ltd. atconsumed by B Ltd.goods to A Ltd. is Rs. 12

Page 23: Indirect Taxation CA Workbook

CA. Raj Kumar Practice Booklet

ANS. 25

(iv) A Ltd. sells motor spirit to B Ltd. at a value of Rs. 31 per liter. But motor spirit has administered price of Rs.30 per liter, fixed by the Central Government.

(i) Transaction value shall be Rs. 110 per unit,

⇒ When the excisable goods are sold by assessee to related person

As per Rule 9 of valuation rules,

-the value of the goods shall be ,Normal

(ii) Transaction value shall be Rs. 100 for sale to B AND Rs. 110 for sale to C – (Rule 10)

Fo d buyers valuation will be done as per Section 4(1)(a).

Inter connected undertakings will be treated as ‘related person’ for under any other clause of the definition of ‘related person’. And in the given case as they are not covered under any other clause, value shall be determined as if they are

(iii) T

transaction value at which these are sold by the related person to non-related person at the time of removal,

r sales to unrelate

not related person.

ransaction value will be Rs. 100,- section 4(1)(a) In c ated n questioase of sale to unrel perso n of cost of production does not arise.

(iv) Transaction value Rs. 31. – section 4(1)(a) – Since the goods are actually sold at this price, administered price is n

.26:- Calculate the assessable value for levy of excise duty from the following particulars:

duty selling price inclusive of sales tax @ 4% Rs. 60,320

Rate of excise duty applicable to 16%

2,400

du y sellin price s. 2,000

ot considered.

Q

Cum

the product

Trade discount allowed Rs.

Freight (to be charged extra from cum t g ) R

ANS. 26

Computation of Assessable Value

Particulars Rs.

Cum duty Selling Price (inclusive of sales tax) 60,320.00

Less: Freight (to be charged extra) (iii) NIL

Less: Trade discount (ii) 2,400.00

57,920.00

ess: Sales Tax 57,920 * 4/104 2,228.00L

Page 24: Indirect Taxation CA Workbook

CA. Raj Kumar Practice Booklet 55,692.00

[55,692*16.48/116.48] 7,880.00

Excise duty @ 16.48% (i)

Ass 47,812.00essable value

Not

es:-

i. As per section 4(3)(d), Transaction value of goods does not include any Sales tax, Excise duty or any other taxes

ii.

iii. part of assessable value and in the question it is provided that freight is to be charged cum-duty price of Rs. 60,320 therefore no question of deduction.

Q.27:- B Ltd. intment and Skin Ointment. Skin ointment is a

Rs. 33/unit respectively. The sale price of both products included 16% excise duty as BED and 8% excise ED. It also includes CST @ 4%

Units cleared: Eye Ointment: 1, 0

ssible /s 4A: 0% . Calculate the excise duty liabilit both the products.

paid or payable on such goods.

It has been assumed that Rs. 60,320 cum duty selling price is before allowing discount. Hence discount has been deducted from such price.

Freight charges do not formextra i.e. not included in

manufactures two products namely, Eye Ospecified product u/s 4A of Central Excise Act. 1944. The sales prices of the products are at Rs. 43/unit and

duty as S

Additional information:-

0,000 units

Skin ointment: 1, 50,000 units

Deduction permi u 4 y of B Ltd. on

ANS. 27

Duty on eye ointment and skin ointment is required to be calculated separately.

Duty on Eye ointment as per section 4 1)(a):(

Particular (Rs.)

Cum duty price 43.00

Less- sales tax@ 4% (43*4/104) 1.65

41.35

(16% 24.72/124.72) 8.20Less-excise duty @ 24.72% +8%+3%of 24%) (41.35*

33.15Assessable Value

t Rs. 8.Total excise duty on eye ointmen = 20*1,00,000 units = 8,20,000

Page 25: Indirect Taxation CA Workbook

CA. Raj Kumar Practice Booklet Dut on Ski lculated as per section 4Ay n ointment is to be ca

89456*1,50,000

y =Rs. 7,34,184

ani Manufacturing Co. Ltd., i are dispatching 100 ‘Mixing Mach to their dealer in U.P. The dealer es Tax on ‘mixing machine’ in State of Delhi is 6%. The retail

clusiv s tax and excise). Dealers get discount of 15% on this price. Excise duty n cess @ 2%. Pac t is Rs 50 per piece. Manufacturer normally sales the goods with the

are Rs. 1,500 ill be total value of Invoice? Prepare an Invoice showing copy, hich will be useful for transport purposes

articulars s.

Assessable value is equal to sale price less abatement as specified.

Hence, Assessable value =Sale price – 40% of SP

=33 – 13.20 = 19.80

Excise duty per unit @ 24.72% =4.89456 (19.80 x 24.72%)

No. of units cleared =4.

Total Dut

Q. 28 M/s J Delh ines’in U.P is not registered under Central Sales Tax Act. Salprice of the machine is Rs. 800 (ex e of saleis 16% plus educatiopacking. Transport charges

king cosextra. What w

w

Ans: 28

P R Rs.

100 Mixing machines @ Rs. 800 80000

Less: Discount 15% 12000

68,000.00

Packing Cost @ Rs. 50 per piece 5,000.00

73,000.00

Add: excise duty 16% 1680 1

Education cess 2% 233.60

H S ec. 1% 116.80

85,030.40

Add: CST – 6% 5,101.82

90,132.22

Add: Transport Charges 1500.00

Total Invoice Value 91,632.22

Note- Other elements as per rule 11 of CER, 2002 should be mentioned in the invoice.

10 M for value of Rs. 12,500 per ton. The transport cost was Rs. 500 per ton. The same variety and quality of Q. 29-Shiva and Co., an assessee, transferred a consignment of 10 tons paper to the depot from Delhi to Chandigarh on

arch, 2009

Page 26: Indirect Taxation CA Workbook

CA. Raj Kumar Practice Booklet rmally being sold at Chandigarh depot on 10th March, 2009 was at a transaction value of Rs. 15,000 per ton to

unrelated buyers. (i) Which transaction value should be considered for assessment to excise duty? (ii) In case there were rch, 2009, but sales were effected on 1st March, 2009

i. A V Rs. 15000/- (ii) A V Rs. 14,000/- adjustment for price if any for increase/ decrease between 10th march and

Q. 30-

ob work. The principle manufacture sell the product at Rs.4,00,000

(b) Processing of inputs sent by a buyer under his own (buyer’s) challan. Processing charges were Rs. 10,000 and cost on

(comp

Excise duty payable is 16% plus ed on cess of 3%. You are required to (a) Find total duty payable, (b) Procedure to be fo atch in each case after carrying out job work )

Ans:

A. Job work is exempt from duty if input is received under Cenvat. Hence, duty is not payable in this case. The Raw

And A. V. will be Rs. 4 lakhs.

B. Excise duty is payable by the principal manufacturer on the price at which he is removing from the factory, since

C. Repair does not amount to ‘manufacture’ as no new product emerges. Hence, there is no liability of Central Excise Duty. The goods should be cleared under manufacture’s own Delivery Note with full details of operations carried out. If repair process is manufacture duty payable on Rs. 28,000.

Q. 31 -A product which is covered under Section 4A provisions has MRP of Rs. 25 printed on the carton. It is cancelled e. Below that price, MRP price of Rs. 21 is shown to

aving which will be made by buyer. The abatement available is 40% on MRP Excise duty rate is 16%. ate the excise duty payable.

paper no

no sales of that variety and quality of paper on 10th Mapreviously for Rs. 14,000 per ton, what would be your answer?

Ans: 29

1th march.

A large manufacturing unit undertook following job work:

(a) Machining of raw materials supplied by the buyer. The material was sent under Cenvat challan. Job work charges were Rs. 30,000. Cost of raw material was Rs. 3,50,000. These were returned after j

inputs was Rs. 2,00,000.

c) Repairs of a component. Original cost of component was Rs. 25,000 and repairs charges were Rs. 3,000. The onent was sent by customer under cover of his letter. In all these cases, raw material was sent by customer.

ucatillowed by manufacturer for disp

30 Duty payable in each case is as follows:

material supplier has to file a declaration before Assistant Commissioner having jurisdiction over factory of manufacturer (job worker) that excise duty liability on final product will be borne by him. (E/N -214/86)

the price is not given therefore duty can not be calculated. The material cost and Job worker charges are not relevant. Goods should be cleared under serially numbered and pre-authentication Invoice. This invoice should indicate the Assessable value on which duty has been paid.

by drawing two lines across the price, but the price is easily readablindicate the sCalcul

Page 27: Indirect Taxation CA Workbook

CA. Raj Kumar Practice Booklet

ill be Rs. 21, even scoring of Rs. 25 is visibl

Ans: 31

As per Sec 4A Av w e

MRP 21.00

Less Abatement – 40% 8.40

Assessable Value 12.60

Excise Duty – 16% 2.02

Education Cess 2% 0.04

Higher and Secondary education cess 1% 0.02

Total Duty payable 2.08

Q. 32- Asha Ltd. supplies raw material to a job worker Kareena Ltd. After completing the job-work, the finished packet. The product in e assessable value under

from the following det ils: * C material supplied Rs. 30,000/- * Job worker’s charges including profit Rs. 10,000/- * Transportation charges for sending the raw material to the job worker Rs. 3,000/- *

td. Rs. 3,000/

done as per MRP Provisions only

R P per packet 20.00

The details given regarding Job Work not relevant

manufacturer manufactured some furniture within the factory for his own use. He purchased material of Rs. r CAS-4, is

ty @ 16%. The manufacturer generally earns profit of 18% on his total cost. e excise duty and sales tax payable.

product of 5,000 packets are returned to Asha Ltd. putting the retail sale price as Rs. 20 on eachthe packet is covered under MRP provisions and 40% abatement is available on it. Determine thCentral excise law a ost of Raw

Transportation charges for returning the finished packets to Asha L

Ans: 32

Where goods covered under M R P Provisions value will be

M

Less Abatement 40% 8.00

Assessable Value per pocket 12.00

Total Assessable for 5000 packets @ Rs. 12 60,000

Note:

Q. 33- A27,500 for this purpose. Cost of the operations carried out by him, as certified by a Cost Accountant, as peRs. 12,200. The furniture is liable for duSales tax on furniture is 10%. Find th

Page 28: Indirect Taxation CA Workbook

CA. Raj Kumar Practice Booklet ns: 33

rniture is captively consumed- duty shall be payable as per Valuation Rule 8 -At C s 10%

t of Production

A

Note: If Fu ost plu

Calculation of cos

Cost of Raw material 27500

Cost of Operations 12200

39700

Add 10% 3970

Assessable Value 43760

Excise Duty @ 16% 6987

Education Cess 2% 140

Higher Secondary Education Cess 1% 70

Total Duty Payable 7197

Q. 34- Dconsumption i

etermine the cost of production on manufacture of the under-mentioned product for purpose of captive n terms of Rule 8 of the Central Excise Valuation (DPE) Rules, 2000 - Direct material – Rs 11,600, Direct

orks Overheads – Rs 6,200, Quality Control Costs – Rs 3,500, Research and Development Costs – Rs 2,400, Administrative Overheads – Rs 4,100, Selling and Distribution Costs – Rs 1,600,

e of Scrap – Rs 1,200. Administrative overheads are in relation to productio l cost e duty Rs. 1,600.

n

Wages & Salaries – Rs 8,400, W

Realisable Valu n activities. Materiaincludes Excis

Ans: 34

Calculation of cost of Productio

Direct material 10000

Direct wages 8400

Works overheads 6200

Quality control costs 3500

Research and development cost 2400

Administrative overheads 4100

34600

Less: Realizable value of scrap 1200

Cost of Production 33400

Add 10% 3340

Page 29: Indirect Taxation CA Workbook

CA. Raj Kumar Practice Booklet Assessable Value 36740

Note: selling and distribution costs are not includible in cost of production. It is assumed that Research and

r processing. The cost of the art silk process charges Rs. 2,500 and profit Rs. 500.

ost of carriage for moving goods to C's place is Rs. 100 and for moving these back to B, after processing, is Rs. 90. B product for Rs. 16,200. What is the assessable value of the goods under section 4 of the CE Act

ns: 35

s per new Valuation Rule 10 A for Job work value will Rs. 16,200

aterial, Job worker charges and transport charges are not relevant.

development costs are relating to production

Q.35-B, a trader, buys art silk yarn and gives it to C, a job work contractor for furtheyarn supplied to C is Rs. 12,000. C bills B at Rs. 3,000 which comprises ofCsells the final

A

A

Cost m

Problems in Central Excise Rules, 2002

Payment of Duty and filling of return- Question . The excise duty liability of Bindal Steel Industries for the, month of April, 2010 is Rs. 50,000. The details of payment of excise duty by Bindal Steel Industries in the financial year 2009-10 are as follows:- S.No.Particulars Amount (Rs.)

Page 30: Indirect Taxation CA Workbook

CA. Raj Kumar Practice Booklet paid in cash 5, 60, 000

2. Amount of excise duty paid by utilizing the CENVAT credit 4, 50,000 ailable on the inputs and capital goods

the information furnished above, answer the following questions:-

Answer

1. Amount of excise duty

avUsing (i)What is the due date for the payment of excise duty by Bindal Steel Industries for the month of April, 2010? (ii)Central Excise Officer argues that Bindal Steel Industries is required to file the return for the month of April, 2010 in the electronic form. Discuss, whether the said argument is correct? CENVAT Credit Rules, 2004

(i) As per third proviso to rule 8 (1) amended by Notification No. 04/2010-CE an assessee shall be deposit the excise duty electronically through internet banking if he has paid the total duty of Rs. 10 lakh or more(including the amount of duty paid by utilisation of CENVAT credit)in the proceeding financial year. Therefore Bindal Steel Industries is required to pay the duty electronically.

Further, rule 8(1) provides that the duty on the goods removed from the factory during a month shall be paid by the 6th

ay of the following month, if the duty is paid electronically. Hence assessee is liable to pay duty for April, 2010 by the 6th May, 2010.

i) The argument of the Central Excise Officer is correct in the light of the provisions of third proviso inserted to rule 2(1) by Notification No. 04/2010 –CE. It has now made the electronic filing of returns mandatory for the assessee

total duty of Rs. 10 lakh or more including the amount of duty paid by utilization of CENVAT credit in e preceding financial year. Therefore, Bindal Steel Industries is required to file the return for the month of

d

(i1who has paid thApril,2010 in the electronic form by 10th May, 2010.

Problems in CCR, 2004

rence, if the assessee is not a manufacturer, but a service provider

Q. 1- M/s RJ imported some inputs and paid Basic Customs duty Rs. 5 lakhs, surcharge on customs duty Rs. 50,000 and CVD 3(1) Rs. 1 lakh. Calculate the amount that he can claim as Cenvat credit. Would it make any diffe

Page 31: Indirect Taxation CA Workbook

CA. Raj Kumar Practice Booklet

Cenvat Credit available on CVD Rs. 1 lakh. No Cenvat Credit Basic customs duty Rs. 5 lakhs. If assessee is a service rovider Credit can be taken on CVD Rs. 1 lakh if the Imported material used in

ased on termine the Cenvat credit available f e in the current year under the Cenvat credit rules, 2004.

Central Excise duty paid at the

Goods time of purchase of goods (Rs.)

(a) Pollution control equipment 25,000

(b) Spares for pollution control equipment 5,000

(c) Equipment used in office 12,000

(d) Storage tank 10,000

(e) Paints used for packing material 6,000

(f) Packing material 4,000

(g) Lubricating oil 8,000

(h) High speed Diesel oil 7,000

ANS. 2

Ans: 1

p providing Output service.

Q.2:- B the following information; de or the us

Goods Cenvat Credit available (Rs.)

a) Pollution control equipment [Ref Note 1] 12,500

b) Spares for pollution control equipment [Ref Note 1} 2,500

c) Equipments used in office [Ref Note 2] Not allowed

d) Storage tank 5,000

e) Paints used for packing material 6,000

f) Packing material [Ref Note 3] 4,000

g) Lubricating oil [Ref Note 3] 8,000

h) High speed Diesel oil [Ref Note 4] not allowed

l available credit during the year Tota 38,000

Notes:

Page 32: Indirect Taxation CA Workbook

CA. Raj Kumar Practice Booklet 1. Pollution control equipment and spares thereof and storage tank are covered by the definition of capital goods.

Thereof credit shall be allowed. But As per rule 4 of Cenvat Credit Rules, 2004 credit of duty paid on capital goods can not exceed 50% of duty paid on such capital goods in the year of receipt. Therefore credit can be taken only 50% of duty paid on such capital goods.

duty paid on such equipment is not allowed.

3. Lubricating oil and packing material used in or in relation to manufacture of final product are covered by the

2. Definition of Capital goods specifically excludes from its ambit equipments used in office. Therefore credit of

definition of inputs. And board has clarified that raw material used for making packing material shall also be eligible as inputs. Therefore credit can be taken of paint used for packing material. Also 100% credit of duty paid on input can be taken in the year of receipt.

4. High speed diesel oil had been specifically excluded From the definition of input. Therefore credit is not allowed.

Q. 3 -Discuss about the eligibility of Cenvat Credit in each of the following situations - (i) 1000 kgs of raw materials were purchased on which duty paid was Rs. 16,000. Whilst in the production yard, they were destroyed by accidental fire which th 1,000 units of inputs purchased were missing; however ‘Du a id on inputs

Ans: 3

1. Credit is available when inputs are destroyed during the course of process. If they are destroyed before issue

004. However credit cannot denied for minor irregularities. In this credit can be available with the permission of AC/ DC of excise.

Q.4-A manufacturer under CENVAT purchased inputs of value at Rs. 60,000 on which duty of Rs. 9,000 was paid @ o months, due to change in production schedule, he found that he does not need the

material. He sold the inputs lying in stock @ Rs. 70,000 on 17th July 2008. However, due to budget change announced how much?

hat would have been your

When inputs cleared as such (without using) an amount equal to cenvat credit availed is payable Rs. 9000.

Note: Increase in rate of duty, exemption from duty and sale at higher value is not relevant.

Q.5-A manufacturer manufactures 3,500 Nos. of a product ‘P’. Its Assessable Value is Rs. 650 per piece. Duty payable ame, on which duty paid w The manufacturer sells 2,000 pieces in India

and 1,500 pieces are exported under bond. How much CENVAT Credit will be available and what is the duty payable

(ii) 1000 kgs of raw materials on which duty paid was Rs. 10,000 was used in manufacture of a final product for e duty payable is Rs. 8000 (iii) The original invoice for

plic te for transport’ copy of invoice is available, which shows that duty of Rs. 10,000 had been pa

for production, therefore no credit will be available.

2. Credit can be utilized to the extent of Rs. 8000 and balances Rs. 2000 can be carry forward

3. Credit is available only based on original copy of invoice with all prescribed particulars under Rule 9 of CCR, 2

15% on 25th January 2008. After tw

earlier, duty on those inputs was increased to 20%. (a) Does the manufacturer have to pay excise duty? If so,(b) If, instead of increase of duty to 20%, the inputs were exempted from duty in the budget, wanswer?

Ans: 4

As per Rule 3 (5) of CCR, 2004-

is 10%. He bought inputs for the s as Rs. 90,000.

Page 33: Indirect Taxation CA Workbook

CA. Raj Kumar Practice Booklet rough PLA?

for Home consumption.

ce 0

th

Ans: 5

Duty payable on units cleared

AV = 2000 @ Rs. 650 per Pie 130000

Duty EC 3%

Duty Payable 10 % 130000 3900

Less Cenvat credit on Inputs 90000 2700

Duty payable through P L A 40000 1200

Note- As per Rule 6 (6) of CCR, 2004- ccr will be available In respect of inputs contained in the final product which is exported under bond. (Called zero rated goods)

Q.6-An assessee cleared his manufactured final Product during the month of March 2009. The duty payable on the final e month is Basic excise duty Rs. 200000, Special excise duty Rs. 1,00,000 and applicable education cess.

the month he has received various inputs total duty paid on the inputs was as follows. Basic Excise duty Rs. I) Rs. 5000 . Excise duty paid on capital goods received during the month was Rs.

12000 Service tax paid on input services Rs. 1000. For all duties and service tax applicable education cess was paid. ow much duty is payable through account current

inal Product

Duty

(CCR)

EC 3%

(CCR) (Liability)

% (Liability)

product for thDuring 50000, Additional excise duty (GS

H

Ans: 6

Duty payable on F

Duty E C 3

Basic Excise duty 200000 000 6

Special excise duty 100000 000 3

Total 300000 000 9

Cenvat credit available

Basic Excise duty 50000 1500

Excise duty on C G 50% 6000 80 1

Service tax

Add. GSI

1000

5000

30

150

Page 34: Indirect Taxation CA Workbook

CA. Raj Kumar Practice Booklet t available 62000 1860 Total Cenvat credi

Less Cenvat Cre

Duty to be paid t

62000dit

hrough PLA

238000

1860

7140

Q.7-Prepare a Cenvat account in the books of A Ltd., and determine the balance as on 30-09-2008 from the following data: -

1. Opening balance as on 01-04-2008 Rs. 47,000.

16,000 -cum duty price @ excise duty rate of 16% on

goods Rs. 2,32,000).

s. 2 lacs; ED @ 16% of the above, 50 % of the inputs were received on 01-10-2008.

6. Common inputs were used in a product, which was exempted from payment of duty cleared at a price of Rs. 100/unit, which included taxes of Rs. 20/unit; quantity cleared 1,000

. On 07-04-2008 duty pai Rs. Cenvat A/c as Rs. 17,687

unt

2. Inputs received on 04-04-2008 involving excise duty paid Rs. 14,747

3. Purchased raw material for Rs. 1,05-04-2008 and received the lathe into the factory on 05-12-2008.

4. On 06-04-2008 paid excise duty on final products @ 16% through Cenvat A/c (cum duty price of the

5. Inputs cleared as such to a job worker on 01-04-2008 not returned in 180 days, quantity 1,000 Kgs; Assessable value R

units.

7 d on inputs amounting to 17,867 was taken credit for in the

Ans: 7

Cenvat Acco on 30/09/2008

Particulars envat Availa le (Dr) zed (Cr)

C b Cenvat Utili Balance

Date Duty EC 3% Duty EC 3% Duty EC 3%

01/04/2008 Opening balance 47000 1410 47000 1410

04/04/2008 Inputs purchase 14747 442 61747 1852

04/042008 Removals 32000 960 (29747) (892)

07/04/2008 Original entry 17687 531 47464 1423

Rectification (180) (5) 47614 1428

(17867 – 17687)

Page 35: Indirect Taxation CA Workbook

CA. Raj Kumar Practice Booklet Payment of amount

(1000 x 80 x 5%)

4000 43614 1428

On common inputs

28/09/2008 Inputs sent for Job work 32000 960 11614 468

Q,8-. Briefly discuss with the reasons whether in the following case Cenvat Credit is available to an assessee

An assessee purchased inputs weighing 1,000 Kgs. The duty paid on inputs was Rs. 10,000.

During transit, 500Kgs inputs were destroyed.

and, if yes to what extent?

ANS. 8

Assessee can take credit only of Rs. 5000 i.e. duty paid on 500 kg. As 500 kg. were destroyed during transit and were not received in the factory.

As per rule 4(1) of Cenvat Credit Rules, 2004-

The CENVAT credit in respect of inputs may be taken immediately on receipt of the inputs in the factory of the manufacturer or provider of output service.

Rule 2 of CCR, 2004 provides that any goods used in or in relation to manufacture are eligible as inputs and inputs are either received nor they can be used in or in relation to manufacture of final product or for providing output service.

re credit cannot be taken of the duty paid on those inputs.

n. Out of these, 940 litres of final ts were lost in process Discuss eligibility of Cenvat credit in the above

cases

Ans: 9

ence in the both cases credit can be availed.

n

Therefo

Q. 9-500 pieces of inputs were received. Duty paid on these goods was Rs. 2,500. These were issued to production. While on production line, a fire broke out and 200 pieces of inputs lying on the shop floor were destroyed. 1000 litres were received on which duty paid was Rs. 18,000. These were issued to productioproducts were manufactured. 60 litres of inpu

Credit can be availed on duty paid on inputs destroyed during the process. Credit can be availed on duty paid on inputs lost during the process credit is available on 1000 ltrs duty paid amount Rs. 18000).

H

Q.10-X availed Cenvat credit of Rs. 42,000 for manufacture of an item chargeable to duty. These goods were lying in his

Page 36: Indirect Taxation CA Workbook

CA. Raj Kumar Practice Booklet factory till 28-02-2009, from 1.3.2009; the final product was made exempt from duty. Now, when the final goods are cleared, should the Cenvat credit of Rs. 42,000 availed earlier be reversed (Ans Yes)

Ans: 10

As per Cenvat Credit Rule 11 (3) when dutiable goods becomes exempt, Cenvat credit availed on inputs, work-in nd finished goods should be reversed. Hence X has to reverse an amount of Rs. 42000/- of credit availed.

ling under chapter headin om supplier ‘X Co’. The invoice was for Rs 23,200, comprising of price of goods as Rs 20,000 and Rs 3,200 as excise duty and education cess. Pass journal entry

how the bala ll appear in Balanc

ns: 11

process a

Q. 11- A manufacturer purchased machinery fal g 84 fr

in accounts book to record the purchase transaction. Explain nce wi e Sheet

A

Capital Goods A/c Dr 20000

Cenvat Credit Receivable (Capital goods)- Dr 1600

Cenvat credit Receivable (Capital goods) deferred A/c - Dr 1600

To Sundry Creditors – X & Co 23200

In the Balance sheet deferred account will be shown under loans and advances.

assessee is not a manufacture, but a service provider?

Q.12- M/s.RC imported some inputs and paid Basic Customs Duty Rs 5lakhs, surcharge on customs duty Rs. 50,000 and CVD Rs 1 lakh. Calculate the amount that he can claim as Cenvat credit. Would it make any difference, if the

ANS. 12

M/s RC can take credit of Rs. 1,00,000 i.e. of additional duty of customs (CVD). Rule 3(1) of CCR allows credit of additional duty of customs imposed under section 3 of CTA. The credit of other two duties i.e. BCD and surcharge on custom duty is not allowed.

It will not make any difference if the assessee is a service provider as credit of additional duty of customs (CVD) can be availed both by manufacturers and the service providers alike but credit of CVD u/s 3(5) will not available to a service provider.

Q.13-Closing balance of Cenvat Credit in of a manufacturer on 3rd July 2008 was Rs. 11,500. On 4th July 2008, following transactions took place: (i) The manufacturer received inputs under Invoice No. 253 dated 5th June 2008,

g payment of duty of Rs. 74,000; (ii) 90 pieces of Final products were despatched under Invoice No. 768. e value was Rs. 1,200 per piece and excise duty rate was 16%; (iii) Some inputs on which cenvat was taken

old for Rs 0,000 as they hen d d inputs was nvat credit taken was Rs. 21, 2008t cabl s 20%;

ome inputs (10,000 pieces) were sent outside on b work Rs. was p e purchasing these inputs.

evidencinAssessablearlier were s15%, and Ce

. 1,2 were found in excess. W000. However on 4th July

the inputs were receivehe excise rate appli

uty rate one on inputs wa

(iv) S 1st July 2008 for jo ),duty of 5,000 aid whil

Page 37: Indirect Taxation CA Workbook

CA. Raj Kumar Practice Booklet 6000 pieces (out of 10 ces) were returned to the factory after job work repa nvat

credit account

Ans: 13

Particulars Cenvat Available (Dr) Cenvat Utilized (Cr) Balance

On 4th July 2008, ,000 pie . - . - P re Ce

Date ty EC 3% Duty EC 3% Duty EC 3% Du

Opening Balance 11500 345 11500 345

05.06.2008 inv 253 Inputs 74000 2220 85500 2565

Inv 768 Removals 17280 518 68220 2047

04.07.2008 Inputs removed as such

21000 630 47220 1417

Q.14-A manufacturer received certain After receipt of the inputs, the cenv

inputs. The cost of inputs was Rs. 2,00,000 and duty paid @ 16% was Rs. 32,000. at credit was availed of by the manufacturer. He further carried out some processes

for the job work?

5) of CCR, 2004-

As p ule

If fi gto b we

. 15:- H. Ltd. Purchased a Boring-Drilling machine at a cum duty price of Rs. 32,14,476. The Excise duty ged on the said machine was @ 16%. The machine was purchased on depreciation @ 25% Straight Line Method. Using the said information answer the following question:

(ii) What is the Cenvat credit allowable under Cenvat Rules? (iii) What is the amount of cenvat credit reversible or duty payable at the time of clearance of the said

f removed as such?

on the inputs. The cost of processing was Rs. 50,000. The semi-processed material was sent to a small-scale unit for a job work. –Is there any duty payable at the time of removal of inputs for the job work ? The material sent was not returned by the small-scale unit after the job work within 180 days. What will be the duty payable on such goods not returned after being sent out

Ans: 14

As per Rule 4 (

No duty payable when goods send for job work and there is no need to reverse the credit availed when material sends for job work. If the material sent for job work not returned with in 180 day, an amount equal to cenvat credit availed is to be paid i.e. Rs. 32000.

er R 4 (6) of CCR, 2004-

nished oods are directly cleared form job worker place, duty is payable based on AV and prescribed procedure is e follo d.

Qrate charfollowing

(i) What is the Excise duty paid on the machine?

machinery, I

ANS.1 5

i. Computation of excise duty

Page 38: Indirect Taxation CA Workbook

CA. Raj Kumar Practice Booklet (In Rs.)

m

6.48% (32,14,476 x 6.48/116.48) 4,795

Cu duty price = 32,14,476

Less- excise duty @ 1 1 = 4,5

Assessable Value = 27,59,681

Excise duty paid is Rs. 4,54,795.

ii. Cenvat allowable in year 2009-10 = 2,27,397

In year 2010-11 = 2,27,398

As per Rule 4(2) of CCR:-

50% Cenvat Credit can be availed in Current financial year and balance 50% of Cenvat is allowable only in following financial year, if the capital goods are in possession.

ii ount of cenvat credit reversible at the time of clearance of such machinery is Rs. 4,54,795 i.e. the amount equal to Cenvat credit availed on goods.

Since the capital goods were in use during the year 2010-11, cenvat of balance 50% is allowable.

i. Am

As per rule 3(5) of Cenvat Credit Rules, 2004:-

When capital goods or inputs on which credit has been taken are removed as such, then the assessee is required to pay an amount equal to cenvat credit availed and such removal shall be made under the cover of an invoice referred to in rule 11.

Q.16-Machinotech Ltd. purchased a lathe machine at a price of Rs. 1,00,000 on which 16% Excise Duty was paid and edit on the said capital goods. The lathe machine was purchased on 27-01-2008 and an the assessee enjoy the Cenvat credit ? Is it ne to reverse the cenvat credit

amount.

ce

the company availed of the Cenvat crit was disposed of on 29-04-2008. C cessaryon disposal of the machine? If your answer is yes, quantify the

Ans: 16

Cenvat Credit reversible at the time of clearan

Cenvat Credit availed in year 07-08 8240

Less : 2.5% per quarter for TWO quarter – (2.5%* 2 quarter) 412 7828

Cenvat Credit availed in year 08-09 8240

Less : 2.5% per quarter for 1 quarters – 2.5% 206 8034

Total amount payable at the time of clearance incl. ec 3 % 15862

Q.17-Surya Ltd. purchased certain inputs for Rs. 50,00,000 and also paid Excise Duty @ 16% ad valorem. The company

also purchased a drilling machine for Rs. 5,00,000 and paid Excise Duty @ 16% ad valorem. The company availed of

Page 39: Indirect Taxation CA Workbook

CA. Raj Kumar Practice Booklet at credit on the inputs and on the capital good in April, 2008, on the same day. On 10-05-2008, the company

cleared the finished goods to Tara Ltd., the cum-duty price of which worked out to Rs. 70,80,000. The final product

Duty @ 16% ad valorem. The company also deposited Rs. 6,00,00 gh TR-6

ay be noted that the inputs were purchased from . and the drilling machine

from Probha Ltd., on credit in both cases. Pass the necessary journal entries in the books of Surya ltd

Ans 17

Purchase of Raw materials Dr. 5000000

the Cenv

(finished goods) sold attracted Excise 0 throu

Challan on 10-05-2008, itself. It m Usha Ltd

Cenvat Credit Receivable account Dr. 800000

Cenvat Credit Receivable (Education Cess) account Dr. 24000

To Sundry Creditors Cr. 5824000

(Being purchase inputs)

Capital goods Dr. 500000

Cenvat Credit Receivable account Dr 40000

Cenvat Credit Receivable (Education Cess) account Dr 1200

Cenvat Credit Receivable deferred account Dr 40000

Cenvat Credit Receivable deferred (Education Cess) account Dr 1200

To sundry Creditors Cr 582400

( Being machinery purchased)

Excise duty on final product 7080000 x 16/116.48 Dr. 972527

Education cess on excise duty 7080000 x 0.48/116.48 Dr. 29176

To Bank (582524 + ec Rs. 17476) 600000

To Cenvat Credit Receivable account (utilized) Cr. 390003

Cenvat Credit Receivable n Cess) accoun

(utilization) 29176-17476

11700 (Educatio t Cr.

It is assumed Rs.6 lakhs paid though GAR 7 is inclusive of EC 3% Rs. 17476

Cenvat credit Balance

Page 40: Indirect Taxation CA Workbook

CA. Raj Kumar Practice Booklet e zed Availabl Utili Balance

Particulars Duty EC Duty EC Duty EC

Inputs 800000 24000 390003 11700 409997 12300

Capital goods 40000 1200 Nil Nil 40000 12000

Capital deferred account credit available Next year Rs. 40000 and Ec Rs. 1200

envat Credit Receivable (Education Cess) account Dr. 1,200

& Entry will be made as follows----

Cenvat Credit Receivable account Dr. 40,000

C

Cenvat Credit Receivable deferred account Cr 40,000

Cenvat Credit Receivable deferred (Education Cess) account Cr 1,200

Q.18:- U&V Ltd. manufactures 10,000 units of Product-W, assessable value of which is Rs. 400 per unit. a on ra 0. U&V Ltd. sells 2,000 units in India and

8,000 units are exported through a merchant exporter. What is CENVAT credit available and what is the dger ccoun (PLA)? Can U&V Ltd. get any refund of CENVAT credit?

(Ignore Education cess).

ANS. 18

Duty payable is 16%. Duty p id w material is Rs. 3, 00,00

duty payable through personal le a t

India (2,000 x 400 x 16%) 1,28,000

Cenvat credit availed on inputs Rs. 3,00,000

Less: Cenvat credit utilized for sale

In

eauty, Tweety. The ompany has availed CENVAT credit of Rs. 4,00,000 on the common inputs used in the manufacture of ‘Nail Polishes’. uring the financial year 2008-09 the company manufactured 1000 litres of each type of ‘Nail Polishes”. The CENVAT

vailed input was used in equal proportion in all the four types of the products. Examine the availability of Cenvat Credit and duty payable Product Nature of Sale Price excluding Sales Tax & other local taxes Sweety Sale to Home

tion Rs. 30 per 20 ml bottle Pretty Sold to a 100% EOU Rs. 40 per 20 ml bottle Beauty Fully exported Rs. 50 per 20 ml bottle Tweety Supplied to Defence Canteen under exemption Rs. 60 per 20 ml bottle

Balance available 1,72,000

Not need to pay any duty in cash through PLA. As no further adjustment is possible thus refund of Rs. 1,72,000 is available. (As per Rule 5 of Cenvat Credit Rules, 2004)

Q.19-M/s Tips and Toes Ld., manufactures four types of “Nail Polishes”, namely Sweety, Pretty, BcDa

Consump

Page 41: Indirect Taxation CA Workbook

CA. Raj Kumar Practice Booklet

Ans: 19

As per Cenvat credit Rule 5 of CCR, 2004 Credit of duty paid on inputs used in pretty and beauty can be availed. Duty Sweety cleared for home consumption.

n be availed on Sweety, Pretty and Beauty.

5% on the A V of tweety is payable.

assessee cleared his manufactured final Product during the month of January 2009. The duty payable on the asic excise duty Rs. 48,000, NCCD Rs.2000 and applicable education cess. During the

month he has received various inputs total duty paid on the inputs was as follows. Basic Excise duty Rs. 40000, Special xcise duty 4000, Service tax paid on inputs Rs. 80 all duties educat s was paid. uty

is payable through account current

Ans: 20

Duty payable on Final Product

Duty EC 3% Duty EC 3%

payable on

With regard to credit of duty on inputs used in Tweety cleared to defence under exemption Assesse has three options as per Rule 6 of CCR, 2004.

Option 1

If separate books of account are maintained for inputs used in tweety and pretty, beauty and sweety. No credit on duty paid on inputs used in tweety will be available. And credit ca

Option 2

If no separate books of account are maintained for inputs used in tweety and preety, beauty and sweety credit can be availed and amount at

Option 3

When no separate records with intimation to AC, assesses can avail prorate credit on the common inputs based on the mathematic formula.

Q.20-An final product for the month is B

e 00, for applicable ion ces How much d

Basic Excise duty 48000 1440

NCCD 2000 60

Total payable (A) 50000 1500

Cenvat Credit available

Basic Excise duty 40000 1200

Page 42: Indirect Taxation CA Workbook

CA. Raj Kumar Practice Booklet Special excise duty 4000 120

Service tax

(to the extent of Rs. 6000 required and balance will be C/F)

000 180 6

Total Cenvat credit available 50000 1500

Less : Cenvat Credit 50000 1500

Duty payable through P L A 0 0

Q. 21- A manufacturer brings some inputs valued at Rs 25,000 on which duty of Rs. 5,000 has been paid @ 20%. And ec 3% Subsequently the manufacturer sold the input as such, which goods he sold for Rs. 30,000. What is the duty

n the date of clearance on input was 10% ? CWA Inter Dec 1998

Ans: 21

As per Rule 3 (5) of CCR, 2004-

Note : Increase in rate of duty, decrease in rate of duty and sale at higher value is not relevant

payable by the manufacturer if - i) rate of duty on the date of clearance on inputs was 25% ii) rate of duty o

When inputs cleared as such (without using) an amount equal to cenvat credit availed is payable Rs. 5000

Q.22:- A Non Small Scale Industrial unit is required to pay the following Central Excise duties by January 15, 2006 for clearances affected from its factory in respect of final products manufactured during the month

rtain inputs were received on January 1, 2006 on which total duty paid by the uppliers of input was as follows: B.E.D. : Rs. 16,000, E.C. :Rs.320, SHE CESS Rs.160. Excise duty paid n Capital goods received during the month was as follows : B.E.D. : Rs. 40,000, E.C. : Rs. 800, SHE

) the credit available for utilization; (ii) the permissible extent to which such credit available may be utilized against payment of

.E.D., S.E.D., N.C.C.D, EC EC gh ent (P.L.A.)

of December, 2005 –

Basic Excise Duty (B.E.D.): Rs. 36,000; Special Excise Duty (S.E.D.): Rs. 18,000; National Calamity Contingent Duty (N.C.C.D.): Rs. 1,000; Education Cess (E.C.): 2% of B.E.D. + S.E.D. + N.C.C.D. SAH Education Cess (E.C.) : 1% of B.E.D. + S.E.D. + N.C.C.D. – Balances available as credit at the beginning of the month i.e. December, 2005 were as follows :-

B.E.D. : Rs. 24,000, N.C.C.D. Rs. 2,000, E.C. Rs. 600,SHE CESS Rs.300. No inputs were received during the month. However, cesoCESS Rs.400. For the month of December, 2005 you are required to determine : (i

B E.C and SAH ; and (iii) the B.E.D., S.E.D, E.C. and SAH payable throuaccount curr

ANS. 22 Statement Showing Computation of Credit Available For Utilization

Opening Bal month of Dec, 2005 tal Credit Credit available for the To

Page 43: Indirect Taxation CA Workbook

CA. Raj Kumar Practice Booklet s lable for the

month of Dec, 2005 Inputs Capital Good avai

Basic Excise Duty (BED)

24,000 Nil (WN-1) 20,000 (WN-2) 44,000

NCCD 2,000 Nil (WN-1) 2,000

Education Cess (EC) 600 Nil (WN-1) 400 (WN-2) 1,000

SHEC (EC) 300 Nil (WN-1) 200 (WN-2) 500

Total Credit Available 47,500/-

Work

1. mmediately on st

it in respect of inputs received as on 1st Jan, 2006 can’t be said to be available for utilization for payment of duty for the month of Dec, 2005, (in contrast, credit of capital goods received in the month of Dec shall be available for utilization for payment of duty of Dec month).

2. Credit on capital goods is available on the receipt thereof in the factory. The credit is available to the extent of 50% hereon (R t Rules, 20 FY 2005 ll be of Rs. 20,000 (BED), Rs 400 (EC) and Rs. 200 (SHEC), Balance credit shall be available as on 1st April, 2006

util ter.

Statement Showing Utilization of Credit

ing Notes:

So far as availment of credit is concerned, credit in respect of inputs and capital goods is available ithe receipt thereof in the factory. Thus, credit of input can be availed as on 1 Jan, 2006 only as inputs have been received on 1st Jan, 2006. Further, proviso to rule 3(4) of Cenvat Credit Rules, 2004 provides for utilization of available credit. It provides that for payment of duty of any particular month, the credit as available upto the last day of that month can only be utilized. Thus, for payment of duty for the month of Dec, 2005 credit available as on 31st Dec, 2005 can only be utilized. Thus, cred

of duty paid t ule 4(2) Cenvat Credi 04). Thus, for the -06, credit available sha

only and thus, can be ized only thereaf

Duties Credit available for Utilized Credit Credit carry forward utilization

Basic Excise Duty (BED) 3) 0 44,000 (WN- 44,00 Nil

Special Excise Duty (SED) Nil Nil Nil

NCCD 2,000 (WN-2) 1,000 1,000

Education Cess (EC) 1,000(WN-1) 1,000 Nil

SHEC 500 (WN-2) 500 Nil

Working Notes:

Page 44: Indirect Taxation CA Workbook

CA. Raj Kumar Practice Booklet

PLA.

3. Credit be utilized CCD pa Rs 2, of NCCD to the extent of Rs. 1,000 can only be utilized. Surplus credit 000 shall be carr use in future period.

4. Credit of BED can be utilized for payment of any duty of excise (except for Health Cess). Outstanding duties on 00/ 36,000 + SED 18,000 Cess: 100 Thus, credit of

BED can be utilized fully.

Duties Amount payable Amount Paid through

Amount payable through PLA

1. Credit of Ed cess can be utilized for payment of Ed Cess only. Ed Cess payable on final product is 2% of (36,000 + 18,000 + 1,000) i.e. Rs. 1,100/- Thus, Credit of EC to the extent of Rs. 1,000 can be utilized fully. And balance to be paid through

2. Credit of SHEC can be utilized for payment of SHEC only. Ed cess payable on final product is 1% of (36,000 + 18,000 + 1,000), i.e. Rs. 550/-, Thus, credit of SHEC to the extent of Rs 500 can be utilized fully. And balance to be paid through PLA.

of NCCD can for payment of N yable on final product is of Rs 1,

000/- Thus, credit ied forward for

final product is Rs 54,1 - [BED- + NCCD Nil + ED ]. NCCD

Credit

Basic Excise Duty (BED) 0 36,00 36,000 (BED) Nil

Special Excise Duty (SED) 18,000 8,000 (BED) 10,000

NCCD 1,000 1,000 (NCCD) Nil

Education Cess (EC) 1,100 1,000 (Ed Cess) 100

SHEC 550 550 (SHEC) 50

Total Amount Payable through PLA 10,150

Q.23-An assessee was availing SSI exemption from 1-4-2010. He crossed turnover Rs. 150 lakhs on 15-11-010 and started payment of excise duty. He had received machinery on 10-11-2010 one which excise duty aid was Rs. 3,20,000. He intends to avail Cenvat credit of this duty. Can he do so?

edit can be utilised when duty ayable on turnover after Crossing the limit of Rs. 150 lakhs ie as on 15.11.2010.

2p

Ans: 23

Assesses can avail cenvat credit on Capital goods 100% in the Current year, but the crp

Question 24. Solidmec Ltd.Purchased 25 computer systems, eligible as capital goods under the CENVAT Credit Rules

4, on 01.04.2009 paying a duty of Rs. 2,600 on each computer system However, since these systems became , it sold 20 co

,200outda damo Answer 24-

te mputer systems out of 25 on 30.3.2011 at a residual value of Rs. 2,000 each. Determine the unt of CENVAT credit required to be reversed in the financial year 2010-2011.

Notification No. 6/2010 –CE has substituted the second proviso to rule 3(5)with the new proviso which provides s follows:- If the capital goods, on which the CENVAT credit has been taken, are removed after being used,

the manufacturer/output service provider shall pay an amount equal to the CENVAT credit taken on the said Capital goods

a

Page 45: Indirect Taxation CA Workbook

CA. Raj Kumar Practice Booklet e percentage points calculated by arter of a

year or part thereof from the date of taking the CENVAT credit, namely:-

lculated ine method

reduced by th straight line method as specified below for each qu

Type of capital goods Percentage points ca by straight l

For each quarter in entage Perc

Year1 10% Year2 8% Year3 5%

Computers and computer peripherals

1%Year 4&5

Therefore, the amount of CENVAT credit required to be reversed in the financial year 2010-11 is as follows: articulars Amount P

Amount of CENVAT credit availed on 20 computers=Rs2,600 X20 52,000 Less: Amount computed as per second proviso to rule 3(5)(Refer to Note below) 29,120Amount of CENVAT credit required to be reversed in the financial year 2010-11 22,880

the Year Amount of CENVAT credit per proviso to Amount(Rs.)

Note: The computation of amount as per second proviso top rule 3(5):-

availed in each year Amount as rule 3(5)

(01.04.2009-30.03.2010) of Rs.52,000=Rs.26,000 (A)

rs X 10% X4

0,400 50% =10% of (A)X4 Quarte=Rs.26,000=Rs.10,400

1

(01.04.2010-30.03.2011) Balance 50% of Rs. 52,000 =Rs.26,000(B)

8% X4

=Rs.8,320 i)10% of (B)X 4 Quarters

(i)8% of (A)X4 Quarters=Rs 26,000 X

(i=Rs 26,000 X 10% X 4 =Rs.10,400

18,720 29,120*Rule 4(2)(a)restricts the quantum of credit in respect of capital goods received in a factory or in a premises of output service provider at any point of time in a given financial year as 50% in the same financial year and balance in one or

ore subsequent financial years provided the capital goods is still in the possession and use of the manufacturer or the output service provider.

herefore, the CENVAT credit availed to the computer systems would be Rs. 26,000 in year 1 and balance Rs. 26,000 year 2.

m

Tin

Page 46: Indirect Taxation CA Workbook

CA. Raj Kumar Practice Booklet

Problems in SSI Units

Q. 1-ALtd. is a small-scale industrial unit manufacturing a product X. The Annual report for the year 2010-09 of the unit shows a gross sale turnover (net of taxes) of Rs. 2,41,40,000. The product attracted an excise duty rate of 16% as BED

%. Determine the duty liability under Notification Nos.

and Sales Tax 10 8/2003 meant for SSI units

Ans: 1

Gross Turnover 2,41,40000

On First 150 lakhs no duty

E.D. on balance 91.40 lakhs @ 16% 14,62,400

Education Cess 2% 29,248

Higher and Secondary education cess 1% 14,624

Total Duty payable 15,06,272

Q.2-M/s. RPL has three units situated in Bangalore, Delhi and Pune. The total clearances from all these Small Scale units of excisable goods were Rs.450 lakhs during the financial year, 2010-2009. However, the value of individual clearances of excisable goods from each of the said units was : Bangalore Unit Rs.250 lakhs; Delhi Unit Rs.100 lakhs;

e Unit Rs.100 lakhs. Discuss briefly with reference to the Notifications governing small scale industrial undertakings under the Central Excise Act, 1944 whether the benefit of exemption would be available to M/s. RPL for

Ans: 2

and Pun

the financial year, 2009-2010.

Page 47: Indirect Taxation CA Workbook

CA. Raj Kumar Practice Booklet 100 + = 450 lakhs. The Turnover is exceeding 400

lakhs hence SSI benefit not available to RPL Limited.

Q.3- Turnover of SSI for the financial year 2010-09 was as follows.

(a) Clearance made under the own brand name Rs. 110 lakhs

(b) Clearance made under the other brand name on payment of full duty Rs. 180 lakhs

and Scrap Rs. 10 lakhs

uty Rs. 225 lakhs under E/N – 83/94

Worker Charges Rs. 30 lakhs, Cost of material Rs. 120 lakhs

(f) Exports Rs. 90 lakhs

Can unit avail SSI Benefit for the F.Y. 09-10

Ans: 3

Calculation of turnover limit for SSI Benefit

Value of the units of R P L Limited is to be clubbed i.e. 250 + 100 +

(c) Waste

(d) Goods exempt from d

(e) Job work under notification No 214/86 Job

Clearance under own brand name 110 Lacs

Waste and scrap 10 Lacs

Turnover for SSI Limit 120 Lacs

Note-

It is assumed that export made to countries other than Nepal & Bhutan

value of excisable goods viz. i.r.o. n. and Steel articles manufactured by M/s. Alpha Ltd., was Rs. 170 lakhs during the financial year 2010-09. The goods attract 16% ad valorem duty. Determine the excise duty liability when the

SI exe ption notifications

not opt for the benefit of E/n 8/2003- (ie normally book the CCR on input , input Services, Capital goods)

Q.4-The

assessee opts for ‘CENVAT Credit’ and ‘opts for not to avail CENVAT Credit’ underrespectively.

S m

Ans: 4

When Opts for Cenvat Credit/

Full duty payable on whole turnover

Excise duty @ 16% on 170 lakhs 27,20,000

Education Cess 2% 54,400

Higher and Secondary education cess 1% 27,200

Total Duty payable 28,01,600

Page 48: Indirect Taxation CA Workbook

CA. Raj Kumar Practice Booklet

When not Opts for Cenvat credit

No duty on first 150 lakhs and balance 20 lakhs normal duty

Excise duty on 20 lakhs @ 16% 3,20,000

Education Cess 2%

SHE cess 1% 3,200

6,400

Total Duty payable 3,29,600

ores in the financial year 10. Due to recession in the industry, they anticipate a fall in turnover of 20% in 2010-09, when compared to the

year 2007-08 (ii) XYZ Ltd. has started its manufacturing operations in the year 2006-07 with an investment of Rs. 3.5

has achieved turnover of Rs. 2.02 crores during the year ended 10. Normal duty payable on the product is 16%. Find the total excise duty payable by the manufacturer during

the year: (i) if the unit has availed CENVAT Credit (ii) if the unit has not availed CENVAT Credit (The turnover xes and duties)

Q.5-Briefly explain whether the following units are eligible for the benefits under Notification No. 8/2003-CE dated 1.3.2003 during the financial year 2006-07 as Small Scale Industry:

(i) ABC Ltd. had registered a turnover for the purposes of the above Notification of Rs. 3.2 cr2007-20

crores in plant and machinery and hope to achieve a sales turnover of Rs. 2 crores in 2007-08.

Ans: 5

In both the cases previous turnover is less than 400 lakhs hence SSI benefit is available in the current year.

Q.6-A small scale manufacturer having a SSI Unit31.03.20

mentioned above is without ta

Ans: 6

When opts for Cenvat Credit

Full duty payable on whole turnover

Excise duty @ 16% on 202 lakhs 32,32,000

Education Cess 2% 64,640

Higher and Secondary education cess 1% 32,320

Total Duty payable 33,28,960

When not Opts for Cenvat credit

Page 49: Indirect Taxation CA Workbook

CA. Raj Kumar Practice Booklet lakhs and balance 52 lakhs normal duty No duty on first 150

Excise duty on 20 lakhs @ 16% 8,32,000

Education Cess 2% 16,640

Higher and Secondary education cess 1% 8,320

Total duty payable 8,56,960

Q.7-Mr. Karan your old class mate is manufacturing a product in two units’, turnover of unit I and Unit II is Rs. 245 d 280 lakhs during 2010-09. Advice whether SSI benefit available

Ans: 7

lakhs an

to be clubbed i.e. 245 + 280 = 525 lakhs. The turnover is exceeding 400 lakhs hence SSI benefit is not available to Mr. Karan chuadhary.

Q.8-The clearances of Raj Nandini Electric Co. Ltd. were Rs.450 lakh during the financial year 2010-09. The following

Rs.

0

pany is of the view that it is not liable to pay any duty on its clearances in the financial year 2010-09 as per Notification No.8/2003 dated 1 March, 2003. Do you agree with the company? State reasons for your answer. Note that

xemption for the year 08-09 (CS final June 2006)

50 Lacs

Value of the unit I and II is

are included in the said clearances:

(i) Export to Nepal and Bhutan 20,00,000

(ii) Exports to countries other than

Nepal and Bhutan 1,00,00,000

(iii) Job work exempted from duty

under Notification No.214/86 90,00,000

(iv) Sales to 100% EOU against

Form CT-3 50,00,00

The com

the Co. is eligible for SSI e

Ans: 8

Total Value of Clearance 4

Less :Exclusions

Export to other Countries 100 Lacs

Clearance to 100% E O U 50 Lacs

Page 50: Indirect Taxation CA Workbook

CA. Raj Kumar Practice Booklet Job Work 214/86 90 Lacs 240 Lacs

Clearances for SSI benefit 210 Lacs

According to above statement out of the 210 Lacs, 150 Lacs will be exempted by notification no. 8/2003 and balance 60 Lacs will be dutiable

Q.9- A Small Scale unit (SSI) has affected clearances of goods of value Rs. 460. lakhs during the financial he following:

Goods manufactured in rural area with the brand name of others: Rs. 70 lakhs

Write a brief note with reference to the Notification governing SSI under the Central Excise Act, 1944

ANS. 9

year 2006-07. The said clearances include t

Clearances of excisable goods without payment of duty to a 100% EOU unit: Rs. 40 lakhs

Export to Nepal and Bhutan: Rs. 50 lakhs.

Job work in terms of notification No.214/86 CE, which is exempt from duty: Rs. 60 lakhs

whether the benefit of exemption would be available to the unit for the financial year 2007-08.

The SSI exemption is available only if the total value of clearances of excisable goods does not exceed Rs. hall be calculated as follows (Rs. Lakhs):-

400 lakhs. For this purpose, the total value of clearances s

Gross value of clearances 460

Less: Clearance to 100% EOU without payment of duty 40

Less: job work in terms of notification No.214/86-CE, which is exempt from 60 duty

Value of clearances 360

Note:- Export to Nepal and Bhutan and goods manufactured in rural area with the brand name of others are the items not to be deducted.

Assessee eligible for exemption: Since the value of clearances is less than Rs. 400 lakhs, the assessee will be

on no : 214/86 CE, which is exempt from duty – Rs. 75 lacs (iii) Nepal and Bhutan – Rs. 50 lacs (iv) Goods manufactured in rural area with the brand name of the others – Rs.

90 lacs. Examine with reference to the notification governing SSI, under the Central Excise Act whether the benefit of ble to the unit.

Total Value of Clearance 475 Lacs

eligible for SSI exemption for the financial year 2007-08.

Q.10-A SSI unit has effected clearances of goods of the value of Rs. 475 lacs during the Financial Year 2007-08. The said clearances include the following: (i) Clearance of excisable goods without payment of excise duty to a 100% EOU unit. Rs. 120 lacs (ii) Job work in terms of notificatiExport to

exemption would be availa

Ans: 10

Page 51: Indirect Taxation CA Workbook

CA. Raj Kumar Practice Booklet Less : Exclusions

Clearances to 100% E O U 120

Job Work 214/86 75 195 lacs

Total clearances =280 Lacs, therefore the benefit of exemption would be available to the unit.

Problems in Customs Valuation

00 to the raw material supplier. # Landing charges as per Customs provisions # Customs duty rates: BCD - 20%, CVD - xchange rate: 1 Euro = Rs. 42. How much Cenvat can be availed by importer, if he is manufacturer?

ns: 1

Computation of Customs value and Duty payable

Q.1-Determine the assessable value and customs duty amount from the following data: # Name of the raw material—X # FOB value – Euro 1 million # Ocean freight – Actual data not available # Ocean Insurance – Actual data not available # Freight from sea port to godown paid in India Rs. 10,000 # Transit insurance in India – Rs. 2,000 # Selling commission paid to agent in India – 5% # Interest payable on raw material imported at 180 days credit (on FOB value) 12% p.a. # Dividend paid to the foreign supplier of raw material on their equity participation for the year 2001-02 - Rs. 2 per share on 1 million shares of face value Rs. 10/ share. # Importer supplied design and drawings worth Euro 10,0foreign 16%, # E

A

Euro

Particulars

FOB Value of Goods 10,00,000.00

Add :

Freight 20% of F O B 2,00,000.00

Insurance 1.125% of FOB 11,250.00

Selling agent commission in India 5% 50,000.00

Design and drawing 10,000.00

Total 12,71,250.00

Page 52: Indirect Taxation CA Workbook

CA. Raj Kumar Practice Booklet Conversion into Rs. Rs.

Convert into INR 1 euro = 42 (CIF) 5,33,92,500.00

Add: 1% towards landing 5,33,925.00

Total CIF (Assessable Value) 5,39,26,425.00

Basic Customs Duty 20% 1,07,85,285.00

Total 6,47,11,710.00

Countervailing duty – CVD -3 (1)@ 16.48% on 6,47,11,710 1,06,64,489.81

Education Cess 3% on Rs 21449774 (10785285 + 10664489)

BCD + CVD 3(1)

6,43,493.24

Landed Cost India 7,60,19,693.05

Countervailing duty u/s 3(5) @ 4% 30,40,787.72

Total customs duty payable Rs. 10785285 + 10664489.81 + 643493.21 + 2,44,90,562.53

3040787.72

The manufacturer can avail cenvat credit on CVD U/s 3(1) & 3 (5) (Rs. 1,37,05,277.53

1,06,64,489.81 + 3040787.72) =

Q.2-Green Gel Ltd. i pute the education cess payable by the company on the basis of following data:

,000

VD) @ 16%

(iv) Education cess @3%

Ans: 2

Computation of Customs value and Duty payable

mports chemicals from Russia. Com

(i) A.V. u/s 14 (1) Rs. 5,00

(ii) Customs duty @ 16%

(iii) Countervailing duty (C

alue) 5,00,000.00

Total C I F (Assessable V

Page 53: Indirect Taxation CA Workbook

CA. Raj Kumar Practice Booklet Customs Duty payable

Basic custom Duty 16% 80,000.00

Total 5,80,000.00

Counter veiling duty @ 16.48% 95,584.00

Education Cess 3% on

Rs. 175584-(80000 + 95584)

5,267.52

Total Customs duty payable

(80000+95584+5267.52)

180851.52

Q.3-Some spares were imported by air from Germany at CIF value of 1,200 DM, which included airfreight of 380 DM and insurance charges of 20 DM. If exchange rate is 23.40 Rs. = 1 DM, find the Customs Value. Rate of customs duty is 20%, Excise duty chargeable on similar goods in India is 16% as per tariff rate. However, as per an exemption

ion, the effective rate of excise duty is 8%. Find the customs duty payable. How much Cenvat can be availed by , if he is manufacturer?

Ans: 3

Computation of Customs value and Duty payable

notificatimporter

1

C I F value ,200

Less Freight 380

Less Insurance 20

F O B 800

Particulars

F O B Value of Goods 800.00

Add :

Freight 20% of F O B or actual which ever is less Insurance 160.00

Insurance 20.00

Page 54: Indirect Taxation CA Workbook

CA. Raj Kumar Practice Booklet 980Total .00

Conversion into Rs. Rs.

Convert into INR 1 DM = 23.40 (CIF Value) 22,932.00

Add : 1% towards landing 229.32

Total CIF (Assessable Value) 23,161.32

Customs Duty payable

Basic Customs Duty 20% 4,632.26

Sub Total 27,793.58

Countervailing duty u/s 3(1) @ 8.24% on 27793.58 2,290.19

Education Cess 3% on Rs. 6922.46 (4632.26 + 2290.19) 207.67

Total Landed Cost in India 30,291.45

Countervailing duty u/s 3(5) @ 4% 1,211.66

Total customs duty payable Rs. 4632.32 + 2290.19 + 207.67 + 8,341.79

1211.66)

The manufacturer can avail cenvat credit on CVD I/s 3 (1) and 3(5)

cluding EC on CVD u/s 3 (1) Rs. 2290.19 + 1211.66

3,501.85

in

Q.4-Compute the Customs duty from the following data:

Machinery imported from USA by Air (FOB) 8,000 US Dollars

(Electric Motor & others) (FOB) 2,000 US Dollars

es 45., Customs duty on Machinery - 25% ad valorem, Customs duty on Accessory

Accessories compulsorily supplied with Machine

Air Freight 3,000 US Dollars

Insurance 100 US Dollars

Local agents’ commission to be paid in Indian Rupees is Rs. 4,500 (say equivalent to US Dollars 100), The exchange rate is 1 US Dollars = Indian Rupe

Page 55: Indirect Taxation CA Workbook

CA. Raj Kumar Practice Booklet ormal rate 30 % ad valorem), Surcharge on Customs duty - 10%, CVD - 16% ( Effective Rate is 8% by a notification),

SAD - 4%.

Ans:4

Computation of Customs value and Duty payable

(n

FOB Value of Machine 8,000.00

Add : Compulsory supplied accessories along with machine 2,000.00

Total FOB Value 10,000.00

Add : Freight – Actual 3000 or 20% of FOB 2000 (Whichever is less)

2,000.00

Add : Insurance 100.00

CIF 12,100.00

Converting In to INR @ Rs. 45 5,44,500.00

ADD : Local Agent Commission in India 4,500.00

5,49,000.00

Add : 1 % towards landing charges 5,490.00

Total 5,54,490.00

Customs Duty payable

Basic customs duty 25% 1,38,622.50

Total 6,93,112.50

Counter veiling duty @ 8.24%ie CVD 3(1) –on 6,93,112.50 57,112.50

Education Cess 3% on Rs. 195735 (138622.50 + 57112.50) 5,872.00

Landed Cost in India 7,56,097.00

Counter veiling duty u/s 3 (5) @ 4%- on 7,56,097 30,247.88

Total customs duty payable

Rs. 138622.50 + 57112.50 + 30243.88 + 5872 2,31,850.88

Page 56: Indirect Taxation CA Workbook

CA. Raj Kumar Practice Booklet he manufacturer can avail cenvat credit on CVD U/s 3 (1) &

87,356.38T

3(5)

Q.5-An importe ,00,000 Yens. Other details are as follows:

re 1% of FOB value.

a consultancy firm in

India.

rer towards charges for

hange as announced by RBI was: 1 yen = Rs. 0.309

announced by Central Government (Board) by notification under (3)

was 20% Excise duty on similar machinery in India would be 16%.

Find the customs duty payable. How much Cenvat can be availed by importer, if he is manufacturer?

Computation of Customs value and Duty payable

r has imported a machine from Japan at FOB cost of 9

(a) Freight from Japan to Indian port was 18,000 Yens.

(b) Transit insurance charges we

(c) Design and development charges of 90,000 Yens were paid to Japan for design of machinery.

(d) Packing charges of 22,000 Yen were charged extra.

(e) Rs. 20,000 was spent in design cost on machine in

(f) An amount of 98,500 Yen was payable to Japanese manufactuinstallation and commissioning the machine in India.

(g) Rate of exc

(h) Rate of exchange as section 14

(a) (i): 1 Yen = 0.302 Rs

(i) Customs duty

Ans: 5

Particulars Yen

FOB Value of Goods 9,00,000.00

Add :

Freight 18,000.00

Page 57: Indirect Taxation CA Workbook

CA. Raj Kumar Practice Booklet Insurance 9,000.00

Design and development 90,000.00

Packing 22,000.00

Total 10,39,000.00

Conversion into Rs. Rs.

Convert into INR 1 yen = 0.302 (CIF Value) 3,13,778.00

Add: 1% towards landing charges 3,137.78

Total C I F (assessable Value) 3,16,915.78

Customs Duty payable- Basic Customs Duty 20% 63,383.16

Total 3,80,298.94

Countervailing duty- CVD – 3(1) @ 16.48 on 380298.94 62,673.29

Education Cess 3% on Rs. 126056.42 (63383.16 + 62673.26) 3,781.69

Landed Cost in India 4,46,753.89

Countervailing duty u/s 3(5) @ 4% 17,870.16

Total customs duty payable Rs. 63383.16 + 62673.26 + 3781.69 + 1,43,926.58

17870.16)

The manufacturer can avail cenvat credit on CVD U/s 3(1) & 3(5)

cluding EC on CVD u/s 3 (1) Rs. 62373.16 + 17870.16)

80,543.42

in

Q.6-Compute the Customs duty liability as per the provisions of the Customs Act, 1962, from the following information. Make suitable assumptions and indicate the same in your answer: Product Imported - ‘X’ Total FOB Value of the goods - US $ 74000 Quantity Imported - 100 MTs. Ocean freight - US $ 10000 Insurance - US $ 740 Landing charges - 1% of CIF value Exchange rate - 1 US $ = Rs. 37. Date of presentation of Bill of Entry - 28.02.2009 Date of Entry Inwards of

y 30% (ii) Countervailing Duty (Additional Customs duty rates on 3.3.2009 - (i) Basic Customs Duty 25%(ii) Countervailing Duty (Additional Duty)

much Cenvat can be availed by importer, if he is manufacturer?

Ans: 6

Computation of Custom value and Duty payable

the Vessel -

03.03.2009 - Customs duty Rates on 28-2-2009 - (i) Basic Customs DutDuty) 12%. - 8%. How

s

Page 58: Indirect Taxation CA Workbook

CA. Raj Kumar Practice Booklet

Particulars US$

FOB Value of Goods 74,000.00

Add:

Freight 10,000.00

Insurance 740.00

Total 84,740.00

Conversion into Rs. Rs.

Convert in to INR 1 $ = 37 (CIF Value) 31,35,380.00

Add : 1% towards landing charges 31,353.80

Total CIF (assessable Value) 31,66,733.80

Customs Duty payable

Basic Customs Duty 25% 7,91,683.45

Sub Total 3,958,417.25

Counter vailling duty u/s 3(1) @ 8.24% on Rs 3958417.25 3,26,173.58

Education Cess 3% on Rs. 1117857.03 (791683.45 + 326173.58) =

33, 535.71

BCD + CVD 3(1)

Landed Cost in India 43,18,126.54

Countervailing duty u/s 3(5) @ 4% 1,72,725.06

Total customs duty payable (791683.45 + 326173.58 + 33535.71 + 13,24,117.80

172725.06)

The manufacture can avail cenvat credit on CVD u/s 3 (1) and 3 (5)

cluding EC (326173.58 + 172725.06)

4,98,898.64

in

Q.7-Determine the total Customs Duty payable from the following data - Quantity imported: 100 MTs, FOB value: Swiss Franc: 10000, AIR Freight: Swiss Franc: 2500, Insurance: Data not available, Exchange rate: 1 Swiss Franc = Rs. 34, Rate of BCD 30%, Rate of Cenvat under First Schedule to CETA: 16%, Rate of SED under Second Schedule to

Page 59: Indirect Taxation CA Workbook

CA. Raj Kumar Practice Booklet cise (GSI) Act : Rs. 10/kg, Rate of NCCD 1%, How

much Cenvat can be availed by importer, if he is manufacturer?

Ans: 7

Computation of Customs value and Duty payable

CETA: 16%, Rate of AED (GSI) under Additional Duties of Ex

Particulars Swiss Franc

F O B Value of Goods 10,000.00

Add :

Freight 20% of F O B or actual (whichever is less) 2,000.00

Insurance 1.125 of F O B 112.50

Total 12,112.50

Conversion into Rs. Rs

Convert in to INR 1 Swiss France = 34 4,11,825.00

Add : 1% towards landing charges 4,118.25

Total C I F (Assessable Value) 4,15,943.25

Customs Duty payable

Basic Customs Duty 30% 1,24782.98

Total 5,40,726.223

Countervailing duty equal to --

First Schedule of CETA 16.48% 89,111.68

Second Schedule of CETA 16.48% 89,111.68

National calamity contingent duty 1.03% 5,569.48

Addl. ED on GST Rs. 10.30per kg 10,30,000.00

Education Cess 3% on Rs. 13385775.82 (Rs. 124782.98 + 89111.68 + 40,157.27

Page 60: Indirect Taxation CA Workbook

CA. Raj Kumar Practice Booklet 30000) 89111.68 + 5569.48+ 10

Landed Cost in India 17,94,676.34

Countervailing duty u/s 3 (5) @ 4% 71,787.05

Total Customs duty payable (Rs. 124782.98 + 89111.68 + 89111.68 +

569.48 + 1030000 + 40157.27 + 71787.05)

14,50,520.15

5

Q.8-From the following particulars, calculate assessable value and total custom duty payable: (i) Date of presentation of bill CBEC Rs. 44.80

30.6.2009 [Rate of BCD 20%; Exchange 44.00].

ht 500 US Dollars, Insurance cost 100 US Dollars

(vi) Assume there is 4% special CVD.

Ans: 8

Computation of customs value and duty payable

of entry: 20.6.2009 [Rate of BCD 25%; Exchange Rate: Rs. 43.60 and rate notified by ].

(ii) Date of arrival of goods by aircraft in India: Rate: Rs. 43.90 and rate notified by CBEC Rs.

(iii) Rate of Additional Customs Duty: 16%.

(iv) CIF value 2,000 US Dollars; Air Freig[Land charges not ascertainable].

(v) Education cess applicable 3%.

e 2CIF value giv ,000.00

Less Freight 500.00

Less Insurance 100.00

FOB 1,400.00

Particulars DM

FOB value of goods 1,400.00

Add:

Freight 20% of FOB or actual (Whichever is less)

280 or Rs. 500

280.00

Insurance 100.00

Page 61: Indirect Taxation CA Workbook

CA. Raj Kumar Practice Booklet Total 1,780.00

Convert into INR @ 44.80 (CIF Value) 79,744.00

Add: 1% towards landing charges 797.44

Total CIF (Assessable Value) 80,541.44

Customs Duty payable

Basic Customs Duty 20% 16,108.29

Total 96,649.73

Counter veiling duty @ 16.48%- on 96,649.73 15,927.88

Education Cess 3% on Rs. 32.03.16 961.08

(16108.29 + 15927.88)

Landed Cost in India 113,538.69

Counter veiling duty u/s 3 (5) @ 4% 4,541.55

Total Customs duty payable

Rs. 16108.29 + 152927.88 + 961.08 + 4541.55) 36,577.71

The manufacturer can avail cenvat credit on CVD U/s 3 (1) & 3 ) including EC on CVD u/s 3 (1) (Rs 15927.88 + 4541.55)

20,469.42(5

Q.9- Zing Yong of China exports Lithium Cell to India, the FOB price of which is one Dollar for 30 cells; however the details of Fright & Insurance were not made available. Investigation reveals that the goods are imported into India at an increased quantity. Similar cells are manufactured in India, the cost of sales per cell of which indicates the following break-up: Direct 0.25,

erheads Re. 0.50,

The exchange rate 1 $ = Rs. 50. Is there any case to impose Safeguard Duty? If yes, what is

duty leviable? Applicable BCD 25%, CVD 16%, and CVD under section 3(5)

Material Rs. 2.00, Direct Labour Re.

Direct Expenses Re. 0.25,

Indirect Material Re. 0.50,

Indirect Labour Re. 0.25,

Indirect Expenses Re. 0.25,

Administrative Overheads Re. 0.50,

Selling and distribution ov

Profit Margin Re. 0.50.

the

Page 62: Indirect Taxation CA Workbook

CA. Raj Kumar Practice Booklet ns: 9

Computation of Customs value and Duty payable

A

Value of Goods ($ 1= Rs. 50) 50.00

F O B

Add:

Freight – 20% of F O B 10.00

Insurance 1.125% of F O B 0.56

60.56

Add : 1% towards handling 0.61

Total C I F (Assessable Value) 61.16

Customs Duty payable

Basic Customs Duty 25% 15.29

Total 76.45

Counter veiling duty @ 16.48%- CVD 3 (1) 12.60

Education Cess 3% 0.84

Total customs duty payable 28.73

Landed cost = AV + Customs duty 89.89

CVD u/3(5) – 4% on above 3.60

Total Landed cost 93.48

Landed cost per cell – 93.48/30 3.12

Safe guard duty can be levied up to Rs. 1.88 (5.00-3.12)

Working Note Calculation of selling price in India

Direct Material 2.00

Page 63: Indirect Taxation CA Workbook

CA. Raj Kumar Practice Booklet Direct Labour 0.25

Direct Expenses 0.25

Prime Cost 2.50

Indirect Material 0.50

Indirect Labour 0.25

Indirect expenses 0.25

Admin overheads 0.50

Cost of Production 4.00

Selling over head 0.50

Cost of Sales 4.50

Profit 0.50

Selling Price 5.00

Q.10-An importer provided the producer with a mould to be used in production of imported goods. The cost of mould is Rs.5, 00,000 which is expected to produce 25,000 pieces. The importer has imported 5,000 pieces in the first lot. Is it necessary to add the cost of mould in transaction value? If yes, what will be the amount to be added? The importer is expecting an increase in the rate of customs duty next month, so he has requested to the proper officer that if cost of mould is required to be added in transaction value, the full cost of mould, i.e., Rs.5,00,000 may be added in the

n value of first lot of 5,000 pieces itself. Is his demand valid in law ?

alue of mould to be includible in valuation will be---

5

transactio

Ans: 10

V

Total cost of the mould 00000

No units mould should produce 25000

Cost per unit = 500000/25000 20

No units imported 5000

Value includible in valuation = 5000 x 20 100000

Hence the demand is not valid in Law

Q.11-An importer has imported a machine from UK at FOB cost of 10,000 UK Pounds, Other details are as

Page 64: Indirect Taxation CA Workbook

CA. Raj Kumar Practice Booklet follows:

(iii) Design and development charge of 10,000 UK pounds were paid to a consultancy firm

(iv) The importer also spent an amount of Rs. 50,000 in India for development work

(v) Rs. 10,000 was spent in transporting the machinery from India port to the factory of

(vii) Rate of exchange as announced by CBEC (Board) by notification Rs. 68.70 = one UK

s 10%. If similar goods were produced in India, excise duty payable as per tariff is 24 %. There is an excise exemption

ind customs duty payable if (a) Improper is manufacturer using the goods himself.

Ans: 11

Computation of Customs value and Duty payable

(i) Freight from UK to Indian port was 700 pounds.

(ii) Insurance was paid to insurer in India: Rs. 6,000.

in UK.

connected with the machinery.

importer.

(vi) Rate of exchange as announced by RBI was: Rs. 68.82 = one UK pound.

pound.

(viii) Rate at which bank recovered the amount from importer: Rs. 68.35 = one UK pound.

(ix) Foreign exporters have an agent in India. Commission is payable to the agent in India Rupees @ 5% of FOB price. Customs duty payable wa

notification which exempts the duty as is in excess of 16%.

F

U

Particulars K Pounds

FOB Value of Goods 10,000.00

Add:

Freight 700.00

Insurance = 6000/68.70 87.34

Design and development 10,000.00

Total 20,787.34

Convert into INR @ Rs. 68.70 14,28,090.26

Local Agent commission 10000*68.70*5% 34,350.00

Total 14,62,440.26

Page 65: Indirect Taxation CA Workbook

CA. Raj Kumar Practice Booklet es Add: 1% towards landing charg 14,624.40

Total CIF (Assessable Value) 14,77,064.66

Customs Duty payable

Basic Customs Duty 10% 1,47,706.47

Total 16,24,771.13

Countervailing duty @ 16.48% 2,67,762.28

Education Cess 3% on Rs. 415468.75 12,464.06

147706.47+267762.28

Landed Cost in India 19,04,997.47

Countervailing duty u/s 3(5) @ 4% 76,199.90

Total customs duty payable

(rs. 147706.47+267762.28+12464.06+76199.90 4,91,668.65

The manufacturer can avail cenvat credit on CVD U/s VD u/s 3(1). 3(1) & 3(5) including EC on C

(Rs. 267762.28 + 76199.90) 3,43,962.18

Q.12-An actual user imports following goods from England per S. S. Vishal: (1) Second hand numerically controlled horizontal lathe machine - Tariff heading – 84.5811, Value FOB - 1,000/- Pound Sterling (2). A. C. motors - Tariff heading – 85.0110, Value FOB - 500/- Pound Sterling. - - Other relevant data are: - Exchange rate 1 UK Pound = Rs. 65, Freight – 150 UK Pounds, Insurance – 25 UK Pounds. - - Rate of duty : Basic customs duty - 25%, CVD - 16%, SAD - 4%, Ignore landing charges. - - It is found that the lathe machine is undervalued. It is proposed to load the FOB value of the lathe machine by 25%. Party does not want show cause notice and personal hearing. Compute – (i) Assessable value;

duty payable.

ns: 12

Computation of Customs value and Duty payable

(ii) Total

A

U.K

Particulars . Pounds

FOB of the Machinery 1,000.00

Add: Loading 25% since under valued 250.00

Page 66: Indirect Taxation CA Workbook

CA. Raj Kumar Practice Booklet 1Total ,250.00

AC Motor 500.00

Total FOB Value including motor 1,750.00

Add : Freight 150.00

Add : Insurance 25.00

CIF 1,925.00

Conversion into Rs. Rs

Convert 1925 pounds @ Rs. 65 125,125.00

Basic Customs Duty 25% 31,281.25

Total 1,56,406.25

Counter veiling duty @ 16.48% 25,775.75

Education Cess 3% on Rs. 57057 (31281.25+25775.75) 1,711.71

Landed Cost in India 1,83,893.71

Countervailing duty u/s 3(5) @ 4% 7,355.74

Total customs duty payable

R. 31281.25 + 25775.75 + 1711.71 + 7355.74 66,124.45

The manufacturer can avail cenvat on CVD U/s 3 (1) & 3 (5) (Rs

5775.75 + 7355.74)

33,131.49

2

Q.13-Compute (keeping in mind the provisions of the Customs Act, 1962 and Customs Tariff Act, 1975), the total customs duty payable by an importer on goods ‘X’ imported by sea into India, from the following details. You may, wherever appropriate, make suitable assumptions, indicating the same in your answer. - * Value of Goods (FOB) $ 1,000 (Dollars) * Weight of Goods 1,000 Kg * Freight Charges $ 100 (Dollars) * Insurance Charges $ 20 (Dollars) * Handling Charges Rs. 200 * Exchange Rate 4 Dollars = Rs. 100 * Date of Presentation of Bill of Entry - 4.5.2010 * Date of Entry Inwards of Vessel - 1.5.2010 Rates of Customs Duty on 1.5.2010 - * Basic 20% Adv. * Education Cess -3% * Additional (CVD) 15%. * Rates of Customs Duty on 4.5.2010 - * Basic 15% Adv. * Education cess -3%* Additional (CVD) 16%. - . - Note: Special CVD under section 3(5) of Customs Tariff Act is applicable. No other particulars are relevant. How m can be availed by importer, if he is manufacturer

uch Cenvat

Page 67: Indirect Taxation CA Workbook

CA. Raj Kumar Practice Booklet Ans. 13

Computation of Customs value and Duty payable

Particulars US$

FOB Value of Goods 1,000.00

Add:

Freight 100.00

Insurance 20.00

Total 1,120.00

Convert into INR 1 $ (CIF Value) 28,000.00

Add: 1% towards landing 280.00

Total CIF (assessable Value) 28,280.00

Customs Duty payable

Basic Customs Duty 15% 4,242.00

Total 32522.00

Countervailing duty @ 16.48% 5,359.63

Education Cess 3% on Rs. 9601.63 (4242+5359.63) 288.05

Landed Cost in India 38169.68

Countervailing duty u/s 3 (5) @ 4% 1,526.79

Total customs duty payable

(rs 4242 + 5359.63 + 288.05 + 1526.79 11,416.47

The manufacturer can avail cenvat credit on CVD U/s 3 (1) & 3 6,886.42(5) including EC on CVD u/s 3(1) 5359.63 + 1526.79

Q.14-An Indian company imported certain consumer goods from abroad with MRP printed in packing cartons. In respect of similar goods manufactured in India, excise duty is payable on basis of MRP. The importer will be using the goods for further processing. Customs authorities contend that CVD will be calculated based on the MRP printed in the oods. Is this proper? Will your answer be different, if the goods are imported for retail sales?

Ans: 14

g

Page 68: Indirect Taxation CA Workbook

CA. Raj Kumar Practice Booklet If goods imported for further process CVD is payable on the basis of transaction value not on MRP basis. On the other hand: If goods imported for retail sale, CVD payable based on MRP).

Problems in Duty Draw back

Q.1:- ‘“A” has exported under-mentioned goods under drawback claim:

SI No.

of DBK

Description of goods and quantity exported Value FOB (Rs.) Rate of DBK

64.01 Leather footwear. Boots 200 nos @ Rs 1000 per pair

2,00,000 11% of FOB subject to a maximum of Rs 85.00 per pair

Page 69: Indirect Taxation CA Workbook

CA. Raj Kumar Practice Booklet 64.11 Leather chappals 2000 nos @ 50 per pair 1,00,000 3% of FOB subject to a maximum of

Rs. 5.00 per pair

71.01 Brass Jewellery 200 kgs @ Rs 200 per kg 40,000/- Rs 22.50 per kg of brass content

71.05 Plastic bangles with embellishment 200 kgs @ Rs 100 per kg

20,000/- Rs. 5.00 per kg of plastic content

On examination it is found that brass jewellery is 50% of weight and in plastic bangles the plastic contents is 50% but the total weight comes to 190 kgs only. Compute DBK on each item and total drawback.

ANS:-1

STATEMENT OF COMPUTATION OF ADMISSIBLE DBK

Items Actual Qty Exported

FOB value DBK value

DBK amount

Leather footwear Boots

200 pairs 2,00,000 11% of FOB (restricted to Rs. 85.00 per paid)

17,000

Leather chappals 2000 pairs 1,00,000 3% of FOB (restricted to Rs.

5.00 per pair)

3,000

Brass Jewellery 200 kgs (in which Brass content is 50%)

Rs. 22.50 per kg of brass content 2250

Plastic bangles with embellishment

190 kgs(in which Plastic content is 50%)ie 95 kg

Rs. 5.00 per kg of plastic content

475

Total Amount of DBK 22,725.00

Q 2:- ‘A’ exported a consignment under drawback claim consisting of the following items: - (1) 200 pieces of pressure

stoves mainly made of brass @ Rs. 80/piece (Chapter Heading 74.04) – FOB Value Rs. 16,000 – Drawback rate is 4%

of FOB (2) 200 kg Brass utensils @ Rs. 200 per kg (Chapter Heading 74.13) – FOB Value Rs. 40,000- Drawback Rate

is Rs. 24/kg (3) 200 kg Art ware of brass @ Rs. 300/kg (Chapter Heading 74.22) – FOB Value Rs 60,000 – Drawback

Rate 17.50% of FOB subject to a maximum of Rs. 38/per kg. – ON examination in docks, weight of brass artware was

found to be 190 Kgs and was recorded on shipping bill. Compute the drawback on each item and total drawback

admissible to the party.

Ans: 2

S.No. of DBK

Description of goods Rate of Drawback Calculation Drawback eligible

Page 70: Indirect Taxation CA Workbook

CA. Raj Kumar Practice Booklet 74.04 Pressure stoves 4% of FOB Rs.

16000 16000 x 4% 640

74.13 Brass Utensils Rs. 24 per kg 200 x 24 4800

74.22 Art ware of Brass 17.5% of FOB Rs. 60000 or Rs. 38 per kg on 190 KG which ever is less

10500 or Rs. 7220 which ever is less

7220

Total DBK Eligible 12660

Q. 3 -An exporter has exported under-mentioned goods under drawback claim:

S .No of Description FOB Value Rs. Rate of Draw-back

Dbk.

74.24 1000 Kg handicrafts of 2,00,000 16.5% of FOB Value

brass @ Rs. 200 per kg maximum- Rs. 33 per

kg of brass content

74.27 1000 kg of Art ware of 3,00,000 Rs. 33 per kg.

copper @ Rs. 300 per kg

85.81 20,000 pc GLS Lamps @ 1,00,000 1% of FOB

Rs. 5 per piece

Note: 1: On examination it is found that brass content in brass handicrafts is 80%. 2: Art ware has copper content of weight 950 kg. Compute the amount of drawback admissible taking into account the above facts

Ans: 3

S.No. of DBK

Description of goods Rae of Drawback Calculation Drawback eligible

74.24 Handicraft of brass 16.5% of FOB of Rs. 2lacs or Rs. 33 per kg whichever is less

33000 or Rs. 26400 which ever is less

26400

74.27 Art ware of copper Rs. 33 per kg 950 x 33 31350

85.81 GLS Lamps 1% of FOB Rs. 1 lac 100000 x 1% 1000

Page 71: Indirect Taxation CA Workbook

CA. Raj Kumar Practice Booklet Total DBK Eligible

58750

BAGGAGE

SELF STUDY QUESTION

Q.1:- An Indian resident visiting Germany brought following goods while retuning to India (a) His personal effects like cloth etc. valued at Rs. 25,000 (b) Two Liter of liquor of Rs. 1,600 (c) New Camera of Rs. 39, 800. What is the customs duty payable?

Q.2:- An Indian resident goes to Nepal on tour. He purchases colour TV of Rs. 18,000, a laptop computer of Rs. 79,000 and hair dryer of Rs. 2,000 in a duty free shop in Nepal and brings the same to India. What is the duty payable (a) If he returns on 3rd day by air (b) If he returns on 3rd day by land route (c) If he returns on 11th day by air (d) If he returns on 11th day by land route.

Q.3:- Mr. and Mrs. Khanna visited USA and bought a personal computer for Rs. 38,000 and a laptop computer of Rs 98,500 while returning to India, besides their personal effects valued at Rs. 86,000. What is the customs duty payable?

Q.4:- Mrs. & Mr. Kapoor visited Germany and brought following goods while returning to India on 8th February, 2010. (i) Their personal effects like clothes, etc., valued at Rs. 35,000. (ii) A personal computer bought for Rs. 36,000. (iii) A laptop computer bought for Rs.95,000. (iv) Two litres of liquor bought for Rs. 1,600. (v) A new camera bought for Rs. 37,400. What is amount of customs duty payable?

Q.5:- Mr. and Mrs. Bapat visited Germany as tourist and bought a personal computer for Rs. 52,000 and a laptop computer of Rs. 78,000 while returning to India, besides their personal effects valued at Rs. 1,33,000. What is the customs duty payable, if duty on baggage is 35% plus education cesses as applicable.

State level- VAT

Q. 1- Calculate the total tax liability under the state vat law and under the CST Act, for the month of October, 2010 from the following particulars;

PARTICULARS Rs.

Inputs purchased within the state 170,000 Capital goods used in the manufacture of taxable goods 50,000

Inputs purchased from a registered dealer who opt for composition scheme under the provision of the act 10,000

High Seas purchases of inputs 1,00,000

Page 72: Indirect Taxation CA Workbook

CA. Raj Kumar Practice Booklet

Finished goods sold- (a) Within the state 2, 00,000

(b) In the course of inter state trade 2, 50,000

Applicable tax rates are as follows:-

(a) Vat rate on capital goods 12.5% Input tax rate within the state 12.5%

Output tax rates within the state 4%

CST rate 2%

(b) Vat rate on capital goods 4% Input tax rate within the state 4%

Output tax rates within the state 12.5%

CST rate 2%

Ans:-(a)

Statement showing computation of Vat liability

Particulars Amount (Rs.)

Total Output Tax-during the relevant period

VAT payable on Local sales (2,00,000 * 4%) = 8000

+ CST payable on Inter State Sales (2,50,000*2%)= 5,000

TOTAL (A)

13,000

Net Input Tax Credit- during the month

Page 73: Indirect Taxation CA Workbook

CA. Raj Kumar Practice Booklet Opening Balance Nil

ITC accruing during the Jan month

→ ITC on local purchases of inputs (1, 70,000*12.5%)= 21,250 → ITC on capital goods (50,000* 12.5%)= 6,250

(-) ITC reversed during the relevant period Nil

(-) Refunds during the relevant period Nil

TOTAL (B)

27500

Excess credit carried forward for utilisation subsequently 14500

Note :(1) - Inputs purchased from registered dealer who opt for composition scheme –shall not be eligible for ITC

Note :(2) - High seas purchases of inputs -shall not be eligible for ITC because no input tax is payable on high seas sales.

Note :(3) -it has been assumed that under provision of relevant state vat law, ITC on capital goods is admissible in full.

Ans:-(b)

Statement showing computation of Vat liability

Particulars Amount (Rs.)

Total Output Tax-during the relevant period

VAT payable on Local sales (2,00,000 * 12.5%) = 25,000

+ CST payable on Inter State Sales (2,50,000*2%)= 5,000

TOTAL (A)

30,000

Net Input Tax Credit- during the month

Opening Balance Nil

Page 74: Indirect Taxation CA Workbook

CA. Raj Kumar Practice Booklet ITC accruing during the Jan month

→ ITC on local purchases of inputs (1, 70,000*4%)= 6,800 → ITC on capital goods (50,000* 4%)= 2,000

(-) ITC reversed during the relevant period Nil

(-) Refunds during the relevant period Nil

TOTAL (B)

8,800

Vat paid by challan (25,000-10,800) =16,200

CST paid by challan = 5,000

21,200

Note :(1) - Inputs purchased from registered dealer who opt for composition scheme –shall not be eligible for ITC

Note :(2) - High seas purchases of inputs -shall not be eligible for ITC because no input tax is payable on high seas sales.

Note :(3) -it has been assumed that under provision of relevant state vat law, ITC on capital goods is admissible in full.

Question 2. Compute the VAT payable at each stage using invoice method’ from the particulars given below:- Stage Particulars profit (as% of cost price) I Sambhav Medicaids Ltd sold the medicines manufactured By it to the distributors of medicines-Rishabh Pharmacy at Rs.4, 000 -- II Rishabh Pharmacy sold the medicines to the Wholesalers-Suhani Medicos 56.25% III Suhani Medicos sold the medicines to the retailers Galaxy Medicines 25% IV Galaxy Medicines sold the medicines to the ultimate consumers 25% Assume that the VAT rate is 4% and that there was no value addition at various stages of sale except profit margin. Answer 2. Computation of VAT payable:- Stage particulars VAT Liability Less VAT

credit Tax to Government

I Medicines sold by Sambhav Medicaids Ltd .to Rishabh Pharmacy at Rs. 4,000

=4,000X4%=160 -- 160

II Medicines sold by Rishabh Pharmacy to =6,500X4%=260 160 100

Page 75: Indirect Taxation CA Workbook

CA. Raj Kumar Practice Booklet Suhani Medicos at Rs. (4,000+160)X156.25%

III Medicines sold by Suhani to Galaxy Medicines at Rs. (6,500+100)X125%

=8,250X4%=330 260 70

IV Medicines sold by Galaxy to ultimate consumers at Rs.(8,250+70)X125%

=10,400X4%=416 330 86

Author;-In our opinion the above solution as suggested by ICAI is not correct.

The correct solution will be as follows- Stage particulars VAT Liability Less VAT

credit Tax to Government

I Medicines sold by Sambhav Medicaids Ltd .to Rishabh Pharmacy at Rs. 4,000

=4,000X4%=160 -- 160

II Medicines sold by Rishabh Pharmacy to Suhani Medicos at Rs. (4,000)X156.25%

=6,250X4%=250 160 90

III Medicines sold by Suhani to Galaxy Medicines at Rs. (6,250)X125%

=7,813X4%=313 250 63

IV Medicines sold by Galaxy to ultimate consumers at Rs.(7,813)X125%

=9,766X4%=391 313 78

Question 3. Rosesh Ltd. of Gujarat made a total purchases of input and capital goods of Rs. 55, 00,000 during the month of January, 2010 .The following further information is available: (i)Goods worth Rs. 15, 00,000 were purchased from Assam on which C.S.T.@2% was paid. (ii)The purchases made in January, 2010 include goods purchased from unregistered dealers amounting to Rs. 18, 50,000. (iii)It purchased capital goods (not eligible for input credit) worth Rs. 6, 50,000 and those eligible for input credit for Rs. 9, 00,000. (iv)Sales made in Gujarat during the month of January, 2010 is Rs.10, 00,000 on which VAT @12.5% is payable. Assuming that all purchases given are exclusive of tax and VAT@4% is paid on them, calculate (i)the amount of input tax credit available for the month of January,2010 (ii)VAT payable for the month of January, 2010 and (iii)input tax credit carried forward. Note: The input VAT credit on eligible capital goods is available in 36 equal monthly installments. Rs Rs Answer 3. A. Purchases made in January, 2010 55, 00,000 Less: (i) Inter state purchases (input credit not available) 15, 00,000 (ii)Purchase from unregistered dealer (input credit not available) 18, 50,000 (iii)Capital goods (not eligible for input credit) 6, 50,000 40, 00,000 15, 00,000 B.Input tax credit available VAT credit on input @4% 4% of (Rs. 15, 00,000 –Rs.9, 00,000)

Page 76: Indirect Taxation CA Workbook

CA. Raj Kumar Practice Booklet i.e.4% of Rs. 6, 00, 000 24,000 VAT credit on eligible capital goods (4% of Rs 9, 00,000) X 1/36 1,000Input credit available for January, 2010 25,000 c. VAT on sales @12.5% of Rs. 10, 00,000 1, 25,000 Less: Input tax credit 25,000Net VAT payable 1,00,000Input tax credit carried forward to February,2010 Nil

Question 4 (a) Calculate the VAT liability for the period jan.1, 2007 to Jan 31,2007 from the following particulars: Inputs worth Rs.1,00,000 were purchased within the state Rs.2,00,000 worth of finished goods were sold within the State and Rs.1,00,000 worth of goods were sold in the course of inter-State trade.VAT paid on procurement of capital goods worth Rs.1,00,000 during the month at 12.5% . If the input and output tax rate in the State are 12.5% and 4% respectively and the central sales tax rate is 2% show the total tax liability under the State VAT law and under the Central Sales Tax Act. (5 marks) (b)Calculate the net service tax payable under the provision of rule 2A of the Service Tax (Determination of value) Rules, 2006 relating to determination of value of services in the execution of a works contract from the following particulars: (i)Gross amount for the works contract (excluding VAT) Rs.1, 00,000 (ii)Value of goods and materials sold in the execution of works contract Rs. 70,000 (iii)CENVAT credit on (ii) above Rs.1,000 (iv)Service tax paid on input services Rs.1,000 (v)CENVAT credit on capital goods issued in the provision Rs.1,000 of works contract service (vi)Service tax rate 10.36% Make suitable assumptions and provide explanations where required. Answer-4 (a)Computation of the tax liability for the period Jan .1, 2007 to Jan 31, 2007

Statement showing computation of Vat liability

Particulars Amount (Rs.)

Total Output Tax-during the relevant period

VAT payable on Local sales (2,00,000 * 4%) 8000

+ CST payable on Inter State Sales (1,00,000 * 2%) 2000

10000

Page 77: Indirect Taxation CA Workbook

CA. Raj Kumar Practice Booklet TOTAL (A)

Net Input Tax Credit- during the relevant period

Opening Balance nil

ITC accruing during the relevant period (12500 +12500) 25000

(-) ITC reversed during the relevant period nil

(-) Refunds during the relevant period nil

TOTAL (B)

25000

Excess credit carried forward to subsequent period 15000

Note – it is assumed that credit on capital goods is available in the first year. (b) Computation of the Net service tax payable: - Rs. Gross amount charged (excluding VAT) 1, 00,000 Less: Value of goods and materials 70,000 30,000 Service [email protected]% 3,090 Less: CENVAT credit of service tax paid on input service CENVAT credit of excise duty paid on capital goods (50%) [Rs.1, 000+ (50% of Rs.1, 000)][Note 2] 1,500Net service tax payable 1,590 Notes- (1) Cenvat credit of Rs.1, 000 on value of goods and materials sold in the execution of works contract shall not be available to the assessee. (2) CENVAT credit in respect of capital goods shall be taken only for an amount not exceeding 50% of the duty paid on such capital goods in the same financial year and balance 50% shall be available in the subsequent financial year [Rule 4(2) of the CENVAT Credit Rules, 2004. Question-5 1. A dealer purchases the following goods in a State during the month of March 20X6 particulars Total amount

(Rs.) Input Tax Paid (Rs.)

Net Amount (Rs.)

4% VAT Goods 10,40,000 40,00,000 10,00,000 12.5% VAT Goods 9,00,000 1,00,000 8,00,000 VAT Exempt Goods 2,00,000 - 2,00,000 Total 21,40,000 1,40,000 20,00,000 2.The input tax paid on purchase of goods is eligible for VAT credit. 3.Sales made by the dealer during the month are as below:

Page 78: Indirect Taxation CA Workbook

CA. Raj Kumar Practice Booklet Particulars Gross Amount (Rs.) Output Tax

Collected(Rs.) Net Sales Consideration (Rs.)

4% VAT Goods 1,1,44,000 44,00,000 11,00,000 12.5% VAT Goods 10,12,500 1,12,000 9,00,000 VAT Exempt Goods 2,50,000 - 2,50,000 Total 24,06,500 1,56,500 22,50,000 Answer-5

1. The dealer passes the following entry to record the goods purchased and input tax paid thereon: 4%VAT Goods Purchase A/c Dr. Rs. 10,00,000 12,5% VAT Goods Purchase A/c Dr. Rs. 8,00,000

VAT Exempt Goods Purchase A/c Dr. Rs.2,00,000 VAT Credit Receivable (Inputs)A/c Dr Rs. 1,40,000 To Bank A/c Rs.21,40,000 2.The dealer passes the following entry to record the goods sold and VAT collected thereon: Bank A/c Dr. Rs.24,06,500 To 4% VAT Goods A/c Rs.11,00,000 To 12.5% VAT Goods A/c Rs.9,00,000 To VAT Exempt Goods Sales Goods A/c Rs.2,50,000 To VAT Payable A/c Rs.1,56,000 3.The dealer passes the following entry to record the liability for VAT payable met by using the balance in the VAT Credit Receivable (INPUTS)Account and balance by bank. VAT Payable A/c Dr Rs. 1,56,000 To VAT Credit Receivable (Inputs)A/c Rs.1,40,000 To Bank Rs.16,500 Question 6 On June 1, 2009 a dealer purchases one machine in a state for the total cost of Rs. 93,60,000 which includes input tax of Rs. 3,60,000 .As per the state VAT laws, input tax paid on purchase of machinery adjustable as VAT credit over 36 equal monthly installments beginning July 1 2009.Toll the end of the year, the “dealer has to utilized the VAT credit available on the machine. Answer-6 1. The dealer passes the following entry to record the machinery purchased and input tax paid thereon Machinery A/c Dr. Rs 90,00,000 VAT Credit Deferred (Cap Goods)A/c Dr. Rs 3,60, 000 To Bank A/c Rs. 93, 60,000 2. When the VAT credit becomes actually available. The dealer passé the following entry to recognize the same every month. VAT Credit Receivable (Cap Goods)A/c Dr. Rs. 10,000 To VAT Credit Deferred (Cap Goods)A/c Rs. 10,000 3. The dealer depreciation on the cost of machinery excluding VAT Credit (i.e. Rs. 93,60,000 – Rs.3,60,000 = Rs. 90,00,000).

Page 79: Indirect Taxation CA Workbook

CA. Raj Kumar Practice Booklet 4. Balances in VAT credit Deferred (Capital Goods)A/c and VAT Credit Receivable (Capital Goods)A/c are disclosed in the balance sheet as on March 31,2010 as below: Extracts from the Balance Sheet ASSETS Amount (Rs.) LOANS AND ADVANCES VAT Credit Deferred (Cap Goods)A/c 2,70,000 VAT Credit Receivable (Cap Goods)A/c 90,000 Question -7- Operation of VAT system and VAT accounting : XYZ Mfg. Co Ltd of Rajasthan Purchased raw material A from Rajasthan for Rs. 10,400(inclusive of 4% VAT), raw material B from Rajasthan for Rs. 22,500 (inclusive of 12.5% VAT),raw material C from China for Rs. 33,000 (inclusive of 10% import duty and raw material D from Maharashtra for Rs. 15,450 (inclusive of 2% CST). The plant and machinery required for manufacture was purchased for Rs. 2,08,000 (inclusive of 4% VAT).The manufacturing and other expenses (excluding depreciation)were Rs. 61, 550 .The plant is to be depreciated at 100%. The manufacturer’s margin is 20% on cost. The VAT rate on the manufactured product is 4% By way of necessary accounting entries and VAT chart, show the mode of operation of VAT system. Ignore the Central Excise implications, assuming that there is no excise duty on the manufactured product. Answer-7 Raw Material A (net of VAT Rs.400) Raw Material B (net of VAT Rs.2,500)

10,000 20,000

Raw material C (import duty will from part of cost, as it is not available as credit Raw material D (CST will from part of cost, as it is not available as credit) Depreciation on plant and machinery (100% of 2,00,000 i.e. price net VAT of Rs. 8,000) Manufacturing and other expenses

33,000 15,450 200,000 61,550

Cost of the product Add: 20% margin

340,000 68,000

Selling Price Add: VAT @4% of 4,08,000 Cost to the purchaser

408,000 16,320 424,320

VAT payable in cash by the manufacturer = 16,320 – 400 -2,500-8,000 = Rs. 5,420 ACCOUNTING ENTRIES

Dr. Rs

Cr. Rs.

1. Purchase of Raw material A - Raw material A A/c Input Credit A/c To Bank

10,000 400

10,400

Page 80: Indirect Taxation CA Workbook

CA. Raj Kumar Practice Booklet 2. Purchase of Raw material B

Raw material B A/c Input Credit A/c To Bank

20,000 2,500

22,500

3. Purchase of Raw material C Raw material C A/c To Bank

33,000

33,000

4. Purchase of Raw material D Raw material D A/c To Bank

15,450

15,450

5. Purchase of Plant and Machinery i.e. capital goods Plant and machinery A/c Input Tax Credit A/c To Bank

200,000 8,000

208,000

6. Manufacturing and other expenses Manufacturing and other expenses A/c

To Bank

61,550

61,550

7. Depreciation Depreciation A/c To Plant and machinery A/c

200,000

200,000

8. Sale of manufactured product Bank A/c To Sales A/c To VAT payable A/c

424,320

408,000 16,320

9. Purchase of Raw material C VAT payable A/c To Input Tax Credit A/c To Bank

16,320

10,900 5,420

Question -8 Determine the TAXABLE TURNOVER,INPUT TAX CREDIT AND NET VAT PAYABLE by a Works contractor from the details given below on the assumption that the Contractor maintains sufficient records to quantity the labour charges Assume Output VAT at 12.5% Rs. Lakhs

(i) Total Contract price (excluding VAT) 100 (ii) Labour charges paid for execution of the contract 35 (iii) Cost of consumables whose property does not get 5 transferred in execution of works contract (iv) Material purchased and used for the contract taxable at 12.5% VAT (VAT included) 45 The contractor also purchased a plant for use in the contract for Rs. 10.4 lakhs . In the VAT invoice relating to the same VAT was charged at 4% separately and the sold amount of Rs. 10.4 lakhs in inclusive of VAT .Assume 100% input credit on capital goods .

Page 81: Indirect Taxation CA Workbook

CA. Raj Kumar Practice Booklet Make suitable assumption whenever required and show the working notes.

Answer-8

COMPUTATION OF TAXABLE TURNOVER &VAT LIABILITY Total contract Value (exclusive of VAT) 100 lakhs Less: Value of “Labour, service and other charges” (40 lakhs) (i) Value of Labour Charges 35 lakhs (ii) Cost of Consumable whose property does 5 lakhs

Not get tfd in execution of works contract 60 lakhs VAT liability on this taxable turnover shall be computed as follows: [60,00,000 *12.5%]= 7,50,000 Assumptions

1. As per the provisions, profit earned by the Work contractor to the extent it is relatable to supply of labour and services shall also be deductible as “value of labour, service and other charges”. For the purpose of given question ,it has been presumed that the amount of labour charges and consumable already includes the profit attributable thereto.

2. It has been assumed no declared goods were used in execution of works contract. In fact ,the given question specifically provides for taxing entire taxable turnover @12.5%

COMPUTATION OF INPUT TAX CREDIT A. Credit of INPUTS used in execution of Works Contract

(45, 00, 000 * 12.5/112.5) 5, 00,000 B. Credit of CAPITAL GOODS (10,40,400 * 4/104) 40,000

5,40,000

COMPUTATION OF NET VAT LIABILITY Gross VAT liability (60 Lakhs * 12.5%) 7, 50,000 Less: Input Tax Credit (as computed above) (5, 40,000) NET VAT LIABILITY 2,10,000 Question 9- VAT payable: Purchases by S&Co. for the month of December are as follows: (1)Rs. 1,00,000 at 4% (2)Rs. 5,00,000 at 12.5% VAT Sales of s&co. for the month of December are as follows:

(1) Sales of Rs. 3,00,000 at 4% VAT (2) Sales of Rs. 3,00,000 at 12.5% VAT

Compute eligible input tax credit and VAT payable for the month. Answer-9 (a)Computation of input tax credit and VAT payable (amounts in Rs.)

4000 62500

66500 12000 37500

Input Tax paid by S& Co on Rs. 1,00,000 @4% Input Tax paid by S& Co on Rs. 5,00,000 @12.5% Total input tax credit eligible Output tax payable on Rs. 3,00,000 @4% Output tax payable on Rs. 3,00,000 @12.5% Total tax payable 49500

Page 82: Indirect Taxation CA Workbook

CA. Raj Kumar Practice Booklet Net input tax allowed to be carried over (Rs. 66,500 –Rs. 49,500) VAT payable by S&Co.

17000 NIL

Question 10- Vat liability at different stages : Manufacturer A extracted raw product X and raw produce Y from mines at Rs. 10,000 and rs. 15,000 respectively and sold the same at 100% margin to Manufacturer B (VAT rate is 4% on produce X and 12.5% on produce Y ). Manufacturer B used X and Y as raw material and sold the resultant product for Rs. 2,00,000 to wholesaler C (VAT rate is 4%). Whole sale C sold the same to Retailer D at 25% above cost (VAT rate is 4%). The retailer D sold the same to a consumer at 20% above cost (VAT rate is 4%).Compute the amount of VAT payable in cash by each person. Answer-10 VAT CHART (all amounts in Rs.) 1.VAT payable in cash by Manufacturer A 4% on produce X i.e. 4% of 20,000 (10,00,000 +100% of 10,000) 800 12.5% on produce Y i.e. 12.5% of 30,000 (15,000 + 100% of 15,000) 3,750

4,550

2.VAT payable in cash by Manufacturer B VAT@ 4% on sale price of Rs.2,00,000 8,000 Less: VAT credit on raw produce X&Y 4,550

3,450

3.VAT payable in cash by Wholesaler C VAT@4% on sale price of Rs. 2,50,000(2,00,00+ 25% of 2,00,000) 10,000 Less: VAT credit on purchases from manufacturer B 8,000

2,000

4.VAT payable in cash by Retailer D VAT@4% on sale price of Rs. 3,00,000(2,50,00+ 20% of 2,50,000) 12,000 Less: VAT credit on purchases from Wholesaler C 10,000

2,000

Total VAT paid on the Government 12,000 The above illustration shows that the VAT paid to the Government at various stages (here, Rs. 12,000)is equal to the VAT collected from the ultimate consumer (here Rs. 12,000 or 4% of 3,00,000) Question-11 Determine the TAXABLE TURNOVER &NET PAYABLE by a Works contractor from the details given below:

(i) Total contract price (excluding VAT): Rs.10,00,000 (ii) Labour, Services & other charges :- Not ascertainable, Applicable % = 25% (iii) Material whose property get transferred in execution of works contract: (a) Declared Goods[Purchase Price: Rs. 2,00,000 (VAT @4% extra)] (b) Other than declared goods [purchase price : Rs. 4,00,000 (VAT @12.5% extra)]

Answer-11

COMPUTATION OF TAXABLE TURNOVER & VAT Liability Total contract Value (exclusive of VAT) 10,00,000 Less: Labour, Services & other charges (25%) (2,50,000)

Taxable Turnover (value of material sold in execution of works contract) =7,50,000

Page 83: Indirect Taxation CA Workbook

CA. Raj Kumar Practice Booklet

VAT liability on this turnover shall be compute as follows: 1. Taxable Turnover pertaining to Declared Goods 9,600/-

[2,40,000 * 4%] 2. Taxable Turnover pertaining to Declared Goods 63,750/-

[5,10,000 * 12.5%] =73,350

Working Notes: Presuming reasonable profit margin of 20% in the given case, taxable Turnover pertaining to Declared goods

shall be Rs. 2,40,000/- How to compute taxable turnover pertaining to goods other than Declared goods?

That shall be compute as balancing figure , i.e. [Total taxable Turnover in execution of works contract-taxable Turnover pertaining to declared goods]

In our case, that comes to [7,50,000 -2,40,000]=Rs. 5,10,000/- COMPUTATION OF NET VAT LIABILITY Gross VAT liability (as computed above) 73,350/- Less: Input Tax credit (58,000) [(2,00,000* 4%)+(4,00,000 * 12.5%)] NAT VAT LIABILITY 15,350/-

Page 84: Indirect Taxation CA Workbook

CA. Raj Kumar Practice Booklet

SSSEEERRRVVVIIICCCEEE TTTAAAXXX

Section: - 67. VALUATION OF TAXABLE SERVICES

Question -Computation of service tax: Ms Nandini rendered a taxable service to a client. A bill for Rs. 40,000 was raised on 29.4.2009; Rs 15,000 was received from the client on 1.5.2009 and the balance on 23.5.2009 .No service tax was separately charged in the bill .The questions are: (i)Is Ms. Nandini liable to pay service tax, even though the same has not been charged by her? (ii)In case she is liable, what is the value of taxable service and the service tax payable? Solution :(i) Yes, she is liable to pay service tax, as the gross amount charged/ amount received by her is deemed to be inclusive of service tax. (ii)The value of taxable service is to be computed in accordance with the following formula:-

Gross amount charged (inclusive of service tax)X 100 Value of taxable service = (100 +Rate of service tax) i.e.110.3 Hence, - Value of taxable service =40,000 X 100÷ 110.3 36,265 Service [email protected]% on the aforesaid value 3,735

Section 68: Procedure for payment of service tax

Question: Mr. AJAR, a chartered accountant, raised an invoice for Rs. 27,575 (25,000+2,575 service tax) to a client on 20.1.2010.The client, however, has paid a lump-sum of Rs. 26,000 on 28.4.2010 for full and final settlement. (i)How much service tax Mr. AJAR has to pay and when does this tax become due to payment? (ii)What will be his liability, if the client refuses to pay service tax and pays only Rs 25,000 in total?

(May 2007, 4 marks) Ans; When the full bill amount (inclusive of service tax) is not received and payment is received in part only, then , in view of provisions of section 67 of the Act, the amount so received is deemed as inclusive of service tax. Therefore, the service tax liability shall be computed by making back-calculations. The Effective rate of service tax is 10.3% (10% service tax +3% education cess on service tax)

Page 85: Indirect Taxation CA Workbook

CA. Raj Kumar Practice Booklet The value of taxable service is to be computed in accordance with the following formula-

Gross amount charged (inclusive of service tax) X 10.3 Service tax = (100+ Rate of service tax) i.e. 110.3 Hence, (i) If payment received in Rs. 26,000 Service tax @ 10.3% on the aforesaid value = 26,000 X 10.3 ÷ 110.3

Rs. 2,428

Hence, (ii) If payment received in Rs 25,000 Service tax @ 10.3% on the aforesaid value =25,000X 10.3 ÷110.3%

Rs. 2,335

Due Date: Such service tax should be paid by 5th July ,2010

Question .Mr.Happy , a service provider, has provided services of Rs. 1,00,00,000. Out of this, Rs. 70,00,000 are taxable output services and Rs. 30,00,000 are exempt output services. Mr. Happy has opted not to maintain separate inventory and accounts and pay prescribed amount on value of exempt output services.Servcie tax paid on his input services, excluding education cess and secondary and higher education cess (EC & SAHEC)is Rs. 6,00,000 which do not include any service specified in rule 6(5) of the CENVAT Credit Rules , 2004,Rate of service tax, excluding .EC and SAHEC , is 10% .Calculate the total amount payable including Service tax, EC and SAHEC by Mr. Happy by GAR -7 challan (CA Final New Syllabus May 2010). Answer: Service tax payable on Rs 70 Lakhs @ 10% is - Rs 7, 00,000 In addition, education cess @2% is- Rs 14,000 and SAHE cess is- Rs. 7,000. As per rule 6(3)(ii) of Cenvat Credit Rules, assessee can avail proportionate Cenvat Credit i.e. service tax (70% of Rs. 6,00,000 )- Rs. 4,20,000 Education cess would be - Rs. 8,400 and SHE cess - Rs 4,200 Thus, assessee will, be required to pay tax by GAR – 7 challan as follows- (a) Service tax – Rs 2, 80,000 (b)Education cess – Rs 5,600 (c) SAHE cess – Rs 2,800.

Question: ABC Ltd. (liable to pay service tax) has agreed to render services to Mr. XYZ. The following are the chronological events:

Contract for services entered into on 31.08.2010 Advance received towards value of taxable service in September, 2010 towards all services 80,000 Total value of services, billed in February, 2011 (exclusive of service tax) 2,50,000 Above includes non-taxable services of 90,000

Page 86: Indirect Taxation CA Workbook

CA. Raj Kumar Practice Booklet Balance amount is received in March, 2011 When does the liability to pay service tax arise and for what amount? Contract contains clear details of services: Consideration and service tax are charged separately, as mutually agreed upon. Solution: Total value of taxable service (2, 50,000- 90,000) =1, 60,000 Total value of taxable service =90,000 2, 50,000 The liability to pay service tax arises at the time of receipt of advance in September, 2010 and at the time receipt of balance consideration in March 2011.Servcie tax is payable as soon as any advance is received as the taxable service includes “service to be provided”and payments received, before during or after the provision of taxable services form part of the gross amount charged for the taxable services. Further, the liability to pay service tax arises only upon the receipt of the value of taxable services and not when the bill is raised. Advance Portion Rs. Advance received towards all services in September,2010 80,000.00 Amount billed for taxable services = 2,50,000 – 90,000 1,60,000.00 Advance received towards taxable services = 80,000 X (1,60,000/2,50,000) 51,120.00 Service tax @10% (since, service tax is charged separately)= 51,200 X 10% = 5,120.00 Education cess @3% 153.60 Total service tax liability 5,273.60 Rounded off u/s 37D 5,274.00 In this case, the due date for payment of service tax will be 5th October,2010 Balance Portion Rs Balance amount received in March 2010 =2,50,000-Rs.80,000 1,70,000.00 Amount received towards taxable services = 1,70,000 X (1,60,000/2,50,000) 1,08,800.00 Service Tax @10% = 1,08,800 X 10% 10,880.00 Education Cess @3% 326.40 Total service tax liability 11,206.40

Rounded off u/s 37D 11,206.00

In this case, the due date for payment of service tax will be 31st March, 2011

Question -State the divergence between accounting and taxation principles in relation to recognition of revenue in case of service tax (RTP June 2009) Ans -As per accrual concept of accounting the value of taxable services performed should be recognized during the period in which it has been performed even though the payment for the same has not been received in that period. However, as per Rule 6(1) of Service Tax Rules, 1994 service tax is payable only on the value of taxable services received i.e.; the liability to pay service tax arises only after the receipt of the value of the services. The following entries are suggested in order to avoid any conflicts with the service tax provisions (a)At the time of billing to the client:

Client A/c Dr. To Service Income A/c

Page 87: Indirect Taxation CA Workbook

CA. Raj Kumar Practice Booklet To Service tax deferred Liability

(b) On receipts of payment

Bank a/c Dr. To client A/c

(c)At the time of realization of the consideration:

Service tax Deferred Liability A/c Dr. To service tax payable A/c

(d) On payment of service tax

Service tax payable A/c Dr. To CCR (Rec.) To cash A/c

Question. Mr.X has let out one building on rent to a bank for commercial purpose on 01.04.2007 @ Rs.2,00,000 p.m. The services have become taxable w.e.f. 1st June 2007.Mr. X has received rent in advance on the 1st of every month.Mr.X is not rendering any other services. The rent was increased w.e.f.1st September 2010 to Rs.2,20,000 p.m. Rent for the month of Feb and March 2011 was received on 7th April 2011. Amount of rent is exclusive of service tax. Compute service tax liability of Mr.X for the financial year 2010-11 and also determine the date upto which service tax should be paid. Solution: Presume in the above question the amounts are inclusive of service tax. Since gross receipt in the earlier has exceeded Rs. 10, 00,000 hence service tax is payable .Since the service provider is an individual, service tax shall be paid on quarterly basis in the manner given below: April to June 2010 Since amount charged is inclusive of service tax hence amount of service tax shall

be =6,00,000 / 110.3 X 10.3 = 56,029.01 Rounded off under section 37D = 56,029 Last date for making payment shall be 5th July 2010 and if payment is through internet banking, last date shall be 6th July 2010

July to September 2010 Since amount charged is inclusive service tax hence amount of service tax shall be =6,20,000 / 110.3 X 10.3 = 57,896.65 Rounded off under section 37D = 57,897 Last date for making payment shall be 5th October 2010 and if payment is through internet banking, last date shall be 6th October2010

October to December 2010 Since amount charged is inclusive of service tax hence amount of service tax shall be =6,60,000 / 110.3 X 10.3 = 61,631.91 Rounded off under section 37D = 61,632 Last date for making payment shall be 5th January 2011 and if payment is through internet banking, last date shall be 6th January 2011

January to March 2011 Since amount charged is inclusive of service tax hence amount of service tax shall be =2,20,000 / 110.3 X 10.3 = 20,543.97

Page 88: Indirect Taxation CA Workbook

CA. Raj Kumar Practice Booklet Rounded off under section 37D = 20,544 Last date for making payment shall be 31th March 2011

Question. Mrs. Bose is rendering taxable services, which were brought into the service tax net w.e.f. 1.5.2009.The following information are made available to you: - (i)Amount received on 10.5.2009 for services provided in April , 2009 – Rs. 2,00,000 (ii)Advance received from one client on 10.5.2009 – Rs. 3,39,000 (iii)For balance services of Rs. 7,00,000 bill was raised on 12.3.2010 and the amount due was received from the above client on 15.3.2010 (iv)Other taxable services billed and received during 1.5.2009 to 31.3.2010 – Rs. 4,00,000 (v)Value of free services rendered in October .2009 – 1,50,000 .—Compute the value of taxable services and service tax payable for the year ended 31.3.2010 (ICWAI Inter June 2010 New Syllabus)

Answer:

-Service tax is not payable on Rs. 2,00,000 .

-Service tax is payable on Rs. 3,39,000 Plus 7,00,000 plus 4,00,000 .

-No service tax is payable on free services .

Thus, service tax is payable on Rs. 16,39,000. Mrs. Bose can claim exemption of first Rs 10 lakhs and pay service tax @10.30% on Rs 6,39,000 i.e. Rs 65,817.

Question X& Co. received the following amounts (i) Date of Receipt 20.04.2009 –Rs .1,00,000 for services rendered in July , 2009 (ii)Date of Receipt 30.06.2009 – Advance for services to be rendered Rs. 5,00,000 .services were rendered in July and August , 2009 (iii)Date of receipt 5.8.2009 – Rs.50,000 .For services rendered in March,2009 (iv)Date of receipt 10.09.2009 – Advance for service Rs. 3,50,000.A sum of Rs. 50,000 was refunded in April,2010 after termination of agreement. For the balance amount, service was provided in September, 2009. Compute: (i)The amount of taxable service for the first two quarters of the Financial Year 2009-10 (ii)The amount of Service tax payable.

Answer: Service tax is payable on receipt basis. Service tax is payable on advance received also. Hence,for quarter April-June 2009, value of service on which tax is payable is Rs. 6,00,000.For quarter July-September 2009,value of service is Rs. 4,00,000.

These amounts are to be taken as inclusive of service tax and back calculations should be made.

Quarter Amount received Service tax liability April-June 2009, 20-04-2009 30-06-2009

1,00,000 5,00,000 6,00,000

6,00,000*10.3/110.3=56029 Service tax -54397.09 Education cess @ 2%-1087.94

Page 89: Indirect Taxation CA Workbook

CA. Raj Kumar Practice Booklet SHE Cess -543.97

July-September 2009 5-08-2009 10-09-2009

50,000 3,50000 4,00,000

4,00,000*10.3/110.3=37353 Service tax -36,264.72 Education cess @ 2%-725.30 SHE Cess -362.65

In respect of amount of Rs. 50,000 refunded in April 2010, if the amount of service tax is also refunded, the service tax amount so refunded can be adjusted against service tax liability arising during April – June 2010.

Question . Ms. Priyanka, a proprietress of Royal Security Agency received Rs 100,000 by an account payee cheque , as advance while signing a contract from proceeding taxable services ; she receive Rs. 5,00,000 by credit card while providing the service and another Rs. 5,00,000 by a pay order after completion of service on January 31,2009 .All three transactions took place during financial year 2008-09.She seek your advice about her liability towards value of taxable service and the service tax payable by her.

Answer:

-She is liable on entire Rs 11 lakhs, presuming that she is not eligible for exemption as small service provider.

-The Rs.11lakhs are to be taken as inclusive of service tax and service tax is payable by back calculations.

- Service tax @10.30% would be Rs. 1, 02,720.

SECTION 69: Read Rule 4 of service tax rules, 1994

Question: RegistrationI B, : State briefly, whether following persons are liable to apply for registration under Finance Act, 1994 and the Service tax (Registration of Special Category of Persons)Rules, 2005 and if so, from which date: (a)An input service distributor who starts his business with effect from 1st January, 2009. (b)A provider of taxable services was Rs. 6,00,000 up to 31.3.2009 (CA Final Nov . 2008 (NS)4 Marks) Ans -The answer in view of service tax (Registration of Special Category of Persons)Rules, 2005 is- (a)Liable for registration, application to be made upto 31-1-2009 (b)A provider of taxable service under a registered brand name of another person is liable to apply for registration within a period of 30 days of commencement of business of providing taxable service, as basic exemption for small service provider is not available to a service provider who provides taxable service under a trade name or brand name, whether registered or not, of another person, otherwiase the limit of Rs. 9 lakh shall apply.

Question; With reference to the Finance Act, 1994 as amended and rules made thereunder relating to service tax, state whether registration is required or not in the case of the following persons or class of persons (i) Input service distributor (ii)Small service provider whose aggregate value of taxable service is Rs. 9,50,000 per annum.

Page 90: Indirect Taxation CA Workbook

CA. Raj Kumar Practice Booklet (iii)India based recipient of taxable services provided from abroad by a non-resident not having any place of business in India (Nov,2006,1X3 =3 Marks) Ans Yes, registration is required in all the 3 cases. Question:Registration: Discuss whether the following persons are liable to apply for registration under the service tax law and if yes, by which date- (a)A provider of taxable service, whose aggregate value of taxable services is Rs. 8,80,000 upto 31-3-2010; (b) A provider of taxable service, whose aggregate value of taxable services is Rs. 9,01,000 upto 1-1-2010; (c) A provider of taxable service, who has provided services as follows:’ Aggregate value of services upto 31-8-2009 (i.e. before the service became taxable)Rs. 5,00,000 Aggregate value of taxable services from 1-9-2009 to 31-3-2010 Rs. 8, 95,000 (d)A provider of taxable service who starts his business w.e.f. 11-8-2009 and whose aggregate value of taxable services as on 10-10-2009 becomes Rs 9,02,000; Ans: The solution is as follows:- (a)Not liable for registration, as aggregate value of taxable services doesn’t exceed Rs 9 Lakhs; (b) Liable for registration ,application to be made upto 31-1-2010; (c)Not liable for registration, as aggregate value of taxable services doesn’t exceed Rs 9 Lakhs; (d)Liable for registration, application to be made upto 9-11-2009

General Exemptions From the levy of Service Tax

Question : A partnership firm, (liable to pay service tax) gives the following particulars relating to the services provided to various clients by them for the half-year ended on 30.09.2010:

(i)Total bills raised for Rs. 9,75,000 (exclusive of service tax)out of which bill for Rs. 95,000 was raised on an approved International Organization and payment of bill for Rs.1,20,000 were not ,received till 30.09.2010.

(ii)Amount of Rs. 70,000 (exclusive of service tax)was received as an advance from Z Ltd .on 25.09.2010 to whom the services were to be provided in October,2010

You are required to work out the: (a)Taxable value of services (b) Amount of service tax payable Solution: Rs Rs. Total bills raised - 9,75,000 Less: Bill raised for an approved international organization (not liable to service tax)

95,000 -

Amount not received till 30.9.2010 1,20,000 2,15,000 7,60,000 Add: Amount received in advance 70,000 Taxable value of services 8,30,000 Service tax @10% 83,000 Add: Education Cess@3% 2,490 Total service Tax Liability 85,490

Page 91: Indirect Taxation CA Workbook

CA. Raj Kumar Practice Booklet Assumption -appropriate amount of service tax also received with rhe value of service received -consideration of service is received in service completion ratio. Question; Computation of taxable value and service tax: Exemption & Advance receipt: J.C.Professionals, a partnership firm, gives the following particulars relating to the services provided to various clients by them for the half-year ended on 30-9-2009 (1)Total bills raised for Rs. 8, 75,000 out of which bill for Rs. 75,000 was raised on an approved international Organization and payments of bills for Rs. 1,00,000 were not, received till 30-9-2009. (2)Amount of Rs. 50,000 was received as an advance from XYZ Ltd on 25-9-2009 to whom the services were to be provided in October, 2009 You are required to work out the: a. Taxable value of services b. Amount of service tax payable. Solution (a)-Assuming that bill amount is exclusive of service tax, Bill amount Rs. 8,75,000

– Exemption in respect of services provided to International Organization Rs. 75,000

– Amount not received ,hence , not liable to service tax Rs. 1,00,000

+Amount received in advance , hence, liable to service tax Rs 50,000

the taxable value of services =Rs 7,50,000. (b)Accordingly, service tax payable =10.3% of 7, 50,000 =Rs. 77250 Question : Mr.X an architect who is liable to pay service tax has submitted information as given below: (i) Received advance Rs. 2,00,000 from ABC Ltd on 10th April 2010 and service were rendered in September 2010 and bill was issued in October 2010. (ii)Rendered services to Mr. A in October 2010 and a bill of Rs.5,00,000 was issued inclusive of service tax and payment of Rs. 3,00,000 was received in February 2011 and balance is yet to be received. (iii)Rendered services to United Nations in December 2010 and payment of Rs. 7,00,000 was received in December 2010 . No service tax has been collected. (iv) Rendered services to a unit in SEZ and Rs. 3,00,000 was received in March 2011 without service tax. (v)Rendered services to the Reserve Bank of India in April 2010 and payment of Rs. 9,00,000 was received in August 2010. No service tax has been collected. (vi) Rendered services to foreign diplomatic mission and Rs. 5,50,000 was received in January 2011 without service tax.

Page 92: Indirect Taxation CA Workbook

CA. Raj Kumar Practice Booklet (vii)Rendered service tax to family members of diplomatic agents and Rs. 1,50,000 was received in October 2010 without service tax. Solution (i) April to June 2010 Taxable value of services = 2,00,000

Service tax payable = 2,00,000 /110.3 X 10.3% = 18.676.34 Rounded off under section 37D =18,676 Last date for making payment shall be 5thJuly 2010 and if payment is through internet banking, last date shall be 6th July 2010

(ii) January to March 2010 Total bills raised = 5,00,000 Less: Amount received = 3,00,000 Amount not received = 2,00,000 Since amount charged is inclusive service tax hence amount of service tax shall be =3,00,000 / 110.3 X 10.3 = 28,014.51 Rounded off under section 37D = 28,015 Last date for making payment shall be 31st March 2011.

(iii)Services provided to United Nations or an international organization is exempt from service tax. (iv)Services provided to a developer of SEZ or a unit of SEZ is exempt from service tax. (v)Services provided to the Reserve Bank of India are exempt from service tax. (vi Services provided to foreign diplomatic mission are exempt from service tax. (vii)Services provided to family members of diplomatic agents are exempt from service tax. Question : With reference to Notification No. 6/2005 St dated 1.3.2005, answer the following questions (RTP Nov.2008 Old) (i) What is the threshold limit of service tax exemption for small service providers? (ii)Is the exemption mandatory to service provider in all cases? (iii)Whether credit on capital goods received during the period of exemption limit can be availed after crossing of exemption limit? (iv)On opting for exemption in the beginning of the financial year, whether the credit of the duty paid on inputs lying in stock has to be reversed? (v)If balance in CENVAT credit is available after the above said reversal, whether the same can be carried forward? (vi)In a case where the service provider provides more than one taxable service, does the exemption limit apply to individual taxable services or all taxable services? Ans (i) Rs 10 Lakh: (ii) No, it is optional; (iii) Yes, credit of capital goods cannot be availed before crossing the exemption limit, but, the same can be availed only after crossing the exemption limit; (iv) Yes. (v)No, the same shall be lapse on service provider availing this exemption ; (vi) It applies to all taxable services provided by an individual service provider.

Page 93: Indirect Taxation CA Workbook

CA. Raj Kumar Practice Booklet SERVICE TAX RULES RULES, 1994

Question –Hotel Royal Regency charges 10% of the bill amount as service charges and Department has asked them to pay service tax on it. The assessee has submitted that the amount @10% collected from customers is subsequently disbursed among the staff, therefore it is not part of their income and cannot be included in the gross amount charged by them. Examine the case and advise suitably.

Solution: As per section 67, the value of taxable service shall be the gross amount charged by service provider for the taxable service provided or to be provided by him. Since the assessee was charging service charges (@10% of bill amount) from its customers for providing the services, therefore, the said amount was liable to be included in the gross amount charged. Therefore, service charges (@10% of bill amount) charged by Hotel Royal Regency are includible in gross amount charged by them and will be liable to service tax.

Question- Pure agent: Mr.X, a custom house agent, charges Rs 1,00,000 from a client. This sum includes Rs. 10,000 towards payment of customs duty on behalf of the client.

Compute the service tax to be charged from the client. Solution: -Expenses incurred by the service provider in his capacity as pure agent do not form part of the value of taxable service. -Here, the custom house agent Mr X has paid Rs. 10,000 towards customs duty on behalf of the client, which is an amount paid by Mr.X in his capacity as ‘pure agent’.

-Since the sum of Rs 10,000 doesn’t include any element of profit, the same will not form part of value of taxable service.

-Hence, taxable value =1,00,000 -10,000 =Rs 90,000.Service tax to be charged = 90,000 X 10.3% =Rs 9,270 Question; pure agent: Mr. y, an architect, finds a client Mr.A who needs services of architect as well as interior decorator. Mr. A asks Mr. Y to find an interior decorator on his behalf , decide the terms of his engagement and also his remuneration ,make payment to him on his(Mr.A’s behalf)and provide a consolidated bill, showing relevant break-ups, to him (Mr A ).Mr.Y finds a interior decorator Mr.Z ,whose remuneration is fixed at Rs. 2,00,000 (inclusive of all taxes).Mr.Y charges Rs 8 Lakhs towards the value of taxable services provided by him.Compute the amount of service tax to be charged in the bill of Mr. Y if- (a)the bill amount includes Rs 2,00,000 additional towards the cost of interior decorator; or (b) the bill amount includes Rs 2,50,000 additional towards the cost of interior decorator Solution : Service tax shall be computed as follows: CENVAT Credit treatment: The interior decorator Mr. Z

Page 94: Indirect Taxation CA Workbook

CA. Raj Kumar Practice Booklet (a) In this case , bill amount includes only the actual cost of interior decorator. -Since Mr. Y has acted as pure agent of Mr. A , he will get deduction of such cost. -Therefore, value of taxable service provided by Mr. Y =10,00,000 – 2,00,000 = Rs 8,00,000 , -on which service tax = 8,00,000 X 10.3% = 82,400

will pay service tax =Rs 2,00,000 X 10.3 ÷ 110.3 =Rs 18,676 .The client Mr. A can claim Cenvat Credit of Rs. 82400+18676 = Rs 101,076

(b)In this case, bill amount includes amount in excess of the actual cost of interior decorator, -hence, Mr. Y will not be ‘pure agent’ of Mr. A and thus, he will not deduction of such cost. -Therefore, the value of taxable service provided by Mr.Y = Rs. 8,00,000 + 2,50,000 = Rs. 10,50,000 , -on which service tax = 10,50,000 X 10.3% = Rs. 108,150

CENVAT credit treatment: The interior decorator Mr. Z will pay service tax =Rs 2,00,000 X 10.3÷110.3 =Rs 18,676 .The services provided by Mr. Z will be treated as input service in case of Mr. Y and Mr. Y can get CENVAT credit of Rs. 18,676, which can be used for payment of his service tax liability. The client Mr. A can claim Cenvat Credit of Rs . 108,150

Penalties Question; Mr. X a chartered Accountant who is liable to pay service tax has submitted particulars as given below: (a) (i) Rendered services in May 2010 and issued bill for Rs. 1, 20,000 inclusive of service tax. (Out of which Rs.75, 000 was received by a cheque on 10th August 2010 and balance on 3rd March 2011). (ii)Rendered services in the month of June 2010 in connection with scrutiny assessment and a bill of Rs. 75,000 were issued. (iii)Rendered free services in July 2010 (market value Rs. 20,000) (iv)Rendered services to different clients in the month of January 2011 and bills of Rs. 7,00,000 was issued inclusive of service tax but only Rs. 5,00,000 was received in the same month in full and final settlement. (v) Received Rs. 70,000 in advance in March 2011 for services to be rendered in April 2011. Compute amount of service tax payable for each quarter and also the last date upto which tax should be paid for the financial year 2010-11 (b)what If, the return for the half year ending September 2010 was filed on 18th October 2010 return for half year ending March 2011 was filed on 27th April 2011,compute penalty payable for each of the return and also determine the last date upto which revised return can be filed. Solution : (a)

Page 95: Indirect Taxation CA Workbook

CA. Raj Kumar Practice Booklet April to June 2010 Nil

July to September2010 Since amount charged is inclusive of service tax hence amount of service tax shall be =75,000 / 110.3 X 10.3 = 7,003.63 Rounded off under section 37D = 7,004 Last date for making payment shall be 5th October2010 and if payment is through internet banking, last date shall be 6th October 2010

October to December 2010 Nil January to March 2011 Since amount charged is inclusive of service tax hence amount of service tax shall be

=5,00,000/ 110.3 X 10.3 = 46,690.84 Rounded off under section 37D = 46,691 Since amount charged is inclusive of service tax hence amount of service tax shall be =45,000/ 110.3 X 10.3 = 4,202.18 Rounded off under section 37D = 4,202 Amount Received in the month of March 2011 =70,000 Service tax payable=70,000/ 110.3 X 10.3 = 6,536.71 Rounded off under section 37D = 6,537 Last date for making payment shall be 31st March 2011.

(b)If the above case there is a delay of 10 days in payment of tax. Compute interest payable under section 75 and penalty under section 76. April to June 2010 Nil

July to September2010 Interest Payable under section 75 7,004 X 13% X 10/365 = 24.95 Penalty payment under section 76 2% of the amount of default for 10 days = 2% X 7,004 X 10/31 =45.19 Penalty calculated @Rs 200 per day for 10 days = Rs. 2,000 Penalty liable to be paid is Rs. 2,000

October to December2010 Nil January to March 2011 Interest Payable under section 75

57,430 X 13% X 10/365 = 204.54 Penalty payment under section 76 2% of the amount of default for 10 days = 2% X 57,430 X 10/31 =382.87 Penalty calculated @Rs 200 per day for 10 days = Rs. 2,000 Penalty liable to be paid is Rs. 2,000

(b)

Page 96: Indirect Taxation CA Workbook

CA. Raj Kumar Practice Booklet If the return for the half year ending September 2010 was filed on 18th October 2010, there is no penalty because return is filed within prescribed time period. Revised return can be submitted within a period of 90days from the date of submission of the original return. If return for the half year ending March 2011 was filed on 27th April 2011, the penalty shall be Rs. 500. Revised return can be submitted within a period of 90 days from the date of submission of the original return. Question; Mr. X is rendering services which are exempt from service tax but the services have become taxable w.e.f.16.07.2010 and he has received Rs. 2,00,000 upto 15.07.2010 and total receipt upto 31.03.2011 is Rs. 32,00,000. He has crossed the limit of Rs. 11, 00,000 on 20.12.2010 and limit of Rs. 12,00,000 bon 27.01.2011.(total receipts) He has taken input services October 2010 and paid Rs. 3,00,000 plus service tax and again he has taken input services in February 2011 and has paid Rs. 4,00,000 plus service tax.. His services are covered u/s 194C of Income Tax Act and are subject to TDS @11% and each of the individual bill was exceeding Rs. 30,000. The bills issued are inclusive of service tax. Compute service tax, and also the last date upto which he should apply for registration under Rule 4 and if registration was delayed by 10 days what will be the penalty under section 77. If service tax payment was delayed by 20 days, compute interest u/s 75 and penalty u/s 76. If return was delayed by 20 days , compute penalty under Rule 7C. Solution : If any service has taken any services and has paid service tax. SERVICE TAX credit shall be allowed service tax paid on input services but if service provider is not liable to pay service tax, no SERVICE TAX credit is allowed. Similarly, if the service provider is claiming exemption for Rs. 10,00,000 , no SERVICE TAX credit shall be allowed for service tax paid by him on input services unless he has achieved the limit of Rs. 10,00,000. In the given case no SERVICE TAX credit is allowed for input services taken in the month of September 2010. He will be allowed SERVICE TAX credit for input services availed in March 2011 because at that time he is liable to pay service tax because the limit of Rs. 10, 00,000 has already been crossed. Rs. Gross receipts 32,00,000.00 Less: Amount received when services were exempt 2,00,000.00

Less: General exemption 10,00.000.00 20,00,000.00 Output tax (20,00,000/110.3 X 10.3) 1,86,763.37 Less: SERVICE TAX credit (4,00,000 X 10.3%) (41,200.00) Net service tax 1,45,563.37

Page 97: Indirect Taxation CA Workbook

CA. Raj Kumar Practice Booklet Rounded off u/s 37D 1,45,563.00 Last date for registration = 19.01.2011

In case of delay, penalty shall be = 200 X 10 =2,000 or 5,000 whichever is higher i.e. Rs. 5,000.

Last date of payment of service tax = 31.03.2011

Interest payable under section 75

1,45,563 X 13% X 20/365 = 1,036.89

Penalty payable under section 76

1,45,563 X 2% X 20/30 = 1,940.84 Penalty calculated @Rs 200 per day for 20 days = 4,000 Penalty liable to be paid is Rs. 4,000

In case of delay in furnishing the return, penalty shall be Rs. 1,000 under Rule 7C. Assumption – payment is received on the basis of work completed.

Page 98: Indirect Taxation CA Workbook

CA. Raj Kumar Practice Booklet

TTTAAAXXXAAABBBLLLEEE SSSEEERRRVVVIIICCCEEESSS

Banking and Financial services

Question Answer Surveillance fee collected by a credit rating agency; (May 2006,2 Marks)

Taxable ,Includible in the value of taxable service]

Examine liability to service tax: Services provided by one scheduled bank to another scheduled bank in relation to inter – bank transactions of purchase and sale of Foreign Currency .(Marks 2,May 2010)

Exempt under Banking and other financial services]

With reference to banking and other financial services, state whether service tax is applicable in the following cases: (i)Services provided by the State Bank of India to the Central Board of Direct Taxes in relation to collection of advance income-tax (ii)Discount charged by SB Ltd., a non-banking financial company, on the facility of bill discounting provided by it. Such discount is shown separately in the bill issued for the purpose (iii)Rich Bank, a schedule Bank, purchases foreign currency from Generous Bank, another Scheduled Bank

Exempt in all the three cases.

Services provided by a banking company or financial institutions to Government of India or a State government

Banking company or financial Institutions have been exempted from service tax payable on the service provided to Government of India or a State Government for collection of any duties or taxes. Other services are liable to tax]

Discuss whether service provided by a money changer in relation to exchange of foreign currency is a forex broking service chargeable to service tax under ’Banking and Other Financial Services’.

Taxable under ’Banking and Other Financial Services’]

Depository services and Electronic Access to Securities Information Services (EASI) provided by the central Depository Services India ltd (May 2008,2 Marks)

Taxable under Banking and Other Financial Services]

The services provided by a money changer in relation to dealing of foreign currency (buying or selling), at specified rates, without separately charging any amount as commission for such dealing, is not liable to service tax as foreign exchange broking under ‘banking and

The statement is incorrect. Purchase or sale of foreign currency, including money changing services are liable to service tax under ’Banking and Other Financial

Page 99: Indirect Taxation CA Workbook

CA. Raj Kumar Practice Booklet other financial services’.(RTP June 2009) Services’]

Is service tax payable on entry and exit load charged by a mutual fund from the investor?

No. Not liable under Banking and other financial services]

Construction of Residential Complex

Question; construction of residential complex having ten bungalows (Nov.2006) Ans. Not taxable: Construction of residential complex having only 12 or less residential units is not covered within the scope of ‘Construction of residential complex service’. Thus, construction of residential complex having 10 bungalows would not be chargeable to service tax.

Question; Construction services provided in respect of a residential complex having 12 residential units (RTP Nov. 2006) Ans.; Residential complex for the purpose of ‘Construction of Residential Complex Service’ complex comprising of a building or buildings, having more than 12 residential units. Therefore, construction services provided in respect of a residential complex having 12 residential units shall not be liable to service tax.

Question The contract for construction is for Rs. 8, 00,000 which includes job charges plus material which is not supplied by contractor .The customer has agreed to supply steel to contractor, the value of which is Rs. 2,00,000.It value is not considered in quotation given by contractor. The land belongs to customer .Its value is Rs. 4,00,000.

Answer: Total value of contract (including material supplied by customer, but excluding value of land)is Rs. 10,00,000.The contractor can opt to pay duty @33% on total value i.e. on Rs. 3,30,000 .Service tax payable will be 10% of Rs. 3,30,000 and education cess will be 2% plus SAH education cess of 1% .The contractor cannot avail any Cenvat credit.

If value of land is not shown separately, then service tax will be payable @25% of gross amount which will be include value of land plus value of material supplied by customer.

Business auxiliary services

Question Answer Explain whether the infrastructural support services provided by ABX Ltd. Shall be liable to service tax. (RTP Nov . 2007)

Taxable under ‘Business Support Services’

Whether the following services are taxable: Services in relation to production of alcoholic liquor on job work basis. (Marks 2, May 2010 (NS))

Yes: Alcoholic liquor is non- excisable goods. Hence, its manufacture on job work basis is liable to service tax under Business Auxiliary Service

whether job work is liable to service tax? (RTP NEW Nov. 2008)

[Hint: Business Auxiliary Service, covers job work as “production or processing of goods for, or on behalf of the client”. Thus job work is liable to service tax. However, if the job work amounts to ‘manufacture’ of ‘excisable goods’ under the Central Excise Act, 1944, then, it is not liable to service tax as the same has been specifically excluded from the definition of

Page 100: Indirect Taxation CA Workbook

CA. Raj Kumar Practice Booklet ‘Business Auxiliary Service’

State the conditions for availing the exemption from payment of service tax in respect of production or processing of goods for or on behalf of the client (RTP May 2006)

Exempt under ‘Business Auxiliary Services’

Service provided by an individual in respect of procuring the inputs and managing the inventory for a company (RTP May 2005)

Taxable under Business Auxiliary Service.

Hindustan Info systems is engaged in study, analysis, design and programming of information technology software. The said services are taxable under the category ‘business auxiliary services’ (RTP June 2009)

Incorrect. The services of study, analysis, design and programming of information technology software are taxable under ‘Information Technology software services’]

Commission received by distributors for distribution of mutual funds units (CA Final June 2009, 2 Marks)

Taxable under Business Auxiliary Services, as commission agent

Discuss applicability of Service Tax: Information Technology Services (May 2004, 2 marks)

Taxable under Information Technology Software service. Job work in relation to Information Technology Services (i.e. Auxiliary Services) are taxable under ‘Business Auxiliary Service’ Various other information technology related services are also liable to service tax. Detailed coverage of all such services in the main book under this service.

Transport of goods by road service Goods Transport Agency)

Question: GTA Service: calculate the value of taxable service of ‘X’ Transport Company engaged in the business of transport of goods by road .Give reasons for taxability or exemption of each item. No freight is received from any of the specified category of consignor/consignee.suitable assumptions may be made whenever required. X does not avail CENVAT credit: 1. Total freight charges received by ‘X’during the year 13,50,000 2. Freight charges received for transporting fruits 1,25,000 3. Freight collected small consignment for persons Who paid less than Rs. 750 for each consignment 75,000 4. Freight collected for transporting goods in small vehicles for persons Who paid less than Rs. 1,500 per trip 1,50,000 (CA Final (New Scheme )June 2009,7 Marks) Ans Computation of value of taxable service: (normal case ie reverse charge is not applicable) Total freight collected 1350000 Less: Freight charges received for transporting fruits (Services in relation to transport of Vegetables, eggs or milk by road is exempt from service tax)

125000

Less: Freight collected less than Rs 750/-For each consignment (This service is exempt , as the freight doesn’t exceed Rs 750 for each individual consignment)

Page 101: Indirect Taxation CA Workbook

CA. Raj Kumar Practice Booklet 75000

Less: Freight collected for transporting goods in small vehicles for persons who paid less than Rs. 1,500 /- Per trip (This service is exempt is the freight on all consignments transported into a goods carriage doesn’t exceed Rs 1,500]

150000

Value of taxable service/ Gross amount charged 1000000 Less: Abatement of 75% on gross amount charged / value of taxable service

750000 Value on which service tax is to be charged 250000 Service tax @10.3% (assuming that no SSI –exemption has been claimed / is available) 25750

Page 102: Indirect Taxation CA Workbook

CA. Raj Kumar Practice Booklet

PAST EXAMINATION QUESTIONS

Note –Strike through portion not relevant in the present scenario

NOV- 2008- NEW SYLLABUS

PART-A

Question 1 (a)How will the assessable value under the subject transaction be determined under section 4 of the Central Excise Act 1944? Give reasons with suitable assumptions where necessary.

Page 103: Indirect Taxation CA Workbook

CA. Raj Kumar Practice Booklet Contracted sale price for delivery at buyer’s premises Rs.9, 00,000. The contracted sale price includes the following elements of cost: (i)Cost of drawing and designs Rs.4,000 (ii)Cost of primary packing Rs.3,000 (iii)Cost of packing at buyer’s request for safety during transport Rs.7,000 (iv)Excise duty Rs.1, 11, 200 (v)VAT (Sales Tax) Rs.37,000 (vi)Octroi Rs.9,500 (vii)Freight and insurance charges paid from factory to “Place of removal” Rs.20,000 (viii)Actual freight and insurance from ‘Place of removal ’to buyer’s premises Rs.42,300 (4 marks) (b)Following transactions took place in the factory of JKA Ltd: (i)An imported consignment of raw materials was received vide bill of entry dated 2.12.07 showing the following customs duty payments: Basic customs duty Rs.25, 000; Additional Duty (CVD) Rs.20, 000; Special additional duty Rs.5, 800 (ii)A consignment of 1,000 kg of inputs was received. The excise duty paid as per the invoice was Rs.10, 000.While the input was being unloaded, and 50 kg were damaged and were found to be not usable. (iii)A vehicle containing machinery was received.The machinery was purchased through a dealer and not from the manufacturer. The dealer’s invoice no.925 dated 3.4.07 marked ‘original for buyer’ certified that the excise duty paid by the manufacturer of machinery was Rs.24, 000. The dealer is registered with the Central Excise Authorities. (iv)Some inputs for final product were received. These were accompanied by a certified Xerox copy (photo copy) of invoice no.286 dated 15.1.08. indicating that excise duty of Rs.6,400 had been paid on inputs. The original or duplicate copy of invoice was not traceable. Indicate the eligibility of CENVAT credit, in each case; under the CENVAT credit Rules,2004 with explanations where necessary. (c) CTL Ltd has a manufacturing unit situated in Lucknow.In the financial year 2006-07, the total value of clearances from the unit was Rs.450 lakh. The break up of clearances is as under: (i)Clearances worth Rs.50lakh of certain non-excisable goods manufactured by it. (ii)Clearances worth Rs.50 lakh exempted under specified job work notification. (iii)Exports worth Rs.100 lakhs (Rs 75 lakh to USA and Rs.25 Lakh to Nepal). (iv)Clearances worth Rs.50 lakh which were used captively to manufacture finished products that are exempt under notifications other than Notification No.8/2003 –CE dated 1.3.2003 as amended. (v)Clearances worth Rs.200 lakh of excisable goods in the normal course. Explain briefly ,the treatment for various items and state, whether the unit will be eligible for the benefits for exemption under Notification No.8/2003 –CE dated 1.3.03 as amended for the year 2007-08. (7 marks) Answer

(a) Computation of assessable value of the excisable goods:- Contracted sale price Rs.9, 00,000

Page 104: Indirect Taxation CA Workbook

CA. Raj Kumar Practice Booklet Less: Excise duty (Note-1) Rs.1,11,200 VAT (Note-1) Rs.37,000 Octroi (Note-1) Rs.9, 500 Actual Freight from ‘Place of removal’ to buyer’s Premises (Note-2) Rs.42,300 Rs.2, 00,000 Assessable value Rs.7, 00,000 Notes- (1) In the given question, for the purpose of determining the assessable value of the excisable goods:- The duty of excise, sales tax and other taxes, if any, actually paid or payable on the excisable goods shall be excluded [Section 4(3)(d)of the Central Excise Act,1944]. (2) The cost of transportation from the place of removal up to the place of delivery of the excisable goods shall be deducted [Rule 5 of the Central Excise Valuation (Determination of Price of Excisable Goods) Rules, 2000] The cost of transportation, worth Rs.20, 000, from the factory to the place of removal shall not be excluded [Explanation2 to rule 5 of the Central Excise Valuation (Determination of Price of Excisable Goods) Rules,2000] Cost of packing Rs.3,000 and Rs.7, 000 shall not be deducted. In this regard, it has been clarified that as per section 4 of the Central Excise Act,1944, packing charges shall form the part of the assessable value whether packing is ordinary or special, or primary or secondary. Any charges recovered for packing are the charges recovered in relation to the sale of the goods under assessment and hence, will form part of the transaction value of the goods [Circular no.354/81/2000]. (b) (i) As per rule 3(1)(vii) of the CENVAT Credit Rules,2004,CENVAT Credit of the additional duty leviable under section 3 of the Customs Tariff Act, 1975 viz., Countervailing Duty (CVD) shall be allowed to a manufacturer or producer of the final products .Thus, credit can be availed in respect of Rs.20, 000 paid as additional duty (CVD) and Special additional duty Rs.5, 800. (ii) Rule 2(k)of the CENVAT Credit Rules,2004 interalia provides that input means all goods used in or in relation to the manufacture of final products, whether directly or indirectly. Thus, the inputs lost before being issued for production cannot be termed as “Used in or in relation to manufacture of final product”. Hence; CENVAT credit of Rs.9, 500 on balance 950 kg of inputs can be availed. (iii) Rule 9 (1)(a) of CENVAT Credit Rules,2004 provides that CENVAT credit shall be taken by manufacturer on the basis of an invoice issued by a first/ second stage dealer .

Further as per rule 9 of the Central Excise Rules, 2002, a first/second stage dealer requires registration. Thus, in the given case, CENVAT credit can be claimed against dealer’s invoice since the dealer is registered

with Central Excise Authorities. (iv) Rule 9 of CENVAT Credit Rules, 2004 states that the CENVAT credit can be availed on the basis of invoice issued by a manufacturer.

Thus, the credit can be availed on any copy of the invoice i.e., whether it is the invoice marked as “Original for buyer” or ‘’duplicate for transporter” as it still an invoice issued.

However, a certified photocopy is not an invoice issued by the manufacturer. Thus; credit cannot be availed on the basis of a certified copy.

Page 105: Indirect Taxation CA Workbook

CA. Raj Kumar Practice Booklet (c) In order to claim the benefit of exemption under Notification No. 8/2003 in a financial year, the total turnover of a unit should not exceed Rs.400 lakh in the preceding year, the total turnover of a unit should not exceed Rs.400 lakh in the preceding year. For the purpose of computing the turnover of Rs.400 lakh:-

(I) Turnover of non-excisable goods has to be excluded. Therefore; clearances of non-excisable goods of worth Rs.50 lakh shall be excluded. (ii)Clearances exempted under job work notifications should not be considered. Therefore, exempted clearances of Rs.50 lakh under job work notification will be excluded. (iii)Export turnover has to be excluded. However, export to Nepal and Bhutan cannot be excluded as these are treated as “clearance for home consumption”.Therefore.clearances worth Rs.75 lakh exported to USA will be excluded while clearances worth Rs.25 lakh exported to Nepal will be included. (iv)Value of intermediate products manufactured has to be included if the final product is exempt under any notification other than Notification No.8/2003 –CE dated 1-3-2003*.Therefore , clearances worth Rs.50 lakh which were used captively to manufacture finished products exempt under notifications other than Notification No.8/2003 will be included. (v)Clearances of excisable goods of Rs.200 lakh in the normal course will be considered.

Therefore, for the year 2007-08 , the turnover of CTL Ltd, for claiming the SSI exemption will be:- =Rs.450 lakh-(Rs.50 lakh + Rs. 50 lakh+Rs.75 lakh) =Rs.275Lakh

Since the turnover is less than Rs.400 lakh, CTL Ltd, will be eligible for exemption under Notification No.8/2003. Note –It is assumed that the value of clearances of final products manufactured from such intermediate products is not included in the total turnover of rs.450 lakh of the unit.

Question 2 (a)M/s.Ganga Marketing supplies 12 bottles of mineral water in a single package to Speed Airways (Airline Company) Maximum retail price was printed on the package .However, individual bottle of 200ml.each did not carry such maximum retail price (M.R.P.) as these were to be distributed to the passengers by the airline company and not intended for resale. M/s.Ganga Marketing pays duty of excise assessing the goods under section 4 of the Central excise Act,1944. The Department has taken a view that the package of 12 bottles is not a wholesale package. The airline company itself is the ultimate consumer. Hence, the package of 12 bottles itself is a ‘retail package’ and duty is payable on the basis of MRP under section 4A of the Central excise Act, 1944. Examine briefly, with the help of decided case law, if any, whether the stand taken by the Department in correct in law. (5 marks) (b)M/s. Om processors, a job worker, was engaged in the processing of manmade fabrics received from the principal supplier. The job worker (assessee) had undertaken to discharge all the duty liabilities Under the Central Excise Act, 1944.The assessee received manmade fabrics on declaration from the principal supplier that the said fabrics had polyester content below 70%; processed the same and cleared the processed fabrics claiming the benefit of concessional rate of duty available by the Department, it was found that the fabrics contained polyester in excess of 70% and thus would attract higher rate of duty. A show cause notice was issued invoking the extended period of limitation under section 11A of the Central Excise Act,1944 demanding differential duty and penalties on the ground of mis-declaration on the part of the assessee. Briefly discuss, with reference to decided case law, whether the stand

Page 106: Indirect Taxation CA Workbook

CA. Raj Kumar Practice Booklet taken by the department is correct in law. (5 marks) (c) M/s.Raj Fibres had filed an appeal to the High Court on Aug.11,2008 under section 35G of the Central Excise Act,1944 aggrieved by an order passed by the Appellate Tribunal .The order appealed against was received by the assessee on Jan. 1, 2008.The High Court dismissed the appeal petition on the ground that the same had been filed beyond the period provided for filing an appeal under section 35G of the Act and the Court had no power to condone the delay .M/s Raj Fibres urged before the High Court that the provisions of the Limitation Act,1963 should be made available and the delay in presenting the appeal ought to be condone. State briefly, with reference to decided case law. if any, whether the High Court could Condon the delay in presenting the appeal pursuant to the provisions of the limitation Act,1963 as urged by M/s.Raj Fibers. (5 marks) Answer No, the stand taken by the department is not valid in law.

Section 4A(2) of the Central Excise Act,1944 stipulates that value of the goods notified by the Central Government under section 4A (1)of the Act shall be the retail sale price declared on such goods less such amount of abatement , if any ,from such retail sale price as the Central Government may allow.

For the purpose of valuation under section 4A of the Central Excise Act, 1944, there should be requirement under the provisions of the Standards of Weights and Measures Act, 1976 or the rules made there under or any other law to declare the retail price of such goods on the package.

Standards for weights and measures (Packaged Commodity) Rules, 1977 have been amended to provide inter

alia that MRP is not required to be printed in case of sale to institutional consumers. Institutional consumers have been defined as those consumers who buy packaged commodities directly from

the manufacturers/packers for service industry like airways, railways etc. Thus, Speed Airways, being an institutional consumer, package of mineral water bottles meant for them is not

required to bear any MRP. Hence, in the present case, the goods are to be valued under section 4 and not under section 4A of the Central

Excise Act, 1944. (b)Proviso to section 11A (1)of the Central Excise Act,1944 stipulates that ate extended period of limitation can be invoked only in case of short payment of duty by reason of fraud, collusion , or willful mis-statement or suppression of facts with the intention to evade the payment of duty. Similar view was expressed by the Apex Court in case of Padmini Products (1989) (SC),

Wherein it was held that failure to pay duty might not necessarily be due to fraud or collusion or willful misstatement or suppression of facts or contravention of any of the provisions of the Act.

If the facts of the case revealed that the appellant had acted bona fide, such act would not attract the penal provisions under section 11A of the Act. If the facts were otherwise, then the penalty would be leived.

In the given case,

There was no requirement for the assessee (processor) to verify the correctness of the declaration filed by the principal (suppliers).

Further; there was no allegation that the assessee was a party to such mis-declaration by the principal supplier. Therefore, extended period of limitation could not be invoked against the assessee. Thus; the action taken by the Department is not valid in law. (c)

Page 107: Indirect Taxation CA Workbook

CA. Raj Kumar Practice Booklet The Apex Court in the case of CCE v.Punjab Fibers Ltd.(2008) 223 ELT 337 (SC) has held that there is no provision for condonation of delay in section 35H of the Central Excise Act,1944 i.e., The High Court was not empowered to condone delay in filing reference application .The Court also observed that the right or privilege to claim benefit of a provision for condonation of delay could be governed only by the law in force at the time of delay. Though the above –mentioned case pertains to a reference application under section 35H of the Central Excise Act, 1944, the principal enunciated therein may also be applied to an appeal filed under section 35G of the Act as section 35G also does not provide for any condonation of delay in filing an appeal. Hence, in the instant case, the High Court is justified in holding that the court has no power for condonation of delay in filing an appeal. In the present scenario above question is not relevant because sec 35G has been amended as follows---

SECTION 35G APPEAL TO HIGH COURT

The Commissioner of Central Excise or the other party aggrieved by any order passed by the Appellate Tribunal

may file an appeal to the High Court and such appeal shall be-

(a) filed within 180 days from the date on which the order appealed against is received by the Commissioner of Central Excise or the other party;

(b) The High Court may admit an appeal and memorandum of cross objection after the expiry of the period of 45 days /180 days respectively, if it is satisfied that there was sufficient cause for not filing the same within that period.

(c) accompanied by a fee of Rs. 200/- where such appeal is filed by the other party; (d) in the form of a memorandum of appeal precisely stating therein the substantial question of law

involved. Question 3 (a)Discuss briefly, whether excise duty is attracted on the excisable goods manufactured in the following cases: (i) In the State of Jammu and Kashmir; (ii) By or on behalf of the Government. (b)What is the period of provisional attachment of property during the pendency of any proceeding under section 11A or section 11D of the Central Excise Act, 1944? (c)Under what circumstances, the rebate of excise duty paid on exported goods can be granted to the exporter in case of export of goods to Nepal? Note-Rebate of excise duty paid on exported goods is granted to Government of Nepal. (d)If a manufacturer manufactures various products, can be avail CENVAT credit on some products and exemption under Notification No.8/2003 –CE dated 1.3.03 on some other products? (e)Briefly discuss the residual penalty under rule 27 of the Central Excise Rules, 2002. (2X5=10Marks) Answer (a) (i) Yes, excise duty is attracted on the excisable goods manufactured in the State of Jammu and Kashmir. Though originally the Central Excise Act, 1944 did not apply to Jammu and Kashmir, its application was extended to the same with the enactment of Taxation laws (Extension to Jammu and Kashmir) Act, 1954.

Page 108: Indirect Taxation CA Workbook

CA. Raj Kumar Practice Booklet (ii) Section 3(1A) of the Central Excise Act, 1944 provides that –

The excise duty shall be leived and collected on all excisable goods other than salt which are produced or manufactured in India by, or on behalf of, Government, as they apply in respect of goods which are not produced or manufactured by Government.

Thus, excise duty will be payable on goods manufactured by, or on behalf of, the Government (both Central & State) also.

(b) As per sec. 11 DDA- Provisional attachment of property can be done for a period of 6 months

This period can be extended with written permission of Chief Commissioner of Central Excise. However; total period of extension cannot be more than 2 years.

(c) Rebate to excise duty paid on goods exported to Nepal shall be granted to the Government of Nepal subject to the following conditions:

Rebate shall not, in each case , exceed the aggregate of the duty of Customs and additional duty of Customs leived by his Majesty’s Government of Nepal on such goods when they are imported into Nepal from any country other than India;

The excisable goods shall be exported after the payment of duty, directly from a factory or warehouse except as otherwise permitted by the Central Board of excise and Customs by a general or special order;

the excisable goods shall be exported within 6 months from the date on which they were cleared for export from the factory of manufacture or warehouse or within such extended period as the Commissioner of Central excise may in any particular case allow;

when the goods are exported by land, the export shall take place through any of the prescribed land customs

stations or such other check-post as may be specified by the Central Board of excise and Customs.

(d) No,

If a manufacturer manufactures various products, he has to avail CENVAT for all items or opt for exemption

for all products.

This view has been upheld in Ramesh Foods products (2004) (SC).where it has been held that simultaneous

availment of CENVAT credit on some products and exemption on some other products is not permissible.

(e) Rule 27 of the Central Excise Rules, 2002 stipulates that-

Where no other penalty is provided in the rules therein or in the Act, a breach of these rules shall be punishable

with

a penalty which may extend to Rs.5,000 and

with confiscation of the goods in respect of which the offence is committed.

Part-B Question 4

(a) Calculate the VAT liability for the period jan.1, 2007 to Jan 31,2007 from the following particulars:

Inputs worth Rs.1,00,000 were purchased within the state Rs.2,00,000 worth of finished goods were sold within the

State and Rs.1,00,000 worth of goods were sold in the course of inter-State trade.VAT paid on procurement of capital

goods worth Rs.1,00,000 during the month at 12.5% . If the input and output tax rate in the State are 12.5% and 4%

Page 109: Indirect Taxation CA Workbook

CA. Raj Kumar Practice Booklet respectively and the central sales tax rate is 2% show the total tax liability under the State VAT law and under the

Central Sales Tax Act. (5 marks)

(b)Calculate the net service tax payable under the provision of rule 2A of the Service Tax (Determination of value)

Rules, 2006 relating to determination of value of services in the execution of a works contract from the following

particulars:

(i)Gross amount for the works contract (excluding VAT) Rs.1, 00,000

(ii)Value of goods and materials sold in the execution of works contract Rs. 70,000

(iii)CENVAT credit on (ii) above Rs.1,000

(iv)Service tax paid on input services Rs.1,000

(v)CENVAT credit on capital goods issued in the provision Rs.1,000

of works contract service

(vi)Service tax rate 10.36%

Make suitable assumptions and provide explanations where required.

Answer (a)Computation of the tax liability for the period Jan .1, 2007 to Jan 31, 2007

Statement showing computation of Vat liability

Particulars Amount (Rs.)

Total Output Tax-during the relevant period

VAT payable on Local sales (2,00,000 * 4%) 8000

+ CST payable on Inter State Sales (1,00,000 * 2%) 2000

TOTAL (A)

10000

Net Input Tax Credit- during the relevant period

Opening Balance nil

ITC accruing during the relevant period (12500 +12500) 25000

(-) ITC reversed during the relevant period nil

(-) Refunds during the relevant period nil

TOTAL (B)

25000

Excess credit carried forward to subsequent period 15000

Note – it is assumed that credit on capital goods is available in the first year. (b) Computation of the Net service tax payable: - Rs.

Page 110: Indirect Taxation CA Workbook

CA. Raj Kumar Practice Booklet Gross amount charged (excluding VAT) 1, 00,000 Less: Value of goods and materials 70,000 30,000 Service [email protected]% 3,090 Less: CENVAT credit of service tax paid on input service CENVAT credit of excise duty paid on capital goods (50%) [Rs.1, 000+ (50% of Rs.1, 000)][Note 2] 1,500Net service tax payable 1,590 Notes- (1) Cenvat credit of Rs.1, 000 on value of goods and materials sold in the execution of works contract shall not be available to the assessee. (2) CENVAT credit in respect of capital goods shall be taken only for an amount not exceeding 50% of the duty paid on such capital goods in the same financial year and balance 50% shall be available in the subsequent financial year [Rule 4(2) of the CENVAT Credit Rules, 2004. Question 5 (a)XYZ Co. involved in the services of uploading of coal from wagon tipping system, stacking/reclaiming of coal to stacker reclaimed system and feeding of coal to boiler bunkers through conveyer system. The Department had taken a view that the charges received be XYZ Co. for such activity were taxable under the category of ‘Cargo handling services’ in terms of section 65(105)(Zr)read with section 65(23) on the Finance Act,1994.M/S . XYZ Co. Claimed that the services rendered by it cannot be brought under ‘Cargo handling service’ as it is engaged only in the handling of coal from railway wagons to the required designation of the thermal power station wherein machines are used with the aid of some manpower. Briefly explain, with the reference to relevant provisions and case law, if any, whether the stand taken is correct in law. (5 marks) (b)M/s Krishna Computer Color lab. Is in the business of developing and printing of color photographic films. It develops the negatives supplied by the customer and provides positive prints as per the order placed by the customer .The Department has demanded service tax on the entire amount charged from the customers without deduction of any amount towards cost of materials. The assessee’s contention is that no service tax could be charged on the material content since service tax is only a tax on services and not on goods. Therefore, the assessee has sought to Bifurcate the gross receipts on account of processing of photographs into the portion attributable to goods and those attributable to services so that service tax could be paid with respect to the value of service alone in their case. Briefly explain, with regard to decided case law, if any, whether the stand taken by M/s .Krishna Computer Color Lab .is correct in law. (5 marks) Answer (a) The facts in the case are similar to the matter decided by the Rajasthan High Court in S.B.Construction Company.

The High Court has held that the service tax has been leived on cargo handling, i.e. on such services which undertake the activities of packing, unpacking, loading, unloading of goods to be transported by any means of transportation namely truck, rail, ship or aircraft.

In the instant case, the services provided by the company under the contract is distinct i.e., transporting coal from wagons to thermal power station by conveyor belt and not by any means of transpiration.

Thus, the services rendered by the company cannot fail under the ambit of cargo handling services and as such is not liable to service tax.

(b)Yes, the request of M/s.Krishna Computer Color Lab is valid in law.

Page 111: Indirect Taxation CA Workbook

CA. Raj Kumar Practice Booklet In a similar case in Adlabs 2006, the Bangalore Tribunal held that –

The assessee was entitled to deduction of the cost of materials which were used during the course of providing the services of photography while computing the value of taxable services.

The question of bifurcating the receipts into an element of goods and the element of service could not and did not arise.

Hence, the stand taken by M/s Krishna color lab, is correct in law Question 6 (a) State whether the following are taxable under the provisions of the Finance Act, 1994 relating to service tax:- (i)Services provided in connection with the management of an organization by a management consultant having no professional qualification. (ii)Advance payment received from the service recipient by a person rendering construction services under section 65 (105)(zzzh)of the Act. (4 marks) (b) State briefly, whether the following persons are liable to apply for registration under the Finance Act, 1994 and the Service Tax (Registration of special Category of Persons) Rules, 2005 and if so, from which date: (i)An input service distributor who starts his business with effect from 1st January 2008. (ii)A provider of taxable service under an unregistered brand name of another person. Aggregate value of taxable service was Rs.6, 00,000 upto 31.3.08 (4 marks) (c)Write a brief note to explain the impact of VAT on lease transactions. (4 marks) (d) Illustrate with an example, whether inter-state purchases liable to central sales tax are eligible for input credit and explain the effect of the same on inter-state transactions. (4 marks) (e)Who are not eligible for composition scheme under the VAT regime? Discuss in brief. (4 marks) Answer (a) (i)Yes, as per section 65(65), no academic or professional qualification is required in order to be taxable under the category of ‘management or business consultant’service.In case, a person is not professionally qualified and is providing service of management provided by him. (ii)Section 67(3) of the Finance Act, 1994 stipulates that-

The gross amount charged for the taxable service shall include any amount received towards the taxable service before, during or after provision of such service.

Thus, the advance payment received from the service recipient by a person rendering construction services under section 65 (105) (zzzh) shall be liable to service tax.

(b) (i)Yes, as per rule 3(1) of the Service Tax (Registration of Special Category of Persons) Rules, 2005-

An input service distributor is liable to apply for registration within a period of 30 days of the commencement of business.

Thus, in the given case, the input service distributor should apply for registration by 31.10.2008. (ii)Yes,

a provider of taxable service under a registered brand name of another person is liable to apply for registration within a period of 30 days of commencement of business of providing taxable service

Page 112: Indirect Taxation CA Workbook

CA. Raj Kumar Practice Booklet as basic exemption for small service provider is not available to a service provider who provides taxable

service under a trade name or brand name, whether registered or not, of another person. (c)Impact of VAT on lease transactions:- 1. Lease is chargeable to tax by virtue of Sub-clause (d) of clause 29A of Article 366 of the constitution of India. This is a tax on the transfer of right to use any goods for any purpose (Whether or not for a specified period) for cash, deferred payment or other consideration. 2. The taxable event is the transfer of right to use any goods and hence immovable property is not covered. 3. The taxable turnover means the valuable consideration paid or payable for any sale in a given period. Certain states have provided for deduction of interest or finance charges for the purpose of determination of taxable turnover. 4. Lease of an asset in the course of inter-state trade cannot be subject to VAT. 5. Transfer of right to use does not presuppose ownership of the goods. A sub-lease as an asset also is taxable unless specifically exempt under the State VAT Law. 6. Sale of leased asset after the end of the lease period is taxable as a normal sale. 7. The maintenance of leased asset involving supply of materials for maintenance/repair by the lessor will not amount to a works contract as there would be no transfer of property in such materials to the lessee. Hence, there would be no VAT on the value of materials supplied during maintenance/repair of the leased asset. 8. The lessor would pay VAT at the time of procurement of goods. However, liability to pat VAT on lease rentals will be spread over the tenure of the lease. Therefore, some states have provided for utilization of input credit for paying output tax only over the entire period of lease. (d)

The Inter-State purchases liable to central sales tax are not eligible for input tax credit. However; the liability to central sales tax can be set off against the input tax credit earned on other eligible

purchases. By way of an example,

A dealer in State K purchases goods from another dealer in state M.The dealer in state M charges CST at 2% on this sale against C form produced by dealer in State K.The tax deposit in state M.

Though it is called CST, the Central Government does not get any share in this and it is a revenue receipt of the selling state.

The dealer in state K sells these goods within the state K and collects VAT which is deposit in state K The question is whether the dealer in state K can claim input tax credit of the central sales tax paid by him

to be set off against the VAT liability in state K . This is not possible as state K will not permit the set off against the tax paid to another state M.This is

because the same if permitted would result in loss of revenue to state K . This is the reason that the credit of CST is not available.

The effect on inter-state trade is that trade will be uncompetitive for the states that are net importers because in those state consumer prices will be high, CST is origin based tax collected by the exporting State while VAT is a destination based consumption tax. They both cannot go together and to that extent the objective behind the VAT regime in lowering of prices for consumers cannot be achieved.

Page 113: Indirect Taxation CA Workbook

CA. Raj Kumar Practice Booklet (e)

Small dealers having gross turnover exceeding Rs 5 lakhs but less than Rs 50 lakhs have option of composition scheme.

They will have to pay a small percentage of gross turnover. They will not be entitled to any input tax credit.

However, the following are not eligible for the composition scheme:

(i) A manufacturer or a dealer who sells goods in the course of inter-state trade or commerce; or (ii) A dealer who sells goods in the course of import into or export out of territory of India; or (iii) A dealer transferring goods outside the state otherwise than by way of sale or for execution of works contract; or (iv) A manufacturer/dealer who makes inter-state purchases; or (v) A dealer /manufacturer who issues vatable invoices.

PART-C

Question 7 A consignment of 800 metric tones of edible oil of Malaysian origin was imported by a charitable organization in India for free distribution to below poverty line citizens in a backward area under the scheme designed by the Food and Agricultural organization. This being a special transaction, a nominal price of US$ 10 per metric tonne was charged for the consignment to cover the freight and insurance charges. The customs House found out that at or about the time of importation of this gift consignment there were following of edible oil of Malaysian origin: S.No Quantity imported in metric tones Unit Price in US $ (CIF) 1. 20 260 2 100 220 3 500 200 4 900 175 5 400 180 6 780 160 The rate of exchange on the relevant date was 1US $ =Rs.43.00 and the rate of basic Customs duty was 15% advalorem. There is no countervailing duty or special additional duty. Calculate the amount of duty leviable on the consignment under the Customs Act, 1962 with appropriate assumptions and explanations where required. (5 marks) Answer Determination of transaction value of the subject goods:- In the instant case, while determining the transaction value of the goods, following factors need consideration;-

1. In the given case ,US $ 10 PER METRIC TONNE HAS BEEN PAID ONLY TOWARDS FREIGHT AND INSURANCE CHARGES AND NO AMOUNT HAS BEEN PAID OR PAYABLE TOWARDS THE COST OF GOODS .Thus, there is no transaction value of the subject goods. Consequently, we have to look for transaction value of identical goods under rule 4 of Custom Valuation (Determination of Value of Imported Goods) Rules, 2007 [Customs Valuation (DVIG)Rules, 2007].

2. Rules 4(1)(a)of the aforementioned rules provides that subject to the provisions of rule 3 , the value of imported goods shall be transaction value of identical goods sold for export to India and imported at or about the same time as the goods being valued .In the six imports during the relevant time , the goods are identical in description and of the same country of origin. It may be presumed that they were produced by the same person.Even otherwise, such consignments can be accepted as identical goods.

Page 114: Indirect Taxation CA Workbook

CA. Raj Kumar Practice Booklet 3. Further, clause (b) of Rule 4(1) of the said rules requires that the comparable import should be at the same

commercial level and in substantially same quantity as the goods being valued.Since, nothing is known about the level of the transactions of the comparable consignments, it is assumed to be at the same commercial level.

4. As far as the quantities are concerned, the consignments of 20 and 100 metric tones cannot be considered to be substantially the same quantity. Hence; remaining 4 consignments are left for our consideration.

5. However, the unit prices in these 4 consignments are different Rule 4(3) of customs Valuation (DVIG) Rules, 2007 stipulates that in applying rule 4 of the said rules, if more than one transactions value of identical goods is found, the lowest of such value shall be used to determine the value of imported goods. accordingly, the unit price of the consignment under valuation shall be US $ 160 per metric tonne.

Computation of amount of duty payable:- CIF value of 800 metric tones: =800 X 160 = US $ 1,28,000 At the exchange rate of $ 1 =Rs.43 CIF Value (in Rupees) (US $ 1,28,000 * 43) = Rs.55, 04,000 Add: landing Charges at 1% = Rs.55,040 = Rs.55, 59,040 15% of Ad Valorem duty On Rs.55, 59,040 = Rs.8, 33,856 Add; Education cess @ 2% (Rounded off) = Rs.16, 677

Add: Secondary and Higher education Cess@1% (rounded off) = Rs.8, 339 Total custom duty payable = Rs.8, 58,872 Question 8 M/s IES Ltd. (assessee) imported certain goods at US $ 20 per unit from an exporter who was holding 30% equity in the share capital of the importer company. Subsequently, the assessee entered into an agreement with the same exporter to import the said goods in bulk at US $ 14 per unit. When the imports at the reduced price were effected pursuant to this agreement, the Department rejected the transaction value stating that the price was influenced by the relationship and completed the assessment on the basis of transaction value of the earlier imports i.e.at US $ 20 per unit under rule 4 of the Customs Valuation (Determination of Value of Imported Goods)Rules,2007,viz transaction value of identical goods .State briefly ,whether the Department’s action is sustainable in law, with reference to decided cases, if any (5 marks) Answer No, the Department’s action is not sustainable in law,

Rule 2(2) of Customs Valuation (Determination of Value of Imported Goods) Rules, 2007 interalia provides that persons shall be deemed to be “related” if one of them directly or indirectly controls the other.

The word “control” has no where been defined under the said rules. As per the common parlance, the control is established when one enterprise holds at least 51% of the equity shareholding of the other company.

However, in the instant case, the exporter company held only 30% of shareholding of the assessee. Thus, Exporter Company did not exercise a control over the assessee. So, the two parties cannot be said to be

related. The fact that assessee had made bulk imports could be a reason for reduction of import price.The burden to prove under valuation lies on the Revenue and in absence by an evidence from the Department to prove under valuation, the price declared by the assessee is acceptable. Conclusion- In the light of foregoing discussion, it could be inferred that Department’s action is not sustainable in law.

Page 115: Indirect Taxation CA Workbook

CA. Raj Kumar Practice Booklet Question 9 (a) Explain briefly, the significance of Indian custom water. (b) Explain briefly, the Customs Act, 1962, with effect from 10.10.2007, and the customs Valuation (Determination of Value of Imported Goods) Rules, 2007 are now fully compatible. Explain with a brief Note (c)Can warehoused goods be transferred from one warehouse to another under the Customs Act, 1962? (d)What is the minimum rate or amount of duty drawback prescribed under the Customs, Central Excise Duties and Service Tax Drawback Rules,1995 made under section 75 of the Customs Act,1962?Explain with a brief note. (e)Briefly discuss the procedure for confiscation of goods or imposition of penalty under section 124 of the Customs Act, 1962. (2X5=10Marks) Answer (a)Significance of Indian customs waters is as follows:

(i)if an officer of customs has reason to believe that any person in India or within the Indian Customs waters has committed an offence punishable under section 132 or section 133 or section 135 A or section 136,he may arrest such person informing him of the grounds for such arrest [Section 104 of the Customs Act,1962] (ii)Where the proper officer has reason to believe that any vessel in India or within the Indian customs waters has been, is being, or is about to be, used in the smuggling of any goods or in the carriage of any smuggled goods, he may stop any such vehicle, animal or vessel or, in case of an aircraft, compel it to land [Section 106 of the Customs Act, 1962] (iii)Any vessel which is or has been within the Indian customs waters in constructed adapted, altered or fitted in any manner for the purpose of concealing goods shall be liable to confiscation[section 115(1)(a)of the Customs Act,1962] (iv)Any goods which are brought within the Indian Customs waters for the purpose of being imported from a place outside India, contrary to any prohibition imposed by or under this Act or any other law for the time being in force, shall be liable to confiscation [Section 111(d) of the Customs act, 1962] (v) Customs officer has the power to search any person who has landed from/about to board/is on board any vessel within Indian customs waters and who has secreted about his person, any goods liable to confiscation or any documents relating thereto [Section100 of the Customs Act,1962]

(b) As per erstwhile section 14 of the Customs act,1962,the ‘Customs value’ was ‘deemed value’.However,the Customs Valuation(determination of Price of Imported Goods)Rules,1988 provided that the value of the imported goods should be based on the concept of ‘transaction value’, where TV can not be taken A.V. (c) Yes, section 67 of the Customs Act, 1962 permits removal of warehoused goods to other warehouse under bond, with the permission of the proper officer, subject to such conditions as may be prescribed. (d)

Minimum rate of duty drawback-Rules 8 (1) of Customs, Central excise Duties and Service Tax Drawback Rules, 1995 provides that no amount or rate of drawback shall be determined in respect of any goods, the

Page 116: Indirect Taxation CA Workbook

CA. Raj Kumar Practice Booklet amount or rate of drawback shall be less than 1 % of the F.O.B.value thereof, except where the amount of drawback per shipment exceeds Rs.500

Maximum rate of duty drawback- Rule 8A of Customs, Central Excise Duties and Service Tax Drawback Rules, 1995 provides that the drawback amount or rate shall not exceed 1/3 of the market price of the export product.

(e) Section 124 of the Customs act, 1962 provides that-

Before confiscating goods or imposing any penalty on any person, a show cause notice must be issued to the owner of goods giving grounds for confiscation or imposition of penalty and

He should be given an opportunity to make representation and being heard. The show cause notice can be issued only with the prior approval of the officer of customs not below the rank

of Deputy Commissioner of Customs. The notice and the representation at the request of the person concerned can be oral.

JUNE- 2009- NEW SYLLABUS

SECTION-A

Question 1 (a)Determine the total amount of excise duty payable on a machine using the details given below:

Page 117: Indirect Taxation CA Workbook

CA. Raj Kumar Practice Booklet (i)Sale price of the machine excluding taxes and duties 2, 00,000 (ii)Sales tax 20,000 (iii)Cost of durable and returnable packing included in the sale price given at (i) above 5,000 (iv)Design and development charges paid by buyer on behalf of seller to a third party 20,000 (v)Warranty charges charged separately by the seller 5,000 Rate of excise duty 16% Education Cess 3% Calculations should be supported by notes, wherever required. (5 marks) (b) X, a manufacturer purchased 500 kgs of inputs on 1.10.2007.Total assessable value of inputs was Rs.10, 000 and excise duty of 16% and 2% of excise duty as education cess was paid on the input. On the day of receipt itself, inputs were sent to the job worker.Job worker sent back 50% of the inputs on 1.4.2008 and balance 50% on 31.5.2008. X received back the processed inputs on the same day.Calculate the CENVAT credit required to be reversed or that can be availed on relevant dates and net availment and reversal in the financial year 2007-08 and 2008-09 (5 marks) (c) Y & Co. is a small scale unit in a rural area and is availing the benefit of small scale exemption under Notification No. 8/2003 –C.E.during the year 2007-08 .Determine the value of the first clearances of the unit and duty liability on the basis of data given below:- Rs.

1. Total value of clearances of goods with own brand name 75,00,000 2. Total value of clearances of goods with brand name of other parties 90,00,000 3. Clearances of goods which are totally exempt under another notification 35,00,000

(other than an exemption based on quantity or value of clearances) Normal rate of Excise duty – 16% Education cess @3% of Excise Duty Calculations should be supported with appropriate notes. It may be assumed that the unit is eligible for exemption under Notification No.8/2003 (5 marks)

Answer (a) Computation of total amount of excise duty payable:-

Particulars Rs. Sale price of the machine excluding taxes and duties 2, 00,000 Add: Design and development charges (Note-3) 20,000 Add: Warranty charges (Note-4) 5,000 Less: Cost of durable and returnable packing (Note-2) 5,000Assessable value 2, 20,000Excise duty @16% 35,200 Education cess @3% 1,056Total excise duty payable (Rs 35,200+Rs.1, 056) 35,256 Notes:

1. Sales tax is not included since the definition of transaction value as per section 4 specifically excludes sales tax paid or payable on the goods.

2. Cost of durable and returnable packing shall not form a part of transaction value.(amortized cost is not given in the question ,therefore it is assumed that amortized cost already included in the value of machine)

Page 118: Indirect Taxation CA Workbook

CA. Raj Kumar Practice Booklet 3. Design and development charges are essential for the purpose of manufacture and to make the product

marketable. Hence, they have to be included in the assessable value, since payment is ‘in connection with sale’.

4. If the warranty charges are charged separately and are not considered as “Price “of the goods by the assessee, then warranty charges will be includible in the transaction value forming basis of the valuation [Circular No.354/81/2000-] er rule 4(5) of CENVAT Credit Rules, 2004, a manufacturer or output service provider is entitled to claim

CENVAT credit on the inputs that are cleared to a job worker for further processing,testing,repair, re-conditioning, or for the manufacture of intermediate goods necessary for the manufacture of final products or any other purpose provided the goods are returned back in the factory within 180 days of their being sent to the job worker. In case inputs are not returned within 180 days, the manufacturer or provider of output service shall pay an amount equivalent to the CENVAT credit attributable to the inputs or capital goods by debiting the CENVAT credit or otherwise, but the manufacturer or provider of output service can take the CENVAT credit again when the provider or capital goods are received back in the factory or in the premises of the provider of output service. Rs.

it that can be taken on 1.10.2007 (Rs,10,000X16.48%) 1,648 CENVAT credit that should be reversed on 30.3.2008(on completion of 180 days) (1,648)

CENVAT credit that can be taken on 1.4.2008 (50% of Rs. 1,648)-(Re-availment) 824 CENVAT credit that can be taken on 31.5.2008 (50% of Rs. 1,648) (Re-availment) 824 CENVAT credit availed in financial year 2008-09 1,648

Computation of the value of first clearances and the duty liability:- Particulars Rs. 1. Value of clearances of goods with own brand name 75,00,000 2. Value of clearance of goods with brands name of other parties (Note-1) 90,00,000

(b) As p

CENVAT cred

(c)

Total value of first clearances 1, 65, 00,000Value on which duty is chargeable (Rs.1, 65, 00,000-1, 50, 00,000)= Excise duty payable @ 16% (Rs.15, 00,000 X 16%) 2,40,000

Education cess payable @ 3% (Rs.2, 40,000X 3%) 7,200

Rs.15,00,000

Total excise duty payable 2,47,200

Notes: 1. SSI units in rural area are eligible to clear goods with other’s brand name availing the exemption as

Per Notification No.8/2003. 2. Notification No.8/2003 also provides that value of clearances of goods totally exempt under other notifications

need not be taken into account for calculation of aggregate value of first clearances.

Question 2 (a) M/s ABC Ltd was a cement manufacturer. The company used ropeway system for bringing crushed limestone

from the mines located 4-5 kms away form the factory. A part of ropeway system was installed in the factory and the system was controlled from the factory. M/s.ABC Ltd .availed CENVAT credit on parts /spares for ropeway system treating the same as capital goods .The Central Excise Department denied CENVAT credit on the ground that ropeway is used for transporting raw materials form the mines to the factory and can not be considered as material handling system within the factory premises. Examine, with the help of a decided case law, whether the stand taken by the Department is correct in law. (5 marks)

Page 119: Indirect Taxation CA Workbook

CA. Raj Kumar Practice Booklet

(b) A port trust used cement concerete armour units in the harbor for keeping water calm. Each unit weighed about 50 tons and is like a tripod and keeps water calm and tranquil. These units are essentially in prismoid form and were made to order. They are harbor or location specific. The central Excise Department contended that the armour units are excisable goods and chargeable to duty. Examine the validity of the Department’s contention in the light of decided case law. (5 marks)

(c) I Scream Ltd. is engaged in manufacture of ice-cream falling under sub-heading 2105 of the Central Excise Tariff Act, 1985.The Company supplied the ice-cream in 4 litres pack to catering industry or hotels, who sell the same in scoops. The pack contained a declaration that the pack was not meant for retail sale. The Department contended that packs have to be assessed on the basis of value arrived at as per the provision of section 4A of Central Excise Act,1944,which provides for assessment based on M.R.P. and not under section 4 of the Central Excise Act,1944.Assessee contended that the ice-cream pack sold was a bulk pack of 4 litres which was not meant to be sold in retail and they were not required to print the Maximum Retail Price and hence the transaction was a whole sale transaction and assessment under section 4 of the Central Excise Act,1944 was correct. Further contention of the assessee was that they are entitled to exemption under rule 34 of the standard of Weights and Measures (PC) Rules, which exempts packs meant for industrial use. Examine the stand taken by the Department is correct in the light of decided case law. (5 marks)

Answer (a) No, Department’s stand is not correct in law.

Rule 2(a) of CENVAT Credit Rules, 2004, inter alia, provides that capital goods means goods “Used in the factory of the manufacturer of final products”.

The Department’s contention is that ropeway is used for transporting raw material form mines located 4-5 kms away and hence it can not be said to have been used in the factory.

The facts of the given case are similar to the case of M/s.Birla Corporation Ltd. 2005 (SC)- In the instant case, the Apex Court followed the principal laid down in case of J.K.Udaipur Ltd. wherein the

same question arose for consideration and the facts were almost identical. Therefore, the assessee was entitled to the CENVAT credit of the spares of the ropeway system because the

ropeway system, used to bring the crushed lime stone form the mines to the factory, could be covered under the expression ‘Precints of the premises’in the definition of the factory under section 2(e)of the Central Excise Act,1944.

Thus, ropeway system is used within the factory. Therefore, parts/spares for ropeway system are covered under the definition of capital goods .

Hence CENVAT credit is admissible. cts of the given case are similar to the case of Board of Trustees 2007 (SC).

The Supreme Court held that in order to constitute “goods”,twin tests have to be satisfied,namely,process constituting manufacture and secondly marketability. In the present case, the second test of marketability was in issue.

In this case, armour blocks, in prismoid form, were made to order and were of certain specifications. They were harbor or location specific. It would depend on the water level required to be maintained in the harbor.

There was no evidence to show that these blocks could not be used in any other harbor.Moreover,the Department failed to prove marketability of the impugned goods.Therefore,assessee’s contention , that goods are not capable or being bought and sold in the market ,has to be accepted .

Therefore, Department’s stand is not correct and no duty is payable on the goods.

(b) The fa

Page 120: Indirect Taxation CA Workbook

CA. Raj Kumar Practice Booklet ue is covered by a decision of the Supreme Court in the case of Jayanti Food Processing (P)Ltd(c)The iss 2007.

It was held that- The product was not covered by the provision of section 4A of the Central Excise Act, 1944. The Supreme Court held that firstly, the assessee could not being sold directly to the consumer, but to the

intermediary i.e.hotels. Secondly, 4 liter pack of ice-cream could have been covered under the definition of “retail package “only if

such package had been intended for retail sale of consumption by an individuals.However,the package specifically displayed that “the pack was not meant for retail sale “.

Moreover, hotel could not be covered in term of ‘individual or a group of individuals’. Hence, the ice-cream package could not be termed as “retail package”.

Therefore, the assessee was not required under Standards of weight and Measures Act and the rules there under to print the MRP on the package.

he Apex Court agreed with the contention of the assessee that they were entitled to exemption under rule 34 which exempts the product form requirement of affixing maximum retail price on the pack.

xempts the package specially packed for the purpose of servicing any industry.

Supreme Court concluded that the sale of pack of ice-cream to the retail industry was squarely covered in the term “servicing and industry”.

Hence, it could be inferred that, since, the requirement of affixing maximum retail price is a pre-condition for application of section 4A of the Central Excise Act, 1944, assessment under section 4 is correct.

Lastly, t

Rule34 of the Standard of Weights and Measures (Packaging Commodity) Rules, 1977 e

Question 3 (A)Explain special procedure and facilities made available to large tax payer under rule 12BB of Central Excise Rules, 2002 in respect of intermediate goods. (6 marks) (b)What are the provisions relating to the payment of excise duty on used capital goods cleared by an assessee on which CENVAT credit has been availed? (2 marks) (c)What are the situations in which duty can be remitted under rule 21 of Central Excise Rules, 2002 (2 marks) Answer (1) A large taxpayer may remove excisable goods (intermediate goods), without payment of excise duty,

Except motor spirit, commonly known as petrol, high speed diesel and diesel oil, light

under the cover of a transfer challan from any of his registered premises (sender Premises) to his other registered premises (recipient premises) for further use in the manufacture of other excisable goods

(subject goods). subject to the condition that the subject goods are manufactured using the said intermediate goods and cleared

on payment of duties leviable thereon or exported within 6 months, from the date of receipt of the intermediate goods in the recipient premises; or

Explanation 1:-

The transfer challan shall be serially numbered and shall contain, the registration number, name, address of the large taxpayer, description, classification, time and date of

removal, mode of transport and vehicle number, quantity and registration number and name of consignee. Provided that,

If the subject goods manufactured or produced using the intermediate goods, are not cleared on payment of duties or are not exported within 6 month,

Page 121: Indirect Taxation CA Workbook

CA. Raj Kumar Practice Booklet duties payable on such intermediate goods shall be paid by the recipient premises with interest specified u/s

11AB. Explanation 2:- The rate of duty shall be of the date of removal of intermediate goods from sender’s premises.

Explanation 3:- If the large taxpayer fails to pay such amount, it shall be recovered along with interest in the manner provided u/s 11A and 11AB.

(3) A large taxpayer may, with intimation of at least 30 days in advance, opt out to be a large taxpayer from the first day of the following financial year.

Proviso to rule3 (5) of CCR,2004, PROVIDES THE METHOD TO BE ADOPTED FOR PAYMENT OF AN AMOUNT ON USED CAPITAL GOODS-

If the capital goods, on which CENVAT Credit has been taken, are removed after being used, the manufacturer or provider of output services shall

pay an amount equal to the CENVAT Credit taken on the said capital goods reduced by the percentage points calculated by straight line method as specified

below for each quarter of a year or part thereof from the date of taking the CENVAT Credit, namely:-

(a) For computers and computer peripherals:

for each quarter in the first year @ 10%

(b)

for each quarter in the second year @ 8% for each quarter in the third year @ 5% for each quarter in the fourth and fifth year @1%

(b) For capital goods, other than computers and computer peripherals @ 2.5% for each quarter. le 21 of the Central Excise Rules, 2002 provides that where it is shown to the satisfaction of Commissioner

empowered to remit the duty, that:- (i) Goods have been lost or destroyed by natural causes or, (ii) By unavoidable accident or (iii) The goods have become unfit for consumption or for marketing.

At any time before removal, he may remit the duty payable on such goods.

Section -B

(c) Ru

Question 4 (a)Purchases by S&Co. for the month of December are as follows: (1)Rs.1, 00,000 at 4% VAT. (2)Rs.5, 00,000 at 12.5% VAT Sales of S&Co. for the month of December are as follows: (1)Sales of Rs.3, 00,000 at 4% VAT (2)Sales of Rs.3, 00,000 at 12.5%VAT Compute eligible input tax credit and VAT payable for the month. (3 marks)

Page 122: Indirect Taxation CA Workbook

CA. Raj Kumar Practice Booklet (b) Calculate the value of taxable service of ‘X’ Transport Company engaged in the business of transport of goods by road. Give reasons for taxability or exemption of each item. No freight is received form any of the specified category of consignor/consignee. Suitable assumptions may be made wherever required .X does not avail CENVAT credit:

Rs. 1. Total freight charges received by ‘X’during the year 13,50,000 2. Freight charges received for transporting fruits 1,20,000 3. Freight collected for transporting small consignment for persons

who paid less than Rs.750 for each consignment 75,000 4. Freight collected for transporting goods in small vehicles for persons

who paid less than Rs.1,500 per trip 1,50,000 (7 marks)

Answer (a)Computation of input tax credit and VAT payable:- Rs. Input tax paid by S& Co Rs.1,00,000@4% 4,000 Input tax paid by S&Co Rs 5,00,[email protected]% 62,500 Total input tax credit eligible 66,500 Output tax payable on Rs.3,00,000 @4% 12,000 Output tax payable on Rs.3, 00,000 @12.5% 37,500 Total tax payable 49,500 Net input tax allowed to be carried over (Rs66, 500-Rs.49, 500) 17,000

putation of value of taxable service: Rs. Total freight collected 13,50,000 Less: freight charges received for transporting fruits (Note-1) 1, 25,000 Less: freight collected less than Rs.750/- for each consignment (Note-2) 75,000 Less: Freight collected for transporting goods in small vehicles for persons Who paid less than Rs.1,500/- Per trip (Note-2) 1,50,000

VAT payable by S &Co. Nil (b)Com

Balance freight charges 10,00,000 Less: abatement of 75% (Note-3) 7,50,000 Value of taxable service 2, 50,000 Notes:

1. Taxable service provided by a goods transport agency in relation to transport of vegetables, eggs or milk by road is exempt form service tax (Notification No.33/2004).

2. Service tax is exempt where: (i) The gross amount charged on all consignments transported in a goods carriage does not exceed

Rs.1,500/- or (ii) The gross amount charged on an individual consignment transported in a goods carriage does not

exceed Rd.750/- (Notification No. 34/2004)

(iii) As per Notification No.13/2008, service tax is required to be paid on 25% of the gross amount charged i.e. 75% of the gross amount charged is allowed as abatement.

Page 123: Indirect Taxation CA Workbook

CA. Raj Kumar Practice Booklet Question 5

(a) The assessee owned a collection centre, which was engaged in drawing of human blood, urine and stool samples on behalf of the principal laboratory in a metro city for conducting pathological tests. The assessee sent the samples through courier and received commission for such service. Department demanded service tax on the ground that the activity amounted to promotion or marketing of the service provided by the principal and hence covered under business auxiliary service. With reference to legal provisions and case law, examine the stand taken by the Department. (5 marks)

(b) The assessee used to finance the purchase of vehicles manufactured by a leading automobile company which used to sell the vehicles through its distributors. Assessee financed part of the vehicle cost of the purchaser of vehicle after getting, an agreement with the purchaser providing the right of repossession of the vehicle in case buyer of the vehicle defaulted in paying installment. Immediately on sale, purchaser of the vehicle became the owner and vehicle was registered in his name. The Department seeks to levy service tax in the category of “Banking and other Financial Services”. Examine whether Department’s stand is correct in terms of legal provisions’ and case law. (5 marks)

Answer (a)

Section 65(106) of the Finance Act, 1994 defines technical testing and analysis services. However, any testing or analysis done for the purpose of determination of the nature of diseased condition,

identification of a disease, prevention of any disease or disorder in human beings or animals has been excluded form the scope of this service.

In this case the Department’s contention is that assessee is providing business auxiliary service to the laboratory and is not eligible for the exemption since assessee is not performing laboratory functions.

Similar issue came up before Punjab &Haryana High Court in the case of Dr.Lal Path Lab (p)Ltd 2007 (P&H).High Court held that-

Merely because the assessee renders any incidental service like putting across or dropping of the name of the principal laboratory, it does not get covered by the definition of business auxiliary service.

The activity of drawing of test sample is covered by the exception to technical testing and analysis service under section 65(106)of the finance Act,1994

ue was examined by the customs Excise, Service Tax Appellate Tribunal (CESTAT) Mumbai in the case of Bajaj Auto Finance ltd(b)The iss

. 2007- The Tribunal distinguished between hire purchase agreement and hire purchase finance agreement. In the case

of former, the title of the goods remains with the hire purchase company. Payments are considered as hire charges and when all periodical payments are made, title to the goods is

transferred to the customer provided he exercises such an option. In case of latter, title to the goods vests with the purchaser right form the beginning and Hire Purchase

Company has only a right to take over possession for non-payment of loan. It was also quoted that-

Since the appellants herein have the right and control over the goods, it cannot be concluded that they had entered into a hire purchase agreement with their customers for the reason that in all hire purchase finance agreements, terms and conditions giving the finance company control over the goods, exist.

Since hire purchase finance is not covered by the definition of banking and other financial services under section 65(10) of the finance Act, 1994, service tax is not attracted.

This judgement has been affirmed later on by the Apex Court in. Bajaj Auto Finance Ltd.2008 (10) (SC).

Question 6 (a) What are the purchases not eligible for input tax credit of VAT paid? (6 marks)

Page 124: Indirect Taxation CA Workbook

CA. Raj Kumar Practice Booklet (b) What are the provisions relating to best judgement assessment under Service Tax Law?

( 2 marks) (c) State whether the following services are taxable under the provisions of the Finance Act,1994 relating to

service tax: (i)A practising Chartered Accountant representing a client before Income Tax Officer in assessment proceeding. (2 marks) (ii)Royalty received form the publisher by an author of a text-book for printing and publishing his book (2 marks)

(d)Give 4 illustrations to explain the scope of service rendered by a consulting engineer. (4 marks) (e)List out four services brought under the service tax net by the Finance Act, 2008 (4 marks)

Answer-(a) Input tax credit is not allowed on the following purchases.

(b) Section 72

(i) Purchases form unregistered dealers; (ii)Purchases form registered dealer who opt for composition scheme under the provisions of the Act;

(vi) Purchase of goods used for personal use/consumption or provided free of charge as gifts (vii)Goods imported from outside the territory of India (commonly known as high seas purchases); (viii) Goods purchased from other States viz. inter –state purchases.

(iii) Purchase of goods as may be notified by the State Government. (iv) Purchase of goods where the purchase invoice is not available with the claimant or there is evidence that the same has not been issued by the selling registered dealer from whom the goods are purported to have been purchased; (v) Purchase of goods, which are being utilized in the manufacture of, exempted goods

to

(c)

empower the Central Excise Officer to make best judgement assessment. The provisions of section are as follows: If any person, liable to pay service tax,- (a) Fails to furnish the return under section 70; (b) having made are turn ,fails to assess the tax in accordance with the provisions of this chapter or rules made thereunder ,the Central Excise Officer , may require the person to produce such accounts, documents or other evidence as he may deem necessary and after taking into account all the relevant material which is available or which he has gathered , shall be an order in writing , after giving the person an opportunity of being heared,make the assessment of the value of taxable service to the best of his judgement and determine the sum payable by the assessee or refundable to the assessee on the basis of such assessment.

(i) Services provided or to be provided by a practicing chartered accountant in his professional capacity, to a client relating to-

representing the client before any statutory authority in the course of proceedings initiated under any law for the time being in force,

by way of issue of notice, form the whole of service tax leviable thereon under section 66 of the finance act, 1994 as amended (Notification No.25/2006)

(ii) As per section 65(55a) of the Finance Act, 1994,”intellectual property right “means- any right to intangible property, namely, trade marks, designs, patents or any other similar intangible property,

under any law for the time being in force, but does not include copyright . Therefore, any royalty or other payment related to publishing involving copyright is not liable to tax.

Page 125: Indirect Taxation CA Workbook

CA. Raj Kumar Practice Booklet (d)Consulting engineer service would include- (i)Design engineering (ii) Pre design services /project report (iii) Supervision or construction and project management (iv) Feasibility (v)Supervision of commissioning and initial operation (vi) post operation and management (vii)Trouble shooting and technical services including establishing system and procedures for an existing plant. Note: Any 4 points may be given. (e)Follow

Note: Any 4 services may be given. Question 7

ing new services have been brought under tax net by the Finance Act, 2008:- 1. Services in relation to information technology software for use in the course, or in furtherance of business or commerce. 2. Services provided in relation to management of investment under unit linked insurance business, commonly known as ULIP.

3. Services provided by a recognized stock exchange in relation to securities 4. Services provided by a recognized or registered association (commodity exchange) in relation to sale or purchase of any goods or forward contracts. 5.Services provided by a processing and clearing house in relation to processing, clearing and settlement of transactions in securities , goods or forward contracts. 6. Services provided in relation to supply of tangible goods without transferring right of possession and effective control of the tangible goods.

7. Services provided in relation to internet telecommunication.

Form the following particulars; determine the assessable value of the imported equipment giving explanation for each item:

Rs. 1. FOB cost of equipment (Japanese yen) 2,00,000Yen 2. Freight charges in Japanese yen 20,000Yen 3. Charges for development connected to equipment paid in India Rs.60,000 4. Insurance charge paid in India for transportation form Japan Rs.15,000 5. Commission payable to agent in India Rs.15,000

Exchange rate as per RBI is 1 Yen =Rs.0.45 Exchange rate as per CBEC is 1 Yen =Rs.0.50 Landing charges: one percent of CIF cost (5 marks)

Answer Computation of assessable value:- FOB value 2, 00,000 Yen Add: Freight form japan (Note-1) 20,000 Yen Total (in yen) 2,20,000 Yen Exchange rate applicable is 1 Yen =Rs.0.05 (Note-4) Total (in rupees) Rs.1,10,000 Add:Insurance charges Rs.15,000 Total Rs.1,25,000 Add: Commission (Note-2) Rs.15,000 CIF value Rs. 1, 40,000 Add: Landing charges @1% of CIF value Rs.1, 400

Page 126: Indirect Taxation CA Workbook

CA. Raj Kumar Practice Booklet Assessable value Rs.1, 41,400

Notes: 1. Freight form Japan is includible in the assessable value 2. Agency commission paid in India is not a buying commission and hence will be added to CIF value. 3. Charges for development paid in India do not form part of assessable value because as per rule 10(1)(b)(iv)of the

customs valuation (Determination of value of imported goods )Rules ,2007, only the development undertaken at a place other than India shall form a part of assessable value.

4. Currency conversion rate as notified by CBEC is to be taken into consideration. Question 8 ABC imported a vessel ‘waterloo’ for the purpose of breaking form XYZ Ltd of U.K.A memorandum of understanding was signed between the buyer and seller on 2.6.97 and ABC took delivery of the vessel on 4.6.97.Vessel drifted and landed in the yarn of B in a damaged condition on 9.6.97. On 24.6.97, ABC filed application to concerned Assistant Commissioner for extension of time to file bill of entry, which was granted on 12.8.97. ABC paid Rs.24 crores to XYZ Ltd.towards the purchase price of the vessel .Thereafter ,ABC sold the vessel to B for Rs.12Crores and B filed bill of entry on 12.9.97.Assessing authority assessed the ship taking the value as Rs.24 crores and ship was taken over by B after assessment order was passed .’B’argues that assessable value should be taken as Rs.12 crores since the vessel was damaged because of the storm which made the vessel drift during appellate proceedings. No application for abatement of duty was made before the assessing authority by ABC OR B .Examine whether benefit of relief under section 22 of the Customs Act,1962 to reduce the value and thereby duty can be extended to B under the above circumstances. The assessment order in respect of bill of entry was passed on 23.12.97. (5 marks) Answer

section 22 for abatement

Supreme Court held that- In order to claim, the benefit of the abatement under section 22, the party claiming the abatement has to satisfy

the Assessing Authority that a case has been made out under of duty

Question 9

on damaged or deteriorated goods.

In the absence of any claim made under section 22 in writing to the assessing Authority, the appellant (B) could not claim the abatement under section 22 and the Assessing Authority did not record rightly to its satisfaction that the appellant was entitled to the abatement of duty.

In this case, damage occurred in Indian shore. Importer sought extension of time to file bill of entry. But, thereafter remitted the purchase price and sold the vessel B.

B in turn filed the bill of entry and the vessel was assessed. No application was made by the buyers i.e. importer (ABC) to the assistant Commissioner for any abatement of duty on damaged goods.

The transaction between the importer (ABC) and the respondent (B) cannot be described as the transaction of purchase and sale during the course of international trade.

Any sale of goods carried out, after the act of ‘import’ within the meaning of the Act is over, can only be described as a sale in the course of domestic trade and not a sale in the course of international trade.

Therefore, the appellant i.e. buyer (B)who had purchased the vessel in the course of domestic trade was not entitled to seek any abatement of duty under section 22 of the Customs Act,1962 on the ground on which it claimed before the Appellant authority.

Hence, in the light of above discussion, it can be inferred that the assessable value of the ship shall be the transaction value at which the importer (ABC) has paid at the time of importation i.e.Rs.24 crores.

(a) Is a person entitled to inspect or obtain copies of report made by any officer to the Settlement Commission? Can

the Settlement commission furnish copies of such report (2 marks) (b)Under what circumstances, provisional assessment under section 18 of the Customs Act, 1962 can be made?

Page 127: Indirect Taxation CA Workbook

CA. Raj Kumar Practice Booklet

Answer s per the provisions of section 127G of the Customs Act, 1962, no person shall be entitled to inspect or obtain

copies of any report made by any officer of customs to the Settlement Commission, but the Commission may, in its discretion, furnish copies thereof to any such person on an application and on payment of specified fee. However, for the purpose of enabling any person whose case is under consideration to rebut any evidence brought on record against him in any such report, the commission shall furnish a certificate copy of relevant part of such report on receipt of an application and payment of fee as specified in rules.

ccording to provisions of onal assessment can be resorted to in the following cases- (i) Where the proper officer is satisfied that an importer or an exporter is not able to produce documents or information necessary for assessment. (ii) Where proper officer deems it necessary to subject the imported or export goods to chemical or other test for the purpose of assessment of duty thereon. (iii) Where proper officer deems it necessary to make further enquiry.

does not mean loss of total package. Normally claim for pilferage would be under following circumstances:

There should be evidence of tampering with packages. There would be blank space for missing article on package

According to section 13 of the Customs Act, 1962,

(2 marks) (c)Explain briefly, legal provisions relating to pilfered goods under the Customs Act, 1962 (2 marks) (d)Explain briefly, legal provisions relating to power to summon persons under the Customs Act, 1962 (2 marks) (e)Define customs area. (2 marks)

(a) A

(b) A section 18 of the Customs Act, 1962, provisi

(c) Pilferage

- If any imported goods are pilferage after unloading and before officer has passed an order for clearance for

home consumption u/s 47 or deposit in warehouse u/s 60, the importer shall not be liable to pay duty leviable on such goods.

However, where such goods are restored to the importer, he becomes liable to pay duty.

(d) Section 108 of the Customs Act, 1962 provides that-

Any gazette Officers of Customs shall have the power to summon any person either to present either to present himself for enquiry or produce a document or any other thing in any inquiry which the officer is making under the Act.

All persons who are summoned shall be bound to attend either in persons or by an authorized agent and state the truth, make statement, produce document and other things.

s per section 2(11)(e)A of the Customs Act, 1962 , customs area means-

The area of a customs station and includes any area in which imported goods or export goods are ordinarily kept before clearance by Customs Authorities

Page 128: Indirect Taxation CA Workbook

CA. Raj Kumar Practice Booklet

NOV 2009 New Syllabus

1. Determine the total amount of excise duty payable under section 4 from the following -

(i.)Price of machinery excluding taxes and duties 5, 50,000 (ii)Installation and erection expenses 21,000 (iii)Packing Charges (primary and secondary) 11,500 (iv)Design and engineering charges 2,000 (v)Cost of material supplied by buyer free of charge 8,500 (vi)Pre-delivery inspection charges 500 Other information:

(a) Cash discount @2% on price of machinery was allowed as per terms of contract since full payment received before dispatch of machinery.

(b) Bought out accessories supplied along with machinery valued at Rs. 6,000. (c) Central Excise duty rate 16% and education cess as applicable @3% Make suitable assumptions as are required and provide brief reasons. Calculations should be supported by notes wherever, required. (5 Marks)

Answer;

STATEMENT SHOWING COMPUTATION OF ED PAYABLE Particulars Rs.

Sale price of machine excluding taxes and duties (WN-1) 5,50,000 Less: Cash Discount (2% of price of machinery, i.e., 2% of 5, 50,000 (WN-2) (11,000) Add: Packing Charges (both primary and secondary) (WN-3) 11,500 Add: Design and engineering charges(WN-4) 2,000 Add: Pre-delivery inspection charges (WN-5) 500 Add: Material supplied by buyer free of charge (WN-6) 8,500 Assessable value 5,61,500 Excise duty payable- (5,61,500 * 16.48%) 92,535

Working Notes:

1. Sales price given is exclusive of taxes and duties, therefore Sales Tax and excise duty need not be deducted. The given sale price “Basic Sale Price”.

2. TV refers to price actually paid or payable for the goods and hence, discounts of any type given at the time of transaction deductible while computing AV. it is not an advance payment but like prompt payment.

Page 129: Indirect Taxation CA Workbook

CA. Raj Kumar Practice Booklet 3. Packing charges are in connection with sale and hence, all kind of packing is includible.

4. Design and engineering charges are also payable by reason of, or in connection with sale, and hence, includible.

5. Pre-delivery inspection charges shall also form part of AV.

6. Material supplied by buyer free of charge is an additional consideration within the meaning of Explanation 1 of Rule 6 of valuation Rules, 2000 and thus, shall be added for the purposes of computation of AV.

7. Regarding bought – out accessories; it is optional for assessee to include their value in AV of the main product. It has been assumed that buyer has opted for non- inclusion of their value in AV of main product.

8. Assuming that installation and erection charges have not been incurred to bring into existence the purchased machinery, these have also not been included in AV of the machinery.

(b)XYZ Co. is engaged in the manufacture of WATER PIPES. From the following details for the month of May, 2009 compute the available cenvat credit under the Cenvat Credit Rules,2004:

Cenvat paid on purchases as detailed below:

Raw steel 22,000 Water pipe making machine 18,000 Spare part for the above machine 7,500 Grease and oil 2,800 Office equipment 20,000 Diesel 12,000 Provide explanation for treatment of various items. (5 Marks) Answer; Goods Cenvat Cr (in Rs.) Supportive explanation Raw steel 22,000 Being an eligible “input”falling under the definition given in Rule

2(k) of the Cenvat Credit Rules, 2004. Full Cenvat Credit available in view of Rule 4(1)of the Cenvat

Credit Rules,2004 Water pipe making machine

9,000 Being an eligible “Capital goods “falling under the definition given in Rule 2(a)of the Cenvat Credit Rules , 2004

The quantum of Cenvat credit available is restricted to 50% of the duty paid theron in view of rule 4(2) of the Cenvat Credit Rules,2004

Spare part for the above machine

3,750 Being an eligible “Capital goods “falling under the definition given in Rule 2(a)of the Cenvat Credit Rules , 2004

The quantum of Cenvat credit available is restricted to 50% of the duty paid theron in view of rule 4(2) of the Cenvat Credit Rules,2004

Grease and oil 2,800 Being an eligible “Input “falling under the definition given in Rule 2(k) of the Cenvat Credit Rules , 2004

Full Cenvat credit available in view of rule 4(1) of the Cenvat Credit Rules,2004

Page 130: Indirect Taxation CA Workbook

CA. Raj Kumar Practice Booklet Office Equipment NIL Being an eligible “capital goods“as doesn’t fall under the

definition given in Rule 2(a)of the Cenvat Credit Rules , 2004 It is also not an “eligible input”

Diesel NIL Not an eligible “input “as doesn’t fall under the definition given in Rule 2(k) of the Cenvat Credit Rules, 2004.

Total 37,550 (c)Small and company a SMALL SCALE INDUSTRY provides the following details. Determine the eligibility for the exemption based on value of clearances for the Financial Year 2009-10 in terms of N/N.8/2003-CE dated 1.3.2003 as Rs.(Lakhs)1. Total value of clearances during the Financial Year 2008-09 (including VAT Rs.50 Lakhs) 870 2 Total exports (including Nepal and Bhutan Rs. 200 Lakhs) 500 3 Clearances of excisable goods without payment of duty to a unit in Software Technology Park 20 4 Job work under N/N 84/94 50 5 Job work under N/N 214/86 50 6 Clearances of excisable goods bearing brand name of Khadi and Village Industries Commission

(KVIC) 200

Make suitable assumptions and provide brief reasons for your answers where necessary. (5 Marks) Answer; STATEMENT SHOWING COMPUTATION OF “400 LAKHS LIMIT” particulars Rs. Eligible Clearances for exemption[refer working note below] 400 Add: Certain Non-dutiable clearances

nil

400The aggregate value of clearances for the preceding FY 2008-09 is not exceeding 400 lakhs limits, and therefore,the unit shall be entitled to SSI exemption for FY 2009-10. Working Notes: STATEMENT SHOWING COMPUTATION OF “150 LAKHS LIMIT”]

Exports to country other than Nepal-300 lakh== Ineligible clearances Exports to Nepal and Bhutan (Non-dutiable clearances)-200 lakh= Eligible Clearance Clearances of excisable goods without payment of duty to a Unit in Software Technology Park (Non-Dutiable

Clearances)-20 lakh = Ineligible clearances Clearances on Job work basis (Non-Dutiable Clearances)-100 lakh= Ineligible clearances Clearances of excisable goods bearing brand name of another person ,Khadi and Village Industries

Commission (KVIC)-200 lakh= Eligible Clearance Total Eligible Clearances- 400 lakh

2(a)I Ltd. is a manufacturer excisable good such as polyester yarn. A ground plan of the factory was provided by the assessee to the jurisdictional CEO and the same was approved. The ground plan showed the area in which manufacturing is carried out as also the areas occupied for purpose of storage, godowns, cyclesheds, canteen as well as the housing complex for staff and workers. The assessee had a captive power plant in the approved area. The electricity generated was supplied to the housing complex as well as for use in the manufacturing activity. I Ltd. claimed cenvat

Page 131: Indirect Taxation CA Workbook

CA. Raj Kumar Practice Booklet credit on the duty paid on furnace oil used for generation of electricity as it was used within the factory and was covered by the expression “for any other purpose “in Rule 2(k) of the Cenvat Credit Rules, 2004.The Central excise department wants to deny the cenvat credit on the duty paid on furnace oil for generation of electricity which in turn is supplied to the housing complex on the ground that it is not used in relation to manufacture of the final product. Examine with the help of decided case Law if the stand of the department is correct in law. (5 Marks) Answer; Facts –

Issue-

As per the facts of the case, assessee has used furnace oil for generation of electricity which has been used for manufacturing activity and for other purposes as well.

The electricity so generated has been used in the in – house residential complex which is situated within the factory.

The department has denied cenvat credit to the extent furnace oil used for electricity generation which has been used in residential complex. However, assessee is claiming credit even thereon stating that even residential complex is a part of factory (as it is clearly shown in ground plan of factory) and thus, electricity used is within the factory, thus related inputs eligible for credit.

Legal Position-

The issue of consideration before us is whether denial of credit on inputs used for electricity generated and used in residential complex is proper.

Conclusion-

Rule 2(k) of the CCR ,2004 defines the term “input “to include,inter –alia,goods used for generation electricity or steam which is used in or relation to manufacture of final product or for any other purpose within the factory of production.

Thus, so long as electricity generated is used within the factory, credit is admissible thereon even though it may not be used in or in relation to manufacturing activity.

That being so, credit is admissible in given situation .

In light of above discussion .Department’s stand as to disallowing cenvat credit is not correct. (b)The assessee manufactured compressors and filters and removed them as “stand alone “items. He also manufactured and removed safety valves and filters on payment of duty .The assessee also supplied bought-out items like V-belts,motor,pulley etc.to their buyers.The Excise department relying on Rule 2(a)of the General Interpretative Rules (GIR)for classification has decide to include the value of safety valves and filters together with value of bought- out items in the value of compressors for purposes of duty under section 4 of the Central excise Act,1944 .Write a brief note with any decided case law whether the stand taken by the department is correct. Answer; Facts –

As per the given facts, assessee has manufactured “compressor and filters “and removing them as stand-alone items upon payment of duty .Alongwith such sale, assessee also supplied certain bought-out items.

The Department has sought addition of value of bought-out items in the value of final product of the assessee. For that purpose , it has relied upon Rule 2(a)of GIR which states that even incomplete or finished article shall be classifiable under the heading of complete or finished goods.

Page 132: Indirect Taxation CA Workbook

CA. Raj Kumar Practice Booklet Department has contended that since even incomplete articles are so classifiable, so even value of bought – out

accessories shall also be includible in the AV of “compressors and filters “removed by assessee as stand-alone items.

Issue-

Legal Position-

The issue for consideration before us is whether reliance by Department upon Rule 2(a)of GIR (which deals

with the classification matter)is proper while determining inclusion of value of bought- out – items in the value of the main product.

Conclusion-

The identical issue came up for consideration before SC in case of FRICKS INDIA LTD – 2007-SC. In that case was held that “the concept of “classification”is different from the concept of “Valuation”.

Classification decides the applicable rate on the commodity . It is followed by valuation i.e., value at which rate to be applied. Thus, relying upon Rule 2(a)of GIR while

determining valuation of goods is not proper .Issue of valuation shall be considered independent.

In the light of the above discussion, the stand of the department is not correct. (c)The assessee’s premise was searched by the Anti-Evasion wing of the excise department. A Show Cause Notice was issued alleging that the assessee had cleared goods without the cover of duty paid invoice and without accounting the same in the stock register. The assessee was required to pay the duty demanded with interest. The assessee filed an application before the Settlement Commission to put an end to the litigation and buy peace. The application was dismissed by the Settlement Commission on the ground that the petitioner in its petition had not admitted the entire duty liability .The assessee’s contention is that department is yet to substantiate the allegations made in the Show Cause Notice (SCN)and the dismissal order is not correct in law. Briefly discuss with a notice whether the action of the Settlement Commission is correct in law. Answer; Facts –

Issue-

As per the facts of the case, the assessee had applied for settlement of case made against him by Department on ground of clandestine removal of goods.

Settlement Commission had dismissed the application on the ground that the assessee did not admit entire duty liability as mentioned in SCN.

Assessee had contended that he had applied for settlement to buy peace of mind only and department had yet to substantiate the allegations made in SCN and therefore, application shall not be rejected on ground of non-admissibility of entire demand raised in SCN.

Legal Position-

The issue of consideration is whether in cases where entire duty liability is not admitted by assessee, the Settlement cannot be made and thus, such application is liable to be rejected by Settlement Commission.

Sec 32-E of the Central Excise Act, 1944 provides that an assessee can apply for settlement of his case by

filing settlement application. His settlement application shall contain “A FULL AND TRUE DISCLOSURE OF DUTY LIABILITY which has not been disclosed before jurisdictional CEO”.

The isssue for consideration before us is whether assessee could be said to have made full and true disclosure of undisclosed duty liability only when the admits the entire demand raised in the SCN or whether where his admitted duty liability falls short of the demand raised in SCN even then it can be a case of full and true disclosure of undisclosed duty liability.

Page 133: Indirect Taxation CA Workbook

CA. Raj Kumar Practice Booklet It shall be noted that applicant has inherent right to disclose any amount above Rs 3 lakhs as the disclosure is

related to assessment documents and not in relation to the SCN, Nowhere it is stated that admission of entire duty demand raised in SCN will mean ‘a full and true disclosure’.

There may be cases where admission of entire demand of SCN is not full and true disclosure as duty evaded by assessee was much higher that what CEO was able to trace.

Similarly, there can be situatation,where even admitted duty demand falls short of demand raised in SCN ,but that is full and true disclosure on part of assessee.

The Settlement Commission shall consider how the assessee is explaining the manner in which he derived his disclosed liability and then only shall decide that whatever assessee’s disclosure of duty liability (whatever he has admitted but in excess of Rs. 3,00,000) is full and true.

Conclusion-

In light of above discussion, rejection of settlement application on ground of non-admissibility of entire demand SCN is not proper.

3(a) Explain briefly the concept of “excisable goods “as amended by the Finance Act,2008. (2 Marks) Answer

”Excisable Goods “have been defined u/s 2(d) of the CETA to mean goods specified in the First and Second Schedule of CETA as being subject to excise duty and includes salt.

Moreover FA,2008 had inserted an explanation to the definition of excisable goods to provide that for the purpose of “excisable goods “the expression “goods “shall now include any article, material or substance which is capable of being sold and such goods shall necessarily be deemed to be marketable.

Eg. the bagasse, aluminium/zinc dross and other such products termed as waste, residue or refuse which arise during the course of manufacture and are capable of being sold for consideration would be excisable goods and chargeable to payment of excise duty.[ Circular No. 904/24/09]

(b)Explain briefly, how the value of goods will be ascertained for purpose of excise duty where the assessee sells the goods partly to a related person and the balance to unrelated third parties. (2 Marks)

Answer; When goods are sold partly to related persons and partly to independent buyers:

As per circular No. 643/34/2002:

There is no specific rule to cover such a contingency. Transaction value in respect of sales to unrelated buyers can not be adopted for sales to related buyer since as

per section 4(1) transaction value is to be determined for each removal. and Rule 9 can not be applied in such cases directly since it covers only those cases where all sales are to be

related buyers only.

FOR SALES TO UNRELATED BUYERS VALUATION WILL BE done as per section 4(1) (a) and, For sale of same goods to related buyers residuary Rule 11 read with Rule 9 or Rule 10 shall be applied.

(c)Write a brief note on cenvat monthly return of information relating to PRINCIPAL INPUTS in form ER –

Page 134: Indirect Taxation CA Workbook

CA. Raj Kumar Practice Booklet (2 Marks) Answer; Rule 9-A of cenvat Credit Rules , 2004 contains provisions relation to filing of monthly return regarding principal inputs. It provides for following in this regard:

(i) This return shall be filed in form: ER -6 (ii) It shall be filed on monthly basis, by 10th of the following month Provided further that where a manufacturer of final products has paid total duty of rupees 10 lakh or more including the amount of duty paid by utilization of CENVAT credit in the preceding financial year, he shall file such declaration electronically (iii) It shall contain details of opening/closing stock of principal inputs, their consumption etc. (iv) It shall be filed only by those assesses who are paying Excise duty (whether through PLA or by utilizing CCr)

of Rs 1 crore or more. Other assesses have been exempted from filing any return of principal input.

(d)Briefly mention any four categories of persons who are exempted from obtaining Registration under Rule 9(2) of the Central Excise Rules, 2002 (2 Marks) Answer; Sec 6 read with Rule 9 (1)of CER ,2002 makes registration mandatory for every person who is engaged in manufacture, storage ,trade or otherwise use of any excisable goods .However, Rule 9(2)empowers CBEC to grant exemption from registration. In exercise of that power, CBEC has exempted following persons from registration:

1. Manufacturer of -100% Exempted goods or Nil Duty goods;(Non dutiable goods) 2. R/M Supplier getting goods manufactured of Job –Work (where ED liability is at end of job-worker only); 3. Manufacturer doing manufacturing process in Customs Warehouse under section 65 of custom act. 4. Trader of excisable goods (other than First Stage Dealer and Second Stage Dealer) 5. User of excisable goods (when he is not availing end-use based exemption) Further, even a 100% EoU need not obtain excise registration if it is having no inter –linkage (neither in terms of sale nor in terms of purchase) with the Domestic Tariff Area. (e)Write a short note on the principal of “UNJUST ENRICHMENT “under Central Excise Law.(2 Marks)

Answer; Doctrine of Unjust Enrichment:

Unjust enrichment refers to a situation in which a person becomes rich unjustifiably, i.e., at the expense of other.

In excise law, this situation arises because of indirect nature of excise duty .No doubt, excise is payable by the manufacturer but the incidence of duty is borne by the ultimate consumer who pays it as a part of the final price.

Now, if on any account, a refund is granted to manufacturer instead of the consumer who borne the burden of duty, such refund will have the effect of making the manufacturer rich at the expense of the ultimate consumer which is unreasonable & against the general principal of equity.

Summarily, the doctrine of unjust enrichment seeks to ensure that the person who has passed on duty incidence does not get the refund of such duty. Under Central Excise Law this doctrine has been incorporated through sec 11-B ,Sec 11-B which deals with refund provides that where duty is admitted to be refundable ,then such duty shall be credited to Consumer Welfare Fund instead of giving it to applicant.

Page 135: Indirect Taxation CA Workbook

CA. Raj Kumar Practice Booklet 4(a) Determine the taxable turnover, input tax credit and net VAT payable by a works contractor from the details given below on the assumption that the Contractor maintains sufficient records to quantity the labour charges Assume Output VAT at 12.5%: Rs. Lakhs i. Total contract price (excluding VAT) 100 ii Labour charges paid for execution of the contract 35 Iii Cost of consumables used not involving transfer of property in goods 5 iv Material purchased and used for the contract taxable at 12.5% VAT (VAT

included) 45

The contractor also purchased a plant for use in the contract for Rs. 10.4 Lakhs. In the VAT invoice relating to the same VAT was charged at 4% separately and the said amount of Rs.10.4 lakhs in inclusive of VAT .Assume 100% input credit on capital goods. Make suitable assumption wherever required and show the working notes. (5 Marks) Answer;

COMPUTATION OF TAXABLE TURNOVER &VAT LIABILITY Total contract Value (exclusive of VAT) 100 lakhs Less: Value of “Labour, service and other charges” (40 lakhs) (iii) Value of Labour Charges 35 lakhs (iv) Cost of Consumable whose property does 5 lakhs

Not get transferred in execution of works contract 60 lakhs VAT liability on this taxable turnover shall be computed as follows: [60,00,000 *12.5%]= 7,50,000 Assumptions

As per the provisions, profit earned by the Work contractor to the extent it is relatable to supply of labour and services shall also be deductible as “value of labour, service and other charges”. For the purpose of given question, it has been presumed that the amount of labour charges and consumable already includes the profit attributable thereto.

It has been assumed no declared goods were used in execution of works contract. Infact ,the given question specifically provides for taxing entire taxable turnover @12.5%

COMPUTATION OF INPUT TAX CREDIT

C. Credit of INPUTS used in execution of Works Contract (45, 00, 000 * 12.5/112.5) 5, 00,000

D. Credit of CAPITAL GOODS (10,40,400 * 4/104) 40,000 5,40,000

COMPUTATION OF NET VAT LIABILITY Gross VAT liability (60 Lakhs * 12.5%) 7, 50,000 Less: Input Tax Credit (as computed above) (5, 40,000) NET VAT LIABILITY 2,10,000 (b)Calculate the VALUE OF TAXABLE SERVICE under service under Cargo Handling Services of Cargo Ltd., providing brief reasons where required with suitable exemptions based on the following information for the month of April ,2009

Page 136: Indirect Taxation CA Workbook

CA. Raj Kumar Practice Booklet 1. Total amount charged for all services 402 Receipts for services in relation to export cargo and Handling of

passenger baggage including in (i)above13

3 Charges for storage and cleaning of empty containers of Shipping lines included in (i)above

10

4 Charges for packing and transport of cargo included in (i)above 35 Charges for handling of agricultural produce included in (i)above 26 Charges for transportation of Cargo included in (i)above 5 (5 Marks) 5(a)M/s M Construction Ltd Constructs, builds and sells premises/flat in a building .During the course of development and construction of building it enters into a FLAT PURCHASES AGREEMENT in terms of which the buyers are allotted flats in the building. The buyers pay the consideration to M/s M construction in installments. The said ‘flat purchases agreement’ reflects the entire consideration for the purchases of flat. This agreement is registered and advance payments in installments are collected. The contention of M/s M Construction is that the consideration is for the purchase and sale of the entire flat/premises and the company does not carry out any construction activity .Examine with a brief note whether the company is liable to pay service tax in terms of the Finance Act, 1944. Answer; “Residential Complex Construction Service”is within the coverage of service tax. What is being covered is service of construction and not the contract of sale of flats.

Hon’ble SC in case of K RAHEJA hold out that sale of immovable property is not covered and that transaction will attract stamp duty liability, not service tax liability .

Doubts have been expressed where a constructor/builder obtains payment in advance (in installments –which may be construction linked (relating to progress of construction) or otherwise fixed installments)

whether then the transaction is not that of sale of immovable property. Regarding this, very recently, CBEC has come up with a clarification stating following points :

In such case, the initial agreement between the PROMOTERS / BUILDERS/DEVELOPERS and the ultimate owner/buyer is in the nature of “AGGREEMENT TO SELL”.

Such a case, as per the provisions of Transfer of Property Act, does not by itself create any interest in or change on such property. The property remains under the ownership of the seller (in the instant case. Builder/Promoter/Developer. It is only after completion of the construction and full payment of the agreed sum that a SALE DEED is executed and only then the ownership of the property gets transferred to the ultimate owner/buyer.

Therefore, any service provided by such seller in connection with the construction of residential complex till the execution of such sale deed would be in the nature of “self-service consequently, would not attract ST.

Even where a buyer makes a construction linked payment after entering into agreement to sell the flat, the nature of transaction is NOT A SERVICE but THAT OF A SALE.

Thus, company is not liable to pay service tax in given case (b)Good Luck Agencies is engaged in the purchase of lottery tickets in bulk from the state government and sell them subsequently on behalf of the Government. However no service tax is paid on the said activity. The Department has sought to levy service tax under the category of ‘Business Auxiliary Service’ Section 65(19)as service in relation to promotion or marketing of service provided by the client. Discuss briefly with a note and decided case law, if any, whether the action of the department is correct in law. 6(a) Briefly explain whether the following are chargeable to service tax under the provisions of the Finance Act, 1994:

Page 137: Indirect Taxation CA Workbook

CA. Raj Kumar Practice Booklet (i) Service provided by a person to another person in relation to information technology software for use in course

or furtherance of business or commerce. (ii) Transaction allowing another person to use goods without giving legal right of possession and control to the

user of the goods. (2 Marks each)

(b) Write a brief note explaining to circumstances under which expenditure or costs incurred by a service provider as a pure agents of the recipients of service shall be excluded from the value of taxable services under the Service Tax (Determination of value)Rules,2005 (4 Marks) Answer; Rule 5 of Service Tax valuation Rules, 2006 provides that while valuating taxable service, any expenditure incurred by service provider as PURE AGENT of the recipient of the service shall be excluding from the value .For claiming such exclusion ,the following conditions shall be satisfied:

(a) the service provider acts as a pure agent of the recipient of service when he makes payment to third party for the goods or services procured;

(b) the recipient of service, received and uses the goods or services so procured by the service provider, in his capacity as pure agent of the recipient of service;

(c) the recipient of service is liable to make payment to the third party; (d) the recipient of service authorizes the service provider to make payment on his behalf; (e) the recipient of service knows that the goods and services for which payment has been made by the service

provider shall be provided by the third party; (f) the payment made by the service provider on behalf of the recipient of service has been separately indicated in

the invoice issued by the service provider to the recipient of service; (g) the service provider recovers from the recipient of service only such amount as has been paid by him to the

third party; and (h) the goods or services procured by the service provider from the third party as a pure agent of the recipient of

service are in addition to the services he provides on his own account. Pure agent: “Pure agent” means a person who:-

(a) Enters into a contractual agreement with the recipient of service to act as his pure agent to incur expenditure or costs in the course of providing taxable service;

(b) neither intends to hold nor holds any title to the goods or services so procured or provided as pure agent of the recipient of service;

(c) does not use such goods or services so procured; and (d) receives only the actual amount incurred to procure such goods or services.

(c)Briefly explain the provisions in the Service Tax Rule, 1994, relating to furnishing of list of records at the time of filing of return for the first time. (4 Marks) Answer; Rule 5 of Service Tax Rules, 1994 contains provisions relating to ‘’Service tax records’’. It provides that records maintained under by any other law are acceptable as ST Records (i.e., no separate records required to be maintained) at time of filing his 1st ST return, the assessee shall submit “List of Records”. The list shall cover following records:

Page 138: Indirect Taxation CA Workbook

CA. Raj Kumar Practice Booklet 1. All records prepared or maintained by assessee for accounting of transactions in regard to

(a) Providing of any service, whether taxable or exempted. (b) Receipt or Procurement of input services and payment for such input services. (c) Receipt, purchase, manufacturing, storage, sale or delivery, as the case may be, in regard of inputs and

capital goods. (d) Other activities, such as manufacture and sale of goods, if any.

2. All financial records maintained by him in the normal course of business.

3. All such records shall be preserved at least for a period of 5 years immediately after the financial year to which such records pertain.

(d)Briefly explain the following with reference to the Service Tax Rule,1944 and the Finance Act,1994:

(i) Gross amount charged (ii) E-payment of service tax. (2 Marks each)

Answer; (i)Gross amount charged: When service is rendered for monetary consideration, then taxable value is “Gross Amount Charged ”[Sec 67 ].Following points shall be noted in this regard.

a. Gross Amount Charged: connects that while valuing any service no deduction shall be allowed in respect of any expenditure or costs incurred in course of providing such service. b. If “Gross amount charged “is inclusive of service tax element, then service tax component shall be excluded while computing the taxable value. c.“Gross amount charged “will include any amount received before, during or after provisioning of service.

(ii) e-payment of service tax: E-payment facility has been recently introduced in service tax. Presently, e-payment is option for most of assessees. However, if assessee has paid service tax of Rs 10 lakhs or more in preceding financial year, then e-payment is mandatory for him. For assesses making e-payment of service tax (whether optionally or mandatorily), the due date shall be 6th of following month instead of 5th of following month

(c)Briefly explain the importance of VAT invoice. (4 Marks)

Answer; Importance of VAT invoice (tax invoice)

Invoices are crucial documents for administering VAT. In the absence of invoices VAT paid by the dealer earlier cannot be claimed as set off. Invoices should be preserved with full care. In case any original invoice is lost or misplaced, a duplicate authenticated copy must be obtained from the issuing dealer.

A VAT invoice:

i. Helps in determining the input tax credit;

ii. Prevents cascading effect of taxes;

iii. Facilitates multi-point taxation on the value addition;

iv. Promotes assurance of invoices;

v. Assists in performing audit and investigation activities effectively;

Page 139: Indirect Taxation CA Workbook

CA. Raj Kumar Practice Booklet vi. Checks evasion of tax.

7. From the following particulars determine duty payable of the imported equipment: (i) ASSESSABLE VALUE of the imported equipment US $ 19,100 (ii) Date of Bill of Entry 25.4.2009 basic customs duty on this date 20% and exchange rate notified by the Central Board of Excise and customs US $ 1=Rs 65. (iii) Date of Entry inwards 21.4.2009 basic customs duty on this date 16% and exchange rate notified by the Central Board of Excise and Customs US $ =Rs. 50. (iv) Additional duty payable under section 3(1)and (2)of the Customs Tariff Act,1975: 15% (v) Additional duty under section 3(5)of the Customs Tariff Act,1975 : 4% (vi) EC @2% and SHEC @1%

Make suitable assumptions where required and show the relevant working and round off your answer to the nearest Rupee. (5 Marks)

Answer;

STATEMENT SHOWING COMPUTATION OF CUSTOMS DUTY LIABILITY

Particulars Amount Assessable value u/s 14 (1)Value - (19,100*65)

6,56,500

Basic Customs Duty[BCD] @ 20% 1,31,300 Total (A) 7,87,800 Countervailing Duty [CVD]- 15.45%(inclusive of EC&SHEC on excisable goods) on total (A) 1,21,715 EC (on imported goods) on BCD+CVD = [1,31,300+1,21,715] =2,53,015 @ 2% 5,060 SHEC (on imported goods) on BCD+CVD = [1,31,300+1,21,715] =2,53,015 @ 1% 2530 Total (B) 9,17,105 Countervailing Duty [CVD]-u/s – 3(5) @ 4% on [Sec 14 (1)Value +BCD +CVD +EC+SHEC] = 9,17,105

36,685

TOTAL CUSTOM DUTY PAYABLE 2,97,290

Notes Bill of entry filed is normal bill of entry. Thus, applicable rate of BCD shall be as prevailing on the date of

filing of this bill of entry i.e.20%. So far as applicable rate of exchange is concerned, applicable rate is as prevailing on the date of filing of bill of

entry. Thus, IS= Rs 65 is the applicable rate of exchange for valuation purposes. 8. The assessee had imported capital goods under a license with a condition to fulfill an export obligation within a certain time limited. The assessee failed to discharge the export obligation within the said time limit. Consequently the department invoked the bank guarantee and realized the amount. However, subsequently the assessee was able to fulfill the exports obligation and the department cancelled the bank guarantee. The assessee the thereafter filed a refund claim for the amount realized by invocation of the bank guarantee by the department..The department rejected the refund claim on the ground that in was barred in terms of section 27 of the Customs Act,1962 as the assessee had not been able to establish that the incidence of duty had not been passed on by him to any other person .Examine briefly with the help

Page 140: Indirect Taxation CA Workbook

CA. Raj Kumar Practice Booklet of any decided case law whether the stand taken by the department is correct in law. (5 Marks) Answer; Facts

Issue-

In the instant case, assessee imported capital goods under license furnishing bank guarantee under taking to fulfill export obligation within certain time. On his failure to fulfill his obligation ,his guarantee was encashed by the department.

Later on assessee fulfilled his obligation as to export and applied for refund of amount encashed by the department.Departemnt has rejected refund only on ground of unjust enrichment.

Legal position

The issue for consideration before us in whether denial of refund on ground of unjust enrichment is correct in law.

Conclusion

It is settled law that refund of duty under customs shall always be subject to doctrine of unjust enrichment .But then it shall be noted that furnishing of bank guarantee was not equivalent of payment of duty as that was just for security purposes.

It is settled principal that bank guarantee can be encashed only for confirmed demands. In the given case, Department didn’t pass any demand order against assessee and directly encashed bank guarantee. This is a case of wrong encashment of bank guarantee.

Assessee basically is claiming repayment of his bank guarantee and not refund of excise duty, Hence, the provisions of unjust enrichment shall not be applicable. On identical facts, it was so held by Tribunal in case of GRASIMS INDUSTRIES LTD -2005.

In view of the above discussion, the denial of refund by department on ground of unjust enrichment is not correct in law. 9(a)Explain briefly the expression ‘Person – in charge’under the Customs Act,1962 (2 Marks) Answer; “Person – In -Charge “have been defined u/s 2(31)of the Customs Act,1962 to mean-

(a) In relation to a vessel --- the master of the vessel; (b) In relation to an aircraft ---- the commander or pilot – in –charge of the aircraft; (c) In relation to a railway train ---- the conductor, guard or other person having the chief direction of the train. (d) In relation to any other conveyance --- the driver or other person – in –charge of they conveyance.

(b)Explain briefly the provisions of the Customs Act,1962 ,relating to the powers vested in the customs officers to take samples? (2 Marks) Answer; Sec 144 of the Custom Act,1962 empowers customs officer to take samples. Samples may be taken and dealt with in the following manner:

Samples may be taken at the time of entry or at item of clearance of goods. Samples can be taken only in presence of owner of goods Samples can be taken only for purposes of examination or testing or for ascertaining the value of goods After the purpose is over, samples shall be restored to the owner of the goods. If such restoration is not

possible on account of sample having got destroyed in course of examination and testing, duty shall not be chargeable on such samples.

Page 141: Indirect Taxation CA Workbook

CA. Raj Kumar Practice Booklet (c)Explain with a brief note how the duty is arrived the Customs Act, 1962 where the imported goods consist of articles liable to different rates of duty. (2 Marks) Answer; Section 19 of Customs Act speaks of assessment of duty in a case where goods consist of “Set of articles”.It provides that Where goods consist of a set of articles, duty shall be calculated as follows:

a. Articles liable to duty with reference to quantity ---------- shall be chargeable to that duty; b. Articles liable to duty with reference to value shall,

• If they are liable to duty at the same rate,----- be chargeable to duty at that rate, and • If they are liable to duty at different rates,-- be chargeable to duty at the highest of such rates;

c. Articles (in the set) not liable to duty ------------ shall be chargeable to duty at the rate at which articles liable to duty with reference to value are liable under clause (b).

Exceptions

SN Nature of Article in set Treatment for assessment Condition

1

Any Article in the set Shall be chargeable to duty separately at the rate as applicable to it

If the importer produces evidence to the satisfaction of the PO regarding its value out of the total value of set.

2. Accessories /Spare parts/ maintenance and repairing Implements, in the set.

Shall be chargeable at the same rate of duty to which the main article is chargeable

If accessories etc. satisfy the conditions of Accessory Conditions Rules, 1963:

(i) these are compulsorily supplied along with main article; and

(ii) No separate charge is made for such supply, their price being included in the price of the main article.

(d)Write a brief note on the ‘residual method’ of determination of value of imported goods under the Customs Valuation (Determination of Value Of Imported Goods)Rules,2007. (2 Marks) Answer; Rule 9 of the customs Valuation Rules , 2007 is known as “residual method of valuation”. The reason being that it is applicable when valuation of goods is not possible by application of other valuation rules (1) Subject to the provisions of Rule 3,

Where the value of imported goods cannot be determined under the provisions of any of the preceding rules, the value shall be determined

using reasonable means consistent with the principles and general provisions of these rules and Sec 14(1) of the customs Act, 1962.

(2) Provided that value shall not be determined on the basis following -

Arbitrary or fictitious values. The selling price in India of the goods produced in India;

Page 142: Indirect Taxation CA Workbook

CA. Raj Kumar Practice Booklet The price of the goods on the domestic market of the country of exportation; The price of the goods for the export to a country other than India; The cost of production Except as per the provision of Rule 8; A system which provides for the acceptance for customs purposes of the highest of the two alternative values; Minimum customs values.

(c)Briefly state the rights of the owner of warehoused goods under the customs Act,1962 (2 Marks)

Answer; Sec 64 of the Customs Act, 1962 grants certain rights to the owner of the warehoused goods. These rights are as follows: With the sanction of the proper officer, and on payment of the prescribed fees, the owner of any goods either before or after warehousing the same:-

(a) Inspect the goods (b) Separate damaged or deteriorated goods from the rest; (c) Sort the goods or change their containers for the purpose of preservation, sale, export or disposal of the goods; (d) Deal with the goods and their containers in such manner as may be necessary to prevent loss or deterioration or

damage to the goods (e) Show the goods for sale; or (f) Take samples of goods without entry for home consumption, and if the proper officer so permits, with out

payment of duty on such samples.

Page 143: Indirect Taxation CA Workbook

CA. Raj Kumar Practice Booklet

NOV- 2009- OLD SYLLABUS

Part-A

Question 1 (a)Briefly explain the following with reference to the provisions of the Central Excise Act, 1944: (i) Adjudicating Authority (ii)Excisable goods (2X2=4 Marks) (b)Mention the last date of filing the following returns under the Central excise rules, 2002; (i) ER-4 for the financial year 2009-10 (ii)ER-7 for the financial year 2009-10 (1X2=2 Marks) (c)Distinguish between compounded levy scheme and duty based on annual production capacity under central excise (4 Marks) (d)WM Ltd.In manufacturing a product which is captively consumed to produce a final product, which is exempt, from the payment of excise duty. The intermediary product, is having a distinct market of its own. The company is of view that since the final product is exempt; no duty liability arises on intermediary product also. The department objected the view of the assessee. Discuss, with reference to a decided case law, if any, whether the view of company is justifiable? (5 Marks) (e)An assessee sold certain goods to PQR Company Limited for Rs.20,000 on 09.09.2009.The buyer is a related person as defined under section 4(3)(b)of the Central Excise act,1944.The buyer did not sell the goods but used it as intermediary product. The cost of production of the goods was Rs.16,000. What should be the assessable value? What should be the assessable value, if the goods were sold to unrelated person for Rs.20,000, who also used it as intermediary product? You may assume that the price charged from the buyer is excluding excise duty and other taxes. (5 Marks)

Answer

(a) (i)- Section

(ii)-Excisa

2(a) of the Central Excise Act, 1944 defines adjudicating authority to mean any authority competent to pass any order or decision under this Act.However,it does not include the following: (a)CBEC constituted under the provisions of Central Board of revenue act,1963: (b)Commissioner of Central Excise (Appeals) (c)Appellate Tribunal

ble goods have been defined vide section 2(d) of the Central Excise Act,1944. It means goods specified in the First Schedule and the Second Schedule to the Central Excise Tariff Act, 1985

as being subject to duty of excise and includes salt. An explanation has been added in this definition with effect form 10.5.2008 which states that for the purposes

of this clause “goods “includes any article material or substance which is capable of being bought and sold for a consideration and such goods shall be deemed to be marketable.

Page 144: Indirect Taxation CA Workbook

CA. Raj Kumar Practice Booklet

ER-4 is filed for the preceding financial year to which it relates is filed annually by 30th November of the succeeding year [Rule 12(2) of the Central Excise Rules, 2002].

Therefore, the last date for filing ER-4 for the financial year 2009-10 shall be 30.11.2010.

ER-7 is filed for the financial year to which it relates is filed by the 30th April of the succeeding financial year [Rule 12(2A) of the Central Excise Rules, 2002].

Therefore, the last date for filing ER-7 for the financial year 2009-10 shall be 30.04.2010

pounded levy scheme:

(b) (i)-

(ii)

(c) Com Under compounded levy scheme, the manufacturer has to pay the prescribed duty for a specified period on the

basis of certain factors relevant to production, like size of equipment employed, production capacity or some other criteria [Rule 15 of Central Excise Rules, 2002]

Compounded levy scheme is presently applicable to stainless steel pattas/patties and aluminum circles. It is an optional scheme i.e. the manufacturer can opt to pay duty as per normal rules.

Duty based on production capacity:

In case of certain products, Central Government, by notification ,can specify that duty on such notified products will be leived and collected on the basis of annual production capacity of the factory ,[Section 3A (1)of the Central Excise Act,1944].

Excise Duty on pan masala and gutkha, unmanufactured tobacco, chewing tobacco, zarda etc. is payable on the basis of annual production capacity under section 3A.

The scheme is compulsory.

The duty of excise is a duty on manufactured goods which are movable and marketable. If any manufactured goods satisfy the movability and marketability conditions, it would become dutiable even if it is an intermediate product and the final product is not dutiable.

Therefore, in the given case the intermediate product would be dutiable even though it is captively consumed and the final product is not dutiable as it has a distinct market of its own and is marketable.

The Supreme Court’s view in the case of White Machine -2008 was also the same. In the above case, the assessee manufactured C.I.Castings which was captively consumed for production

of C.I. Chilled Rolls. These chilled Rolls were exempt from duty. The Apex Court opined that since the final product was exempt the C.I.Castings would become dutiable if

they satisfied the marketability condition. Therefore, the company’s view is not justifiable and the Department’s view is acceptable.

The proviso to rule 9 of the Central Excise Valuation (Determination of Excisable Goods)Rules,2000 lays down that in a case where the related person does not sell the goods but uses or consumes such goods in the production or manufacture of articles ,

the value thereof shall be determined in the manner specified in rule 8. Rule 8 provides that where the excisable goods are not sold by the assessee but are used for consumption by him or his behalf in the production or manufacture of other articles, the value shall be 110% of the cost of production or manufacture of such goods.

Therefore, when the goods are sold to a related person, the assessable value shall be 110% of Rs.16, 000(Rs.16, 000+Rs.1, 600) i.e.Rs.17, 600.

(d)

(e)

Page 145: Indirect Taxation CA Workbook

CA. Raj Kumar Practice Booklet However, when the goods are sold to unrelated buyer, the assessable value will be Rs.20, 000. (Rule 9 of

valuation Rules, will not be applicable in this case .Here transaction value shall be taken as A.V.) Question 2: (a)M/s.XYZ Ltd. shifted its factory form Sitapur to Rampur and transferred all the available inputs and capital goods to the new site. The inputs, capital goods and the balance of unutilized CENVAT credit were duly received and accounted for in the registers of the new unit. The said balance of unutilized CENVAT credit transferred was Rs.8, 00,000.However, the quantum of CENVAT credit attributable to the inputs and capital goods so transferee to the new site was Rs.6, 00,000 only. The Department raised the plea that the assessee was entitled to transfer only Rs.6, 00,000 of CENVAT credit had not the entire balance of unutilized credit of Rs.8,00,000. Explain, with the help of a decided case law, if any, whether Department’s plea is justified in law? (b)What are the circumstances under which the certificate of registration can be revoked or suspended under central excise? (4 Marks) (c)To whom should the matter be referred in a case where the Committee of Chief Commissioners of Central Excise differs in its opinion as to the legality or propriety of the decision or order of the Commissioner of Central excise and how will the matter be settled under section 35E(1)of the Central Excise Act,1944? (3 Marks) (d)Discuss in brief the powers of Settlement Commission to grant immunity form prosecution. (3 Marks) (e)What are the difference between the short levy and short payment? (2 Marks) Answer (a) As per Rule 10 of the CENVAT Credit Rules, 2004,

if a manufacturer of the final product shifts his factory to another site with the specific provision for transfer of liabilities of such factory, he shall be allowed to transfer the CENVAT credit lying unutilized in this accounts to the new site

If the stock of inputs as such or in process, or the capital goods is also transferred along with the factory or business premises and the inputs, or capital goods, on which credit has been availed of, are duly accounted for to the satisfaction of Assistant/Deputy Commissioner of Central Excise.

The Madras High Court in the case of CCE, Pondicherry Vs.CESTAT 2008 (Mad)has also affirmed this position. In this case, the High Court has held that-

Rule 10 of the CENVAT credit Rules, 2004 does not provide that the assessee could transfer the CENVAT credit corresponding only to the quantum of inputs or capital goods transferred to the new factory.

Thus, the plea of Department is not justified presuming that M/s.XYZ Ltd. shifted its factory form Sitapur to Rampur with the specific provision for transfer of liabilities of such factory and the inputs or capital goods on which credit has been availed of are duly accounted for to the satisfaction of assistant/Deputy Commissioner of the Central Excise.

The certificate of registration under the Central Excise Act, 1944, can be suspended or revoked by the Deputy Commissioner or the assistant Commissioner of Central Excise under the following circumstances.

(b)

Page 146: Indirect Taxation CA Workbook

CA. Raj Kumar Practice Booklet When the assessee or any person under his employment has committed any breach of any condition of the

Central Excise Act or any rules made there under. The assessee or a person under his employment has been convicted of an offence under section 161 of the

Indian Penal Code.(e.g. bribe to officer)

The Finance Act,2008 has added a proviso to section 35E (1)to cover a case where the Committee of Chief Commissioners of Central Excise differs in its opinion as to be legality or propriety of the decision or order of the commissioner of central excise.

In Such a case the committee shall state the point or points on which it differs and make a reference to the BOARD,

(c)

which after considering the facts of the order, if is of the opinion that the decision or order passed by the

commissioner or any other Commissioner to apply to Appellate Tribunal for determination of such points arising out of decision or order.

The Settlement Commission, subject to certain provisions, has the power to grant immunity from prosecution in respect of the case covered by the settlement, if the applicant has co-operated with the commission and has made full and complete disclosure.

If the payment is not made as per the settlement order or any particulars are concealed or any false is given, the immunity is withdrawn

Immunity can be granted only in respect of prosecution for any offence under the Central Excise Act and not in respect of prosecution for any offence under the Indian Penal Code or any other Central Law.

(d)

(e) Short levy:- Short levy arises when the charge itself is done at a lower rate. It may arise out of wrong classification. Short Payment: -Short payment arises out of a short levy or short payment of a correct levy. It is a case of less payment of excise duty than what is due. Question 3. (a)M/s.silver Enterprises is manufacturing packing material namely printed cartoons of paper and paper board with brand name of another company. Its turnover for the year 2008-09 was Rs.2 Crore and in the year 2009-10 it is expected to increase by 50% The firm has approached you as a consultant whether it is eligible for concession under Notification No. 8/2003 dated 01.03.2003 as amended. Is exemption available to the firm? If yes, what is the limit of turnover, for which the exemption is available for the years, i.e.2008-09 and 2009-10? (5 Marks) (b)Which reasons shall not be considered as special and adequate for awarding sentence of imprisonment for a term of less than 6 months as per section 9(3) of the Central excise act, 1944? (4 Marks) (c)How can the goods kept in a warehouse for the purpose of export be diverted for home consumption? Explain briefly. (4 Marks) (d) Can the Department file an appeal in respect of same assessee, if in respect of some years, no appeal was filed involving identical dispute? (2 Marks) Answer: (a) yes,

The SSI exemption will be available to M/s.silver Enterprises as Notification No. 8/2003 has been amended to provide that SSI exemption will be available to packing material including printed cartoons of paper or paper board affixed with brand name of another company.

Page 147: Indirect Taxation CA Workbook

CA. Raj Kumar Practice Booklet It has been presumed that the turnover of M/s.silver Enterprises for the year 2007-08 was less than Rs.

400Lakh. For the year 2008-09, the exemption will be available, since, in the year 2008-09, the turnover of M/s.Silver

Enterprises is Rs.2 crore which is less than Rs.400 lakh, it is eligible for small scale exemption in the year 2009-10 .For the year 2009-10,turnover upto Rs.150 lakh

will be fully exempt.

As per Central excise Act, 1944, for the purposes of section 9(1) or 9(2), the following shall not be considered as special and adequate reasons for awarding a sentence of imprisonment for term of less than 6 months namely: (i) the fact that the accused has been convicted for this Act. (ii)the fact that in any proceeding under this Act, other than a prosecution, the accused has been ordered to

or the goods in relation to such proceedings have been ordered to be confiscated or any other action has been taken against him for the same act which constitutes the offence; (iii) The fact that the accused was not the nd was acting merely as a carrier of goods or otherwise was a secondary party in the commission of the offence: (iv) The f the accused (too young or too old)

The goods kept in a warehouse for being exported can be diverted for home consumption in the following manner:-

he permission of the Deputy or Assistant Commissioner of Central Excise, the goods can be cleared for home consumption on invoice prepared under Rule 8 on payment of duty, interest and any other charges on GAR-7 challans. Necessary entries are to be made in the export warehouse register maintained by the exporter in the warehouse.

t will be permitted in the Running Bond Account equivalent of the duty involved in the goods so diverted, which shall not exceed amount of duty debited on the basis of ARE-3 on which such goods were received in the warehouse.

oods can also be diverted for home consumption even after clearance of goods form warehouse under ARE-1 .The documents will be cancelled as per Notification No.46/2001 CE(NT)dated 26.6.2001 as amended. The intimation of such cancellation is to be given to Deputy /Assistant Commissioner having jurisdiction over the warehouse .Credit in Running Bond Account will be permitted in the same manner as mentioned above.

xporter has to pay an interest @24% p.a.on the amount of duty payable on such goods form the day of clearance form the factory of production or any other premises approved till the date of payment of duty and clearance.

It has been held by the Supreme Court in the case of C.K.Gangadharan

(b) section 9(3) of the

the first time for an offence underpay a

penalty

principal offender a

age o

(c)

(i) With t

(ii)Credi

(iii) G

(iv) The e

(d) 2008 (Sc) that –

merely because in some cases Revenue has not preferred an appeal that does not operate as a bar for the Revenue to prefer an appeal in another case where there is just cause for doing so or it is in public interest to do so or for a pronouncement by the Higher Court when divergent views are expressed by the different High Courts.

However, the Supreme Court in the case of J.K.Charitable Trust 2008 (SC ) has held that if in some years , in respect of same assessee, no appeal was filed involving an identical dispute , revenue can be precluded form filing an appeal if the fact situation in subsequent years remains the same.

Page 148: Indirect Taxation CA Workbook

CA. Raj Kumar Practice Booklet

PART-B Question 4 (a)Briefly explain the following with reference to the Customs (Determination of Value of Imported Goods) Rules, 2007: (i)Goods of the same class or kind (ii)Computed value (3+3=6Marks) (b)Mr.Vasu, the assessee, was summoned under section 108 of the Customs Act, 1962, to give his statement in an inquiry. He filed the application for anticipatory bail in the District and Session Court which was disposed on the ground of being premature. However, he later moved to the High Court, which granted him anticipatory bail with a direction to the authorities that he should not be arrested without giving a seven day’s prior notice to him. The Revenue contended that the order passed by the High Court was illegal and erroneous. Explain, with the help of a decided case law, if any, whether the stand taken by revenue is sustainable in law?(5 Marks) (c) Goods manufactured or produced in India, which were earlier exported and thereafter imported into India will be treated at par with other goods imported into India. Is the given proposition correct or any concession is provided on such import? Discuss briefly? (5 Marks) (d)Ascertain whether the exporter is entitled to duty Drawback in the following independent cases and if yes, what is the quantum of such duty drawback: (i)FOB value of goods exported is Rs.50; 000.Rate of duty drawback on such export of goods is 1% (ii)FOB value of 2,000 kgs goods exported is Rs.2, 00, 000.Rate of duty drawback on such export is Rs.30 per kg.Market Price of goods is Rs.50, 000 (in whole sale market) Answer

(i) As per rule 2(1)(c) (a)

of Customs Valuation (Determination of Value of Imported Goods)Rules,2007- Goods of the same class or kind, means imported goods that are within a group or range of imported goods

produced by a particular industry or industrial sector and includes identical goods or similar goods. (ii) As per Rule 2(1)(a)of the said rules, computed value means- the value of imported goods determined in accordance with Rule 8. As per Rule 8, subject to the provisions of rule 3, the value of imported goods shall be based on a computed value, which shall consist of the sum of-

The cost or value of materials and fabrication or other processing employed in producing the imported goods.

Page 149: Indirect Taxation CA Workbook

CA. Raj Kumar Practice Booklet An amount for profit and general expenses equal to that usually reflected in sales of goods of the same class or

kind as the goods being valued which are made by procedures in the country of exportation for export to India; The cost or value of all other expenses under sub-rule (2) of rule 10.

and taken by the Revenue is valid. A similar issue has been dealt by the Supreme Court in the case of Padam (b)The st

Narain Aggarwal 2008, wherein the Apex Court has observed that – The power to arrest by a Custom Officer under section 104 of the Customs Act, 1962 is statutory in charter and

cannot be interfered with. Supreme Court pronounced that the direction to issue 10 days prior notice before arrest even in case of non-

bailable offence could not be said to be legal or in consonance with law. Firstly, the order passed by the High Court was a blanket one and granted protection to respondents in respect

of any non-bailable offence. Secondly, it illegally obstructed, interfered and curtailed the authority of Custom Officers form exercising statutory power of arresting a person said to have committed a non-bailable offence by imposing a condition of giving 10 days prior notice , a condition not warranted by law.

Hence, the order of the High Court was set aside. Therefore, in the given case, it can be concluded that the stand taken by the revenue is sustainable in law.

The given proposition is correct i.e., goods produced in India, which were earlier exported and thereafter imported into India will be treated at par with other goods imported into India [Section 20 of the Customs Act, 1962]. However, the following concessions are being provided in this regard:-

Re-import for Repairs

(c)

The re-importation is for repairs within 3 years The goods must be re-exported after repairs within 6 months from the date of import, then no duty shall be payable.

Re-import after Repairs:

Indigenously manufactured goods exported for repairing are subsequently re-imported then, importer will be liable to pay duty, on the value comprising of fair cost of repair including cost of material used in repairs, insurance and freight

charges both ways. Further, it is not applicable if the repairs amount to re-manufacture.

Re-import on Rejection:

If the same goods which were exported are re-imported within 3 years, by the same person ; no custom duty is payable .

BUT assessee has to pay, EXCISE DUTY (which was exempted at the time of export) and DUTY DRAWBACK (allowed at the time of export.)

However, the notification is not applicable to exports from EPZ or EOU, etc.

(d)

Page 150: Indirect Taxation CA Workbook

CA. Raj Kumar Practice Booklet s per Rule 8 (1) of the Customs, Central excise duties are Service Tax Drawback Rules, 1995,

No amount of drawback shall be allowed if the rate of drawback is less than 1% of the FOB value, except where the amount of drawback per shipment exceeds Rs.500.

Further, as per section 76(1) (c) of the Customs Act, 1962 drawback is not allowed where the drawback due in respect of any goods is less than Rs.50.

In the given case, since the rate of duty drawback is not less than 1% and drawback due is Rs.500 (1% of FOB value) which is more than Rs.50, duty drawback shall be allowed.

ion 76(1)(b)of the Customs Act,1962 inter alia provides that- No drawback shall be allowed in respect of any goods, the market price of which is less than the amount of

drawback due thereon . In this case, the market price of the goods is Rs.50,000,which is less than the amount of duty

drawback,i.e.2,000kgs X Rs.30=Rs.60,000.Hence ,no drawback shall be allowed.

(i) A

(ii) Sect

Question 5 (a) Assessable value of certain goods imported from USA is Rs.10, 00,000.The packet contains 10,000 pieces with maximum retail price of Rs.200 each. The goods are assessable under section 4A of the Central Excise Act, 1944, after allowing an abatement of 40%.The excise duty rate is 8% ad valorem.Calculate the amount of additional duty of customs under section 3(1) of the Customs Tariff Act, 1975 assuming basic customs duty 2@10% ad valorem. (4 Marks) (b)What are the provisions made under the customs Act, 1962 regarding control of customs over the warehoused goods? (4 Marks) (c) What are the orders that are appealable to the High Court under the customs Act, 1962?Can the delay in filing an appeal be condoned by the High Court? (4 Marks) (d) What are the provisions made under the Custom Act 1962, regarding personal hearing and order under advance ruling Answer (a)As the goods are assessable under section 4A of the Central Excise Act, 1944, additional duty of customs will be payable on the basis of maximum retail price printed on the packing; less abatement as permissible [proviso to Section 3(2) of the Customs Tariff Act]. Rs. Maximum retail price Rs.200X 10,000 20, 00, 000 Less: Abatement 40% 8, 00,000Assessable value under section 3(2) 12, 00,000Additional duty of customs @8% 96,000 Add: Education Cess @ 2% 1,920 Secondary and higher education cess @1% 960Additional customs duty payable 98,880

Section 62 of the Customs Act, 1962(b)

provides for the control of customs over the warehoused goods. The provisions are discussed as follows:- (i)All warehoused goods shall be subject to the control of the proper officer. (ii)No officer shall enter a warehouse or remove any goods there from without the permission of the proper officer. (iii)The proper officer may cause any warehouse to be locked with the lock of the customs department and no person shall remove or break such lock. (iv)The proper officer shall have access to every part of a warehouse and power to examine the goods therein. (c)

Page 151: Indirect Taxation CA Workbook

CA. Raj Kumar Practice Booklet As per section 130(1) of the Customs Act, 1962,

An appeal can be made to the High Court within 180 days from the date on which the order appealed against is received by the Commissioner of Customs or the other party.

There is no provision in the Act to condone the delay in filling an appeal. This view has been confirmed in Hongo India (2009) (SC).

The High Court may admit an appeal and memorandum of cross objection after the expiry of the period of 180 days, if it is satisfied that there was sufficient cause for not filing the same within that period.

Hence, the High Court can condone the delay in filing an appeal.

The provisions regarding personal hearing and order under advance ruling are contained in section 28-I of the Customs Act, 1962.Section 28-I inter alia provide that

If an application for advance ruling is received, the authority of advance ruling will examine the material submitted by the applicant or obtained by the authority and issue an order either allowing or rejecting the application.

However, no application shall be rejected unless an opportunity has been given to the applicant of being heard. Where an application is allowed, personal hearing can be given before the pronouncement of the advance ruling, if requested by the applicant.

Such hearing can be given to the applicant himself or to his duly authorised representative. Authority then pronounces its advance ruling within 90 days of the receipt of the application. Copy of the order, signed by members of authority and certified in the prescribed manner is sent to the

applicant and the commissioner.

PART-C

(d)

Question 6 (a)State with reasons in brief, whether service tax is payable in the following cases: (i)Services provided to Special economic Zone (SEZ) unit and (SEZ) developer, except where services are wholly consumed within SEZ. (ii)Services provided by SEZ to DTA (Domestic Tariff Area) (iii)Services provided by Indian agents undertaking marketing in India of goods of a foreign seller. (2X3=6Marks) (b)Briefly state the concept of export of taxable services under rule 3 of the Export of Services Rules, 2005 (3 Marks) (c)As certain whether the refund of service tax paid on input service can be claimed in the following case: Total Credit of Service Tax on Input service Rs.6,000 Total turnover of output service Rs.30, 000 Output service exported Rs.20, 000 (3 Marks) (d) Explain the provisions of best judgement assessment provided under section 72 Finance Act, 1994 as amended. (3 Marks) Answer (a)(i)

service tax is payable on the services provided to SEZ unit or SEZ developer irrespective of whether services are wholly consumed within SEZ or not.

However, the SEZ unit or SEZ developer can get the refund of service tax paid by them by filing a refund claim as per the prescribed procedure.

Page 152: Indirect Taxation CA Workbook

CA. Raj Kumar Practice Booklet has clarified that a special economic zone unit will be liable to

pay service tax on services provided by it to a domestic tariff area unit. (ii) Yes, CBEC&C vide circular no.105/08/2008 ST

(iii) Rule 2 of the Export of Services rules, 2005 inter alia provides that the provision of any taxable service in sub-rule (1) is treated as export of service when such service is provided from India and used outside India.

In this regard,.CBEC&C vide Circular No.111/05/2009 ST has clarified that ‘used outside India’ means ‘benefit should accrue outside India’. A service can be treated as exported even if all relevant activities take place in India. So long as the benefit of these services accrue outside India

Thus, services provided by Indian agents undertaking marketing in India of goods of a foreign seller are ‘export of service’ on the presumption that the services provided by Indian agents are exported in accordance with Export of Services Rules, 2005 and hence service tax is not payable on them.

s per Rule 3 of the Export of services Rules, 2005, the following are export of services:-

(i)When the service is provided in respect of immovable property which is situated outside India. (ii) When the service is performed outside India. (iii)When the service is provided from India but the recipient of service is outside India.

le 5 of CENVAT credit Rules, 2004 inter alia provides that the CENVAT credit in respect of the input services in providing output services which are exported shall be allowed to be utilized towards payment of service tax on taxable output services. However, where such adjustment is not possible, the refund of credit shall be allowed.

In this case the service tax liability on taxable services of Rs.10,000 (Rs.30,000- Rs.20,000) is

(b)A

(c) Ru

[email protected]%.Therefore,there is an e

(d) Secti

xcess credit of Rs.4, 970 (Rs.6, 000-Rs.1, 030) which can not be utilized. Thus, the refund of such credit can be claimed.

However, the refund will be restricted to the extent of ratio of export turnover to the total turnover for the given period, i.e.Rs.4, 000 [Rs.6, 000X (Rs.20, 000-Rs.30, 000)] on 72 provides that best judgement assessment can be done by the Central Excise Officer if;-

(i)the assessee fails to furnish the return under section 70; (ii) The assessee files the return but fails to assess service tax as per the provisions of the Finance Act, 1994 and rules made thereunder. For making best judgement assessment, the Central Excise officer may ask the assessee to produce such accounts, documents or other evidence as he may deem necessary. After taking into account all the relevant material, the Central Excise Officer shall make the assessment of the value of taxable service to the best of his judgement and determine the sum payable by, or refundable to, the assessee. He shall pass the order in writing, after giving an opportunity of being heard to the assessee.

Page 153: Indirect Taxation CA Workbook

CA. Raj Kumar Practice Booklet

MAY- 2010- NEW SYLLABUS

Part A Question 1 (a)Compute the assessable value and amount of excise duty payable under the Central Excise Act, 1944 and rules made there under from the following information: particulars No. of Units Price at Factory

Per Unit Price at depot per unit

Rate of duty ad valorem

(i) Goods transferred from factory to depot on 8 February

1,000 Rs.200 Rs.220 10% th

(ii) Goods actually sold at depot on 18 February

750 Rs.225 Rs.250 8% th

(b) M/s Royal Industries started its production activities on 15 March, 2010. In the month of march,2010,1,000 units of raw materials were purchased at Rs. 150 Per unit, paying excise duty @ 8%.800 , units of raw material were consumed in manufacturing process and finished output was sold for Rs.1,40,000 (excluding excise duty @8%.)For simplification, you may ignore the conversion cost and assume the rates of excise duty to be inclusive of education cess. Pass the journal entries in the book of the assessee and show the balance s in balance sheet as on 31.3.2010 (5 marks) (c) Mahesh Ltd., which is engaged in manufacturing of excisable goods, started its business on 1 June, 2009.It availed SSI exemption during the financial year 2009-10. The following are the details available to you. Rs

th

st

(i) 12,500 kg of inputs purchased @Rs.1,190.64 per kg (inclusive of Central Excise Duty @8.24%)

1,48,83,000

(ii) Capital goods purchased on 31.5.2009 (Inclusive of excise duty @14.42%)

80,09,400

(iii) Finished goods sold(at uniform transaction value throughout the year) 3,00,00,000 You are required to calculate the amount of excise duty payable by M/s Mahesh Ltd. in cash, if any, during the year 2009-10.Rate of duty on finished goods sold may be taken as 12.36% for the year and you may assume the selling price exclusive of Central excise duty.

Page 154: Indirect Taxation CA Workbook

CA. Raj Kumar Practice Booklet There is neither any processing loss nor any inventory of input and output. Output input ratio may be taken as 2:1 (5 marks) Answer (a)

Assessable Value= 1000 units X Rs.220=Rs.2,20,000

Calculation of Central Excise Duty

According to rule 7 of the Central Excise Valuation (Determination of Price of Excisable Goods) Rules, 2000, in cases where the goods are not sold at the factory gate, but they are transferred by the assessee to his depot,

the assessable value for the goods cleared from factory and sold from depot shall be normal transaction value of such goods at the depot at or about the same time at which the goods being valued are removed from the factory.

Basic excise duty @10% (Rs.2,20,000 X 10%) 22,000 Education cess @2% 440 Secondary and Higher education cess @1% 220Total duty payable 22,660 Note: It has been assumed that the price at depot on 8th February is the normal transaction value for sale to independent buyers. (b) Journal entries in the books of M/s.Royal Industries:-

Purchases A/C Dr 1,50,000 CENVAT Credit Receivable A/c Dr. 12,000 To Sundry Creditors A/c 1,62,000

On the date of purchase

(Purchase of 1,000 units of raw material & CENVAT credit receivable on it)

Sundry Debtors A/c Dr. 1,51,200 To Sales A/c 1,40,000 To Excise Duty payable A/c 11,200

At the time of sale

(Sold goods & excise duty payable on it)

On the payment of duty Excise Duty payable A/c Dr 11,200 To CENVAT Credit Receivable A/c 11,200

(Excise duty paid out of CENVAT credit available)

Balance sheet as on 31 March, 2010 (Relevant portion of Assets side only)

st

Current Assets , Loans and Advances Amount (Rs.)

(A) Current Assets 30,000

Page 155: Indirect Taxation CA Workbook

CA. Raj Kumar Practice Booklet Inventory of raw material

(B) Loans and Advance CENVAT Credit Receivable

800

Note: ot

(c)

her balances are not shown here

Computation of the excise duty payable by M/s Mahesh Ltd.during the finance year 2009-10:- Excise Duty on dutiable clearances:- Particulars

Amount (Rs.)

Clearances of finished goods made during the year 3,00,00,000 Less: Exemption of Rs.150 Lakh under Notification No.8/2003 dated 01-3-2003 1 ,50,00,000 Dutiable clearances 1 ,50,00,000Excise Duty @12.36% (Rs.1, 50, 00,000 X 12.36%) 18,54,000 CENVAT credit available on inputs:- (Used in the manufacture of dutiable clearance. No CENVAT credit will be available in respect of exempt clearances) % of Dutiable goods in the finished goods sold = 1,50,00,000 X 100 3,00,00,000

50%

Excise duty paid on the value of inputs consumed in manufacture of Dutiable clearances = Rs.1,48,83,00 X 8.24

108.24 X 50%

(Since output input ration is fixed )

5,66,500

CENVAT credit available on capital goods:- CENVAT credit will be available @100% of total excise duty in the current finance year 2009-10.

Excise duty payable

Particulars

Amount (Rs.)

(100% of Rs. 80.094 lakh) X 14.42 114.42 CENVAT credit can be availed on capital goods , but can be utilized only in respect of duty payable on clearances after first clearances of Rs.150 lakhs

Rs. 10,09400

Particulars Amount

Page 156: Indirect Taxation CA Workbook

CA. Raj Kumar Practice Booklet

Question 2 (a)-An assessee classified his product as per Central Excise Tariff subject to nil rate of duty. The Department contended that when the entries in the Harmonized System of nomenclature (HSN) and the central Excise Tariff are not aligned, reliance should be placed upon HSN for the purpose of classification of goods under the said tariff, Relaying upon the HSN for the purpose of classification of the impugned product, the Department classified it under another heading attracting 8% duty. Do you think that the Department’s plea is valid in law? Discuss briefly, with reference to a decided case law, if any. (5 marks) (c)An assessee made an application under section 32e of the Central Excise Act, 1944 to the settlement commission. The settlement commission was not satisfied saying that the applicant had not made a true and full disclosure of his duty liability and the manner in which same was arrived at was also not correct and rejected the application. The assessee contenteded that obligation to make truthful disclosure of duty liability would arise only after the application was admitted and not before that. Is plea taken by the assessee acceptable in law? Explain in brief, with the help of decided case law, if any. (5 marks) Answer

(a) No,

The Department’s plea is not valid in law. Central Excise Tariff is based upon HSN, but it is not a copy of HSN.

In 08) (SC), the Supreme Court ruled that the entries in HSN and the Excise Tariff are not aligned, reliance cannot be placed upon HSN for the purposes of classification of goods. Central Excise Tariff of India should be followed in such cases.

It should be appreciated that since the entries under the HSN and the entries under the said Tariff were completely different, the Department could not relied on the entries in the HSN.

(C) The applicant is not correct.

The matter of the cases is similar to the case of Customs & Central Excise Settlement Commission v. Mars Therapeutics & Chemicals Ltd. 2008 (HC)-the High Court held that the application made under section 32E of the Central Excise Act,1944 could be admitted and proceeded with only .

When the Settlement commission is satisfied that the applicant has made true and full disclosure of the duty liability and the manner in which the same was arrived at.

The High Court clarified that the onus is on the applicant to make full and true disclosure of the duty liability and the manner in which the same was arrived at. And the Settlement Commission will admit the application only when the Commission is satisfied on the true and full disclosure of the duty liability and the manner it was arrived at.

The High Court also clarified that the object behind the enactment of the provisions of Settlement Commission is the creation of a forum of self surrender and true confession and to have matter settled once for all. The Settlement Commission is not a forum to challenge the legality of the order passed under the provisions of the Act.

(Rs.)

case of Camlin Ltd. (20

Excise duty on dutiable goods 18,54,000 Less: CENVAT credit available on inputs 5,66,500 Less: CENVAT credit available on capital goods 10,09400 Excise duty payable 2,78,100

Page 157: Indirect Taxation CA Workbook

CA. Raj Kumar Practice Booklet The Settlement Commission , in the case on hand, having found that the applicant did not fully satisfy the

mandatory requirements of full and true disclosure of its true liability, can reject the application. Question 3 (a) Difference between “non-excisable goods “and “non-dutiable goods”? (3 marks)

(b) Under Excise Audit, 2000, the selection of duty for audit is based on ‘risk factors’. Explain in brief the term ‘risk factors’ giving any two examples. (3 marks)

(c) Explain the validity of the following statements with reference to the Central Excise Laws, as amended: (i) Records seized by Department during investigation but not relied upon in the show because notice should be returned within 30 days of issue of show cause notice. (ii)Special audit under section 14A and 14AA can be done by a cost accounted only. (iii) Authority of Advance Ruling under the Income tax act, 1961 will be authority for purposes of Central excise also. (iv) High Court empowered to condone delay in filling appeal and cross objection filed under section 35G and 35H of the Central Excise Act, 1944, beyond the prescribed period. (4X1=4 marks)

Answer (a) Excisable Goods Non Dutiable Goods Section 2(d)of the Central excise Act,1944 defines ‘excisable goods’ as goods specified in the First Schedule AND (to be read OR) the Second Schedule to the Central Excise tariff Act, 1985 as being subject to a duty of excise and the includes salt. Goods which are not listed in Tariff or goods which are mentioned in Tariff, but the column of rate of duty is blank are non-excisable goods, e.g. water (there is no entry in Tariff).Excise law is not applicable or non excisable goods.

Non dutiable goods are excisable goods listed in Excise Tariff .Excise law is applicable to them, but they are not liable to excise duty. Non dutiable goods may be of two types- (i)Nil duty goods i.e. Tariff rate is nil, and (ii) Exempted goods i.e.100% exemption under section 5A

(b) ’Risk factors’ under Excise Audit, 2000 means that the assessee who have a bad track record are taken up for audit on priority as opposed to those who enjoy a clean track record For example: (i) Assessee having past duty evasion cases. (ii) Late payment of duty/late filling returns (iii) Major audit objections against them (iv) No cash payment of duty (all CENVAT adjustment) (v) Past duty dues, etc Note: Any two examples may be given.

(i) he statement is valid as per rule 24A of the Central Excise Rules, 2002 . (ii) atement is not valid. As per amendment made by the finance Act, 2009, Chartered Accountants,

(c) Yes, t

No, the st In addition to a cost accountant, are also eligible for special audits under section 14A and 14AA of the Central Excise Act, 1944.

Page 158: Indirect Taxation CA Workbook

CA. Raj Kumar Practice Booklet (iii) the statement is valid as per the amendment made by the Finance Act, 2009 which has substituted the definition of authority by a new definition. [Section 23A (e)]

(iv) tatement is valid as per amendment made by the Finance Act, 2009, i.e. sub-section (2A) inserted after section 35G (2) ,thereby empowering the High Court to condone the delay in filing appeal and cross objections filed under sections 35G of the central Excise Act,1944 after the expiry of the prescribed period.

Yes,

Yes, the s

PART B Question 4 (a)Mr. Happy, a service provider, has provided services Rs. 1,00,00,000 .Out of this Rs. 70,00,000 are taxable output services and Rs 30,00,000 are exempt output services .Mr.happy has opted not to maintain separate inventory and accounts and pay prescribed amount on value of exempted output services. Service tax paid on his input services, excluding education cess and Secondary and higher education cess (EC &SAHEC) is Rs.6, 00,000 which does not include any service specified in rule 6(5) of the CENVAT Credit Rules, 2004.Rate of service tax, excluding EC and SAHEC, is 10%.Calculated the total amount payable including service tax. EC and SAHEC by Mr. Happy by GAR-7 challan. (5 marks) (b)The particulars regarding sale, purchase etc. of Shubham Udhyog for the last quarter of the year 2009-10 are as under: Rs. 1. Purchases of raw material within the State (i) taxable at 1% 40,00,000 (ii) taxable at 4% 60,00,000 (iii) taxable at 12.5% 10,00,000 2. Sale of goods manufactured from raw material purchased @4% tax rate (i) Taxable sale within the State (tax rate 4%) 20,00,000 (ii) Exempted sale within the State 10,00,000 (iii) Sale in the course of Inter – State trade or commerce (tax rate 4%) 10,00,000

3. Sale of raw material purchased @1% tax rate 44,00,000 4. Goods manufactured from the raw material purchased @12.5% tax rate

were given on lease. The deemed sale price of such goods is Rs. 12,00,000,taxable @12.5%

You may assume that input tax credit of tax on raw material used in manufacture of leased goods is available immediately. Compute the amount of Value Added Tax (VAT) payable by M/S. Shubham Udhyog for the relevant quarter. There was no opening or closing inventory. How can he utilized the balance of input tax credit available, if any? (5 marks) Answer (a) Calcu lation of service tax and total amount payable under rule 6(3) (i) of the CENVAT Credit Rules,2004:-Particulars Rs. Service tax payable on taxable services (Rs. 70, 00,000 X 10%) 7,00,000 Education cess @2% (Rs. 7, 00,000 X 2%) 14,000 Secondary and higher cess @1% (Rs. 7,00,0000 X 1%) 7,000 Amount payable on exempt services under rule 6(3)(i)(Rs. 30,00,000 X 6%)[Note (1)] 1,80,000 Total 9,01,000

Page 159: Indirect Taxation CA Workbook

CA. Raj Kumar Practice Booklet Particulars Service tax and amount

payable under rule 6(3)(i) Education cess Secondary and higher

education cess Amount payable (A) Rs,7,00,000

+Rs.1,80,000=Rs.8,80,000 Rs.14,000 Rs.7,000

Less: CENVAT credit (B)

Rs.6,00,000 Rs.6,00,000 X 2% =Rs.12,000

Rs.6,00,000 X 1% = Rs.6,000

Net amount payable = (A)-(B)

Rs.2,80,000 Rs.2,000 Rs.1,000

Amount payable by GAR-7 challan =Rs.2, 80,000 + Rs.2, 000 +1000= Rs.2, 83,000 Notes:

(b) Computation of VAT payable for the quarter ending 31 March, 2010:-

(1) Education cess & Secondary and higher education cess are not payable on ‘amount’ payable @ 6% of the value of exempt services under rule 6(3)(i) of the CENVAT Credit Rules,2004

(2) Credit of education cess and Secondary and higher education cess and Secondary and higher education cess payable on output services (or on excisable goods)respectively [First and Second proviso to rule 3(7)(b) of the CENVAT Credit Rules,2004]

Particulars Rs.

st

(A) Output tax payable

(i) On sale of taxable finished goods within the state (Rs.20, 00,000 X 4%) 80,000

(ii) On raw material (Rs. 44, 00, 000 X 1%) 44,000

(iii) On leased goods (12,00,000 * 12.5%) [deemed sale] 1,50,000

Total (A) 2,74,,000

(B) Input tax credit available

(i) On raw material purchased@1% (Rs.40, 00,000 X 1%) 40,000

(ii) On raw material purchased@1% (Rs.60, 00,000 X 4%) X 75% (Note- 1) 1,80,000

(iii) On raw material purchased @ 12.5% (Rs.10, 00,000 X 12.5%) 1,25,000

Total( B) 3,45,000

(71,000) Net VAT payable = (A)- (B)

CST payable on inter state sale adjusted (Rs.10, 00,000 X 4%) 40,000

Balance of arried forward to next quarter (Rs.40,000 – Rs.71,000) input tax credit c 31,000

Notes:

Question 5

1. If the goods manufactured from raw material are exempt from tax, no input tax credit available on such raw material. Out of total sales of Rs. 40, 00,000 of goods manufactured from raw material purchased @ 4%.Credit will not be allowed in respect of the inputs used in the manufacture of exempted goods sold. It has been assumed that the amount of the exempted sale in the total sales of goods manufactured from raw material purchased @4% assuming the input output ratio to be constant. Hence, input tax credit has been allowed to the extent of 75%.

2. If finished goods are sold in the course of inter-state trade and commerce, credit is allowed.

Page 160: Indirect Taxation CA Workbook

CA. Raj Kumar Practice Booklet (a)The term ‘business auxiliary service ’was inserted by Finance Act, 2003 which came into force on 1.7.2003. The Parliament by Finance Act, 2008 inserted an explanation in the relevant sub-clause (ii) of section 65(19) stating that for the purpose of this sub-clause. *Service in relation to promotion or marketing of service provided by the client includes any service provided in relation to marketing of games of chance, organized , conducted or promoted y the client, in whatever form or by whatever name called , whether or not conducted online, including lottery,lotto,bingo.” (5 marks) Discuss, whether the explanation appended to sub-clause (ii) of section 65 (19) is clarificatory in nature so as to be constructed having retrospective effect and retroactive operation. You may take help of a decided case law, if any

(5 marks) (b) Mr. Yes is the owner of a collection centre with facilities and trained employees for collection of human blood, urine and stool samples for biological testing .he sends the samples collected to its principal lab for actual test to be done. The assessee receives 25% of the price charged by the principal lab as commission for work of collection. The Revenue wants to charge service tax on such collection service as it amounts to promotion or marketing of services provided by its principal lab. Mr. Yes seeks your advice in this regard with reference to a decided case law, If any. Answer (a)

The Apex Court, in a case of Martin lottery Agencies Ltd (2000) (SC), ruled that by reason of an explanation, a substantive law may also be introduced.

If a substantive law is introduced, it will have no retrospective effect. Subject to the constitutionality of the Finance Act,1994 in view of the explanation appended to this, the Apex Court opined that the service tax, if any ,would be payable only with a prospective effect and not with retrospective effect.

In a case of this nature, the Court must be satisfied that the Parliament did not intend to introduce a substantive change in the law. For the aforementioned purpose, the expressions like’ for the removal of doubts’ are not conclusive. The said expressions appear to have been used under assumption that organizing games of chance would be rendition of service.

It held that the explanation is not clarifactory or declaratory in nature. Hence, it could not be constructed having retrospective effect and retroactive operation.

(b) The facts of the given case are similar to the case decided by High Court i.e. Dr. Lal Pathological Lab (P)Ltd 2007 (P&H)

The High Court held that merely because the assessee renders incidental services like putting across or dropping of the name of the principal company, it would not amount to business auxiliary services.

The High Court also observed that the activity conducted by the assessee was covered by the exception postulated by the provision of section 65(106) of the Finance Act,1994 which excluded any testing or analysis service provided in relation to human beings or animals from the scope of the taxable technical testing and analysis service.

If affirmed the decision of the Tribunal that the service provided by the assessee is incidental to testing and analysis service.

Drawing of samples and providing for testing and analysis are inter – connected services or incidental to testing and analysis.

Hence, these services are exempted from service tax. Hence, the contention of the Department is not correct.

Page 161: Indirect Taxation CA Workbook

CA. Raj Kumar Practice Booklet Question 6 (a)State with reasons in brief, whether the following services are taxable, under the provision of the Finance Act, 1994 relating to service tax: (i) Services in relation to production of alcoholic liquor on job work basis. (ii)Service of the transport of goods in container by Government Railway. (2X2=4marks) (b)Whether the value of material supplied by the contractee to the contractor for use in the execution of the works contract shall be included in the value of works contract for payment of service tax under the composition scheme? What is the present rate of service tax under this scheme? Can the service provider avail CENVAT credit also? (c)What is the general exemption available to small service providers? Who are the persons excluded from this exemption? (4 marks) (d)Explain in brief they disadvantages of composition scheme available for small dealers under VAT system. (4 marks) (e)What is VAT invoice? What are the mandatory provisions to be complied with while issuing a VAT invoice by a registered dealer? (4 marks) Ans (a)

(i)Yes, As per amendment made by the Finance Act, 2009, the definition of business auxiliary service under

section 65(19) of the Finance Act, 1994 has been amended to exclude any activity that amounts to manufacture of excisable goods.

Therefore, Services provided to manufacture of alcoholic liquor, which is not excisable, for or on behalf of the client shall attract service tax.

As per section 65(105) (zzp), any person other than Government railway is liable to service tax.

Therefore, services of transport of goods in container by Government Railway shall not service tax.

according to amended explanation to rule 3 of the Works Contract (Composition Scheme for payment of Service Tax)Rules,2007 ,

the gross amount charged for the works contractwhether supplied under any other contract for a consideration or

otherwise. Present rate of service tax under the composition scheme is ve of education cess and secondary

and higher education cess) of gross amount charged for the works contract. The assessee cannot avail CENVAT credit of inputs.

A small service provider is eligible to avail exemption from service tax o aggregate value of taxable services not exceeding Rs. 10 lakhs in any financial year

subject to the condition that during the preceding financial year, the aggregate value of all taxable services provided by him did not exceed Rs. 10 lakh.

(ii)No, transport of goods in container by rail by

(b) Yes,

shall include the value of all goods used in or in relation to the execution of the works contract,

4.12% (inclusi

(c)

Such general exemption is not applicable in following cases:- (i) The tax brand name or trade name, whether regi

(ii) If the availing the CENVAT credit. (iii) Wher

Reverse Charge]

able service provided by a person under a stered or not, of another person.

provider of taxable service ise the service receiver is liable to pay Service tax as per provisions of section 68(2) read with rule 2(1)

(d) of the Service tax Rules, 1994. [

Page 162: Indirect Taxation CA Workbook

CA. Raj Kumar Practice Booklet

Small dealers with annual gross turnover not exceeding Rs. 50 lakh who are otherwise liable to pay VAT, shall have the option for a composition scheme with payment of tax at small percentage of his gross turnover. The dealers opting for this scheme will not entitled to input tax credit.

As VAT chain is broken under the composition scheme, following are its disadvantages-

(d)

1. The purchaser shall not get any tax credit for the purposes made by him from, the dealer operating under the

composition scheme. Hence, the dealers who have desired to avail input tax credit on their purchases may not prefer to buy from composition dealers.

2. The dealer cannot avail input tax credit in respect of input tax paid by him. He will not be able to pass on the benefit of input tax credit, which will add to the cost of goods.

VAT invoice is a document listing goods sold with price, tax charged and other details as may be prescribed and issued by a dealer authorized under the Act. Mandatory provisions to be complied with: (i) Every registered dealer whose turnover of sales exceeds the specified amount shall issue to the purchaser a

serially numbered tax invoice, cash memo or bill with the prescribed particulars. (ii) The VAT invoice shall be dated and signed by the dealer or his regular employee, showing the required

particulars. (iii) The dealer shall keep a counterfoil or duplicate of such VAT invoice duty signed and dated. Exception: the dealer who opts for Composition scheme cannot issue a VAT invoice.

PART C

(e)

Question-7 (a) Miss Priya imported certain goods weighing 1,000 kgs with CIF value US$ 40,000.Exchange rate was 1 US$ -Rs. 45 on the date of presentation of bill of entry. Basic customs duty is chargeable @10% and education cess as applicable. There is no excise duty payable on these goods, if manufactured in India. As per Notification issued by the Government of India, anti-dumping duty has been imposed on these goods. The anti-dumping duty will be equal to difference between amount calculated @US $ 60 per kg and ‘landed value of goods. ’You are required to compute duty and anti-dumping duty payable by Miss Priya (5 marks) (b)M/s Marwar Industries imported finishing agents, dye – carriers, printing paste etc. to be used for manufacture of textile articles. The importer claimed exemption for additional duty of customs(CVD)leivable under section 3 of the customs Tariff Act, 1975 , on the ground that there was an exemption for excise duty in respect of said goods used in the ‘same factory’ for manufacture of textiles articles. The Department contended that CVD is payable on the ground that the goods which were to be used must also be manufactured in the ‘same factory’. You are requested to comment upon the contention of Department, with reference to a decided case law, if any (5 marks) Answer

(a) Computation of customs duty payable:- Particulars Rs. Total CIF value in INR = US $ 40,000 X Rs.45 18,00,000 Add: Landing charges @ 1% 18,000Assessable value (AV) 18,18,000 Basic customs duty (BCD)@10% 1,81,800 Education cess (EC)@2% on BCD 3,636 Secondary and higher education cess (SAHEC)@ 1% on BCD 1,818

Page 163: Indirect Taxation CA Workbook

CA. Raj Kumar Practice Booklet Landed value of imported goods 20,05,254Total customs duty payable (BCD +EC + SCHEC) 1,87,254

Computation of Anti dumping duty payable:- Particulars Rs. Value of goods in INR as per Notification =1000 kgs X US $ 60 X 45 27,00,000 Less: Landed value of goods 20,05,254Anti-dumping duty payable 6,94,746

The contention of the Department is not valid in law. The Supreme Court in a similar case that literal meaning should be availed if it leads to absurdity.

When the goods are imported, obviously, the same would not be manufactured in the same factory and thereof, it would become impossible to apply the provision of section 3(1)of the Customs Tariff Act, 1975.

It was observed countervailing duty (CVD) is that importer should not be placed at some more advantageous position vis-à-vis purchaser/manufacturer of similar goods in India.

Considering the purpose of exemption, it was held that –

‘Same factory’ means imported goods should be used in factory belonging to importer where manufacturing activity takes place.

Hence, the exemption will be available to imported goods also and CVD is not applicable.

(b) of Malwa

industry Ltd (2009) (SC) held

Question 8 (a) Explain in brief the duty exemption to baggage under section 79 (1) of the Customs Act, 1962. (3 marks) (b) Can an application be withdrawn in the following cases? If yes, state the time limit for withdrawn of such application:

(i) Application for advance ruling (ii) Application for settlement.

(c)Clearly mention the relevant date in the following cases of goods warehoused under bond:

(i)Rate of exchange, when goods are removed for home consumption. (ii)Rate of duty, when goods are removed for home consumption. (iii)Rate of duty if the goods are not removed from warehouse within the permissible period.

(4 marks) Answer (a) Sectio

(i) A

(ii) A

(b) (i) A

n 79 (1) of the Customs Act, 1961 exempt the bonafied baggage of the passengers, following baggage is passed free of duty-

rticles in use by passenger or his family or bonafied gifts or souvenirs within the limits prescribed in the Baggage Rules.

rticles for use by passenger or his family or bonafied gift or souvenirs with in the limit prescribed in the Baggage Rules.

pplication for advanced ruling can be withdrawn within 30 days from the date of application [Section 28H (4) of the Customs Act, 1962]

Page 164: Indirect Taxation CA Workbook

CA. Raj Kumar Practice Booklet pplication once made cannot be withdrawn in case of settlement [Section 127(B) (4) of the Customs Act,

1962]

The relevant date for rate of exchange is the date on which the bill of entry is presented for warehousing under section 46 of the Customs Act, 1962

and not when the bill of entry is presented under section 68 for clearance from warehouse.

As per section 15(1)(b)of the Customs Act,1962 rate of duty as prevalent on date of presentation of bill of entry for home consumption for clearance from warehouse is presented under section 68 for clearance from warehouse is applicable

and not the rate prevalent when goods were removed from customs port.

Goods which are not removed within the permissible period are deemed to be improperly removed on the day it should have been removed.

Thus, duty applicable on such date i.e. last date on which goods should have been removed is relevant and not the date on which the goods were actually removed.

NOV. 2010-NEW SYLLABUS

(ii)A

(c) (i)

(ii)

(iii)

Answer any five questions from the remaining six questions.

Question 1. (a)Determine the assessable value for purpose of excise duty under the Central Excise Act,1944 in the following cases:

(i)An assessee sells his excisable goods for 120 per piece and does not charge any duty of excise in his invoice. Subsequently, it was found that the goods were not exempted from excise duty, but were liable at 20% advalorem. (1 mark)

(ii)Certain excisable goods were sold for 120 per piece and 20% advalorem is the rate of excise duty .subsequently,it was found that that price cum duty was in fact 140 per piece as the assessee had collected 20 per piece separately. (2 marks)

(iii)The cum duty price per piece was 120 and the assessee had paid duty at 20% advalorem. Subsequently, it was found that the rate of duty was 30%advalorem and the assessee had not collected anything over and above 120 per piece. (2 marks)

Answer:- (a)

Page 165: Indirect Taxation CA Workbook

CA. Raj Kumar Practice Booklet No. Formula Assessable Value (i) Price cum duty*100/(100+ Rate of Duty) 120 *100/ (100+20) = Rs .100

(ii) Price cum duty*100/(100+ Rate of Duty) (120+20) *100/ (100+20) = Rs .116.67

(ii) Price cum duty*100/(100+ Rate of Duty) 120 *100/ (100+30) = Rs. 92.30

(b)T Ltd imported some goods from LMP Inc. of United States by air freight. You are required to compute the value for purposes of customs duty under the Customs Act,1962 from the following particulars:

CIF Value US $ 6,000

Freight paid US $ 2,000

Insurance cost US $ 700

The bank had received payment from the importer at the exchange rate of US $ 1 = 46 while the CBEC notified exchange rate on the relevant date was US $ 1 = 45.5

(Make suitable assumptions where required and provide brief explanations to your answer) (5marks)

Answer:- Computation of assessable value

CIF Value 6,000 US $Less: Freight 2,000 US $

Less: Insurance 700 US $FOB value 3,300 US $Add: Freight (20% of FOB value)- (See Note – 1)

660 US $

Add: Insurance (actual) 700 US $CIF for customs purpose 4,660 US $Add: Landing charges(1% of CIF value) (See Note – 2)

46.60 US $

Value for customs purpose 4,706.60 US $Assessable Value =Rs. 45.50X 4,706.60 US $ Exchange rate as per CBEC =Rs. 45.50 per US $ (See Note – 3)

=Rs. 2,14,150.30

Notes: 1. As the material has been imported by air, the freight has been restricted to 20% of FOB value i.e. 660 US $ [Proviso to rule 10(2) of the Customs ValuationRules,2007] 2. Landing charges at the rate of 1% of the CIF value of the imported goods shall be added ,whether ascertainable or not.

Page 166: Indirect Taxation CA Workbook

CA. Raj Kumar Practice Booklet 3. As per the explanation to section 14(1),the exchange rate shall be the rate as notified by CBEC .Therefore ,applicable exchange rate would be Rs. 45.50 per US $ as notified by CBEC.

Answer

(c)From the following particulars, arrive at the VAT liability for the month of January 2010 and also determine the amount of input tax credit to be carried forward for the next month.

(i) Input tax rate 5% and output tax rate is 15% in the State.

(ii)Inputs purchased in the month from within the State – 48,00,000

(iii) Output sold to buyers within the State during the month – 15,00,000

(iv)Output sold during the month to buyers as interstate sales – 3,00,000

(CST rate 2% against C Form)

(v)Inputs purchased from other States as interstate purchases against C Form @2% - 2,00,000(Provide suitable explanations where required with appropriate assumptions, if necessary) (5 marks)

: - Calculation of VAT liability for the month of January 10 and amount of input tax credit to be carried forward to next month.

(i) Input tax credit available on the purchase of inputs from within the State will be 5% of Rs. 48 lakh i.e Rs.2,

40,000.

(ii) No input tax credit will be available on inputs purchased from other States. (iii) VAT payable on sales within the State – 15% of Rs. 15 lakh i.e. Rs. 2,25,000

(iv) State VAT liability will be nil as the input tax credit (Rs. 2,40,000) is more than the output VAT of the

State(Rs. 2,25,000) (v) CST to be paid Rs. 6,000, i.e. 2% of inter-State sales of Rs. 3 lakh. This amount can be paid from the balance

credit of Rs. 15,000(i.e.Rs. 2.40 lakh -2.25 lakh) (vi) The amount available for carry forward to next month will be Rs. 9,000 i.e. Rs.(15000-6000) .

(d)X Bank Ltd.furnishes the following information relating to services provided and the gross amount received.

(Lakhs)

Merchant banking services 8

Assest Management (including portfolio management) 3

Service charges for services to the Government of India 1.5

Interest on overdraft and cash credits 2

Banker to the issue 5

Page 167: Indirect Taxation CA Workbook

CA. Raj Kumar Practice Booklet

Answer

Locker rent 2

Repayment of financial lease made by the customer to the bank 80 lakhs which includes a principal amount of 50 lakhs.

Compute the value of taxable service under “banking and other financial services” under the Finance Act 1994 and the service tax liability of X Bank Ltd.considering the rate of service tax at 10.3% (5 marks)

:- Calculation of value of taxable services of X Bank Ltd Particulars Rs. Amount received for merchant banking services Amount received for asset management (including portfolio management)services Service charges for services provided to Government of India (Note1) Interest received on overdraft and cash credits (Note2) Amount received for acting as banker to an issue Amount received at locker rent Amount received towards repayment of financial lease 80,00,000 Less: Principal amount included in the above installments 50,00,000Interest on financial lease 30,00,00010% of the interest amount (Rs. 30,00,000X 10%)[Note - 3]

Total Value of taxable services 21,00,000Service tax payable thereon @10% Education Cess @2% Secondary and Higher Education Cess @1%

8,00,000 3,00,000 Nil Nil 5,00,000 2,00,000 3,00,000 2,10,000 4,200 2,100

Total service tax payable including Cess 2,16,300

Notes:

1. Service charges for banking and other financial services provided to Government of India are exempt from service tax. (Notification No. 13/2004 ST)

2. Interest received on cash credit and overdraft is exempt from service tax (Notification No. 29/2004 ST)

3. Service tax is payable only on 10% of the interest on financial lease. The interest is the difference between the installment paid towards repayment of the lease amount and the principal amount contained in such installment(Notification No. 4/2006 ST )

Question 2. (a)XYZ Co.engaged in the service of over burden removal in mines had acquired some tipper trucks and taken CENVAT credit to the extent of the excise duty paid on the said trucks acquired by them. Subsequently, on becoming aware that the said trucks do not qualify as capital goods or inputs under the CENVAT Credit Rules, 2004, the said XYZ Co. reversed the credit taken, all of which had remained unutilized. The Department has issued a show cause notice that interest under section 11AB on the amount of credit taken, but not utilised should be paid. Write a note whether the action of the Department is correct in law. (2marks)

Page 168: Indirect Taxation CA Workbook

CA. Raj Kumar Practice Booklet Answer: Yes, the action of the Department is correct in law. Rule 14 of the CENVAT Credit Rules,2004 provides for recovery of CENVAT credit wrongly taken or utilized wrongly or erroneously refunded .Earlier, there was a view that if an assessee had taken the credit but not utilised the same for payment of taxes or duty, then no interest payment is required. The Central Board of Excise and Customs has clarified that where credit is taken or wrongly utilised or erroneously refunded, the same along with interest should be recovered from the manufacturer or provider of output service. Hence, in this case, XYZ Co. having taken the credit wrongly will have to pay interest under section 11AB even if the credit has been utilised.

(b)’Pure & Lovely’ are manufactures of ‘hair oil’ falling under Chapter 33 of the Central Excise Tariff (CET).Disputes arose in respect of two other categories ‘edible oil’ or ’coconut oil’. The Department contends that the coconut oil falling under these two categories are meant for sale as hair oil and should be classified under Chapter 33 of the CET. The manufacturers contended that they are not printing the specific use of such oil as ‘hair oil’ and this should be classified as vegetable oil under Chapter 15 of the CET irrespective of its use by the consumer. Chapter Note 2 of Chapter 33 prescribes a condition that Heading No. 3305 (which covers hair oil) applies to products put up in packing of a kind sold by retail for such use. Section Note 2 to section VI provides that goods falling in Heading 3305 by reason of being put up for retail sales are to be classified in the said Heading. Briefly discuss whether coconut oil packed in larger packs generally used by consumers for edible purposes would merit classification under Chapter 15. (4 marks)

Answer:- Yes, the coconut oil packed in larger packs generally used by consumers for edible purposes would merit classification under Chapter 15.

The dispute in this case with the Departmnet is restricted to ‘edible oil’ or ‘coconut oil’. The assessee ‘Pure&Lovely’pays excise duty under Chapter 33 in respect of all packages that are marked for use as ‘hair oil’.

The Section Note 2 to Section VI supports the interpretation that although a product is capable of being classified under more than one heading, owing to the nature of its retail packing which is indicative of its use as hair oil. The classification under heading 3305 would get priority.

What follows is that if the same coconut oil is packed in larger packs of say 1 liter or 2 litre generally used by consumers for edible purposes (even though some customers may use it as hair oil).

It would be classified under Chapter 15. Hence, the classification of coconut oil would depend upon the fact as to how the majority of the customers use the product.

Equally, if the oil is put in packages generally meant for sale as ‘hair oil’, then it would be classified under Chapter 3305 even though some consumers may use it as edible oil.

Answer:

(c)Sona Ltd., purchased a lathe machine at a cum-duty price of 18,63,680.The excise duty rate charged on the said machine was 16% plus education cess 2% plus secondary and higher education cess 1%.The machine was purchased on 1-7-2007 and was disposed of on 30-09-2009 for a price of 10,00,000 in working condition as second hand machine. Calculate the amount of CENVAT credit allowable for the financial years 2007-08 and 2008-09 and also specify the amount payable towards CENVAT credit already taken at the time of disposal of the machinery in the year 2009-10. (5 marks)

- Amount of CENVAT credit allowable for financial years 2007-08 and 2008-09:- particulars Amount (in Rs.)

Page 169: Indirect Taxation CA Workbook

CA. Raj Kumar Practice Booklet Cum duty price of the machinery 18,63,680 Rate of duty including education cess and Secondary and higher education cess

16.48%

Duty paid on machinery = Rs. 18,63,680 X 16.48 116.48

2,63,680

CENVAT credit allowable during the Financial Year 2007-08@50% Rs.1,31,840 Financial Year 2008-09@50% Rs.1,31,840

Amount payable towards CENVAT credit on disposal of machinery during the year 2009-10 Particulars Amount (in Rs.)Total CENVAT credit availed on the machinery Less: Amount of reduction from the said credit on (i)First 50% = [1,31,840x2.5%] x 9 quarters (i)Next 50% = [1,31,840x2.5%] x 6 quarters

2,63,680

Rs. 29,664

Rs. 19,776Amount payable on disposal of machinery Rs. 2,14,240Notes: 1. In respect of capital goods CENVAT credit shall be taken only for an amount not exceeding 50% of duty in the same financial year and balance may be taken in any subsequent year [Rule 4(2)(a)of the CENVAT Credit Rules ,2004]

2. If capital goods are removed after being used, the manufacturer shall pay an amount equal to CENVAT credit taken on the said capital goods reduced by 2.5% of each quarter of a year or part thereof from the date of taking the CENVAT credit [Second Proviso to rule 3(5)of the CENVAT Credit Rules ,2004]

3. It is assumed that first 50% of the CENVAT credit has been taken on 01.07.2007 and the balance 50% of the credit has been taken on 01.04.2008.

Answer

(d)Y Ltd (raw material supplier) supplies raw material to a job worker (manufacturers) Z Ltd. The job worker after completion of the job delivers 5000 packets of finished goods to Y Ltd . In all these packets retail sale price (maximum)of 30 per packet is marked. The product in packaged form is subject to excise duty on the basis of MRP under section 4A of the Central Excise Act, 1994 and in respect of the same, 30% abatement has been prescribed. Determine the assessable value for purpose of excise duty after considering the following particulars:

Cost of raw material supplied 45,000

Job charges including profit 15,000

Transport charges for dispatch to job worker 4,000

Transport charges for return of finished product 4,000

(Provide brief explanation and make assumptions where required) (5 marks)

:-

Section 4A of the Central Excise Act, 1994 provides for valuation of excisable goods with reference to retail sale price.

Page 170: Indirect Taxation CA Workbook

CA. Raj Kumar Practice Booklet AS PER SEC 4A(2); notwithstanding anything contained in section 4 , such value shall be deemed to be the

retail sale price declared on such goods less such amount of abatement, If any, from such retail sale price as the Central Government may allow by notification in the Official Gazette.

Thus, the provisions of section 4A of the Central Excise Act,1994 are overriding provisions,i.e.,if a product is covered under provisions of section 4A ,valuation will be done on the basis of section 4A and not on the basis of material cost plus job charges.

Hence, in this case, the assessable value of the product will be computed as follows:

Particulars Amount in Rs.Retails sale price per packet

30

Less: Abatement of 30% = Rs. [30x 30/100] 9Assessable value of each packet 21Assessable value for the purpose of excise duty =Rs. 21 x 5,000 packets

1,05,000

NOTE-The information pertaining to raw material cost, Job charges, profit, transport charges etc. need not to be considered.

Question 3 (a)Write a brief note on whether input tax credit of the VAT paid on purchases of goods that are stock transferred is available (2 marks)

Answer:-

Inter State transfers do not involve sale and are therefore are not subjected to sales tax.The same position continues under VAT.

However, tax paid on the purchases of goods which are stock transferred will be available as input credit after retention at 2% of such tax by the State Government. Ie (VAT RATE –CST RATE; 2%)

Answer:

(b) X Ltd. is having a manufacturing unit at Faridabad. In the financial year 2008-09,the value of total clearances from the unit was 850 lakhs as per the following details:

(i) Exports to USA Rs. 100 lakhs to Nepal : 50 lakhs

(ii) Clearances to a 100% export oriented unit : 75lakhs

(iii) Clearances as loan licensee of goods carrying the brand name of another upon full payment of duty : 200 lakhs

(iv) Clearance exempted vide Notification No.214/86-C.E. dated 25.3.86 : 125 lakhs.

(v) Balance clearances of goods in the normal course: 300 lakhs

You are required to state with reasons whether the unit is entitled to the benefit of exemption under Notification No. 8/2003 – C.E.dated 1.3.2003 as amended for the financial year 2009-10 (4 marks)

- As per Notification No. 8/2003 dated 01.03.2003 in order to claim exemption as small scale industry in the financial year 2009-10,the total eligible clearances of the manufacturing unit should not exceed Rs. 400 lakh in the previous financial year 2008-09.

For the purpose of computing the eligible clearances:-

Page 171: Indirect Taxation CA Workbook

CA. Raj Kumar Practice Booklet (i) Export turnover of Rs. 100 lakh to USA would be excluded, while the export to Nepal of Rs. 50 lakh would be

included.

(ii) Clearances of Rs. 75 lakh without payment of duty to a 100% EOU would be excluded.

(iii) Clearances of Rs 200 lakh as a loan licensee of goods carrying the brand name of another on payment of duty would be excluded.

(iv) Clearances of Rs 125 lakh exempt under Notification No. 214/86 would be excluded.

(v) Normal clearances of excisable goods of Rs. 300 lakh would be considered.

Thus, the eligible turnover for exemption is as follows: Rs300+50=Rs. 350 lakh

Since the eligible clearances do not exceed Rs 400 lakh in the financial year 2008-09, the manufacturing unit will be entitled for SSI exemption in financial year 2009-10.

Answer

(c)Briefly explain the documents required for filing claim of rebate of duty on export of goods under rule 18 of the Central Excise Rules, 2002 (5 marks)

:- The following documents are required for filing claim of rebate:

(i) A request on the letterhead of the exporter containing claim of rebate, A.R.E.1 numbers and dates, corresponding invoice numbers and dates on each A.R.E.1 and its calculations.

(ii) Invoice issued under rule11. (iii) Self attested copy of shipping bill. (iv) Self attested copy of Bill of Lading. (v) Original copy of the A.R.E.1

Answer

(d)An assessee had cleared goods without payment of excise duty. However, on coming to know from his sources that a show cause notice demanding duty on such clearances is likely to be issued but before actual issue of such notice the assessee made full payment of excise duty with interest. A show cause notice was issued subsequently as to why the assessee should not be subject to mandatory penalty equal to the excise duty sought to be evaded under section 11AC of the Central Excise Act, 1994.Briefly examine, with a note, the conditions for levy of penalty under section 11AC and state whether there is any discretion to reduce such penalty. (5 marks)

:- Facts- Assessee has cleared goods without payment of duty, and ON COMING TO KNOW FROM HIS SOURCES THAT A SHOW CAUSE NOTICE DEMANDING DUTY ON SUCH CLEARANCES IS LIKELY TO BE ISSUED, the assessee made full payment of excise duty with interest. Issue- whether he is liable to pay mandatory penalty u/ s 11 AC Legal Position-

The conditions and the circumstances that would attract the imposition of penalty under section 11AC of the Central Excise Act, 1944 have been laid down by the Apex Court in case of Rajasthan Spinning and Weaving Mills 2009 -(SC)

Page 172: Indirect Taxation CA Workbook

CA. Raj Kumar Practice Booklet Supreme Court clarified that both section 11AC as well as proviso to sub-section (1)of section 11A use the

same expressions :”…………by reasons of fraud, collusion or any wilful mis-statement or suppression of facts, or contravention of any of the provisions of this Act or of the rules made thereunder with intent to evade payment of duty ……”

Hence, it drew the inference that the penalty provision of section 11AC would come into play only if the notice under section 11A (1)states that the escaped duty was the result of any conscious and deliberate wrong doing and in the order passed under section 11A (2),there is a legally tenable finding to that effect.

The Apex Court elaborated that the payment of the differential duty, whether before/after the show cause notice is issued, cannot alter the liability for penalty. The conditions for penalty to be imposed are clearly spelt out in section 11AC of the Act.

Further, Central Board of Excise and Customs vide Circular No. 889/09/2009 CX dated 21.05.2009 has clarified that when the conditions spelled out under section 11AC are fulfilled, there is no discretion to reduce the mandatory penalty which is equal to duty even though the duty is paid before the issuance of show cause notice.

Conclusion- in the light of what is stated above, assessee is liable to pay mandatory penalty u/ s 11AC even if he has paid differential dues before isssue of show cause notice.

Question 4. (a)D & Co. is engaged in the services of site preparation and clearance, excavation, earth moving and demolition services. The gross amount received during the quarter ended 30.06.2009 for the services provided by them are given below:Core extraction services for construction 1,80,000 Land reclamation work 80,000 Services in relation to agriculture 2,00,000 Renovating or restoring water sources 3,50,000 Horizontal drilling of passage of cables or drain pipes 1,00,000 Soil stabilization 90,000 Construction of transport terminals 55,000 Calculate the value of taxable services under ‘Site preparation and clearance, excavation, earth moving and demolition services ’under the Finance Act,1994 and the service tax payable at 10.3% (Provide brief explanations where required) (4 marks) Answer:- Computation of value of taxable service Gross amount received Rs.

For core extraction services fro construction 1,80,000

For land reclamation work 80,000

For horizontal drilling for passage of cables or drain pipes 1,00,000

For soil stabilization 90,000

Total value of taxable services 4,50,000

Service tax payable on above @10% 45,000

Education cess @2% 900

Secondary and Higher education cess @1% 450

Total service tax payable 46,350

Page 173: Indirect Taxation CA Workbook

CA. Raj Kumar Practice Booklet

Notes:

1. Amount received for site formation and clearance, excavation and earth moving and demolition services provided in relation to agriculture and for renovating or restoring water sources are specifically excluded from the definition of ‘site formation’ and clearance ,excavation and earth moving and demolition”

2. Amount received for such services provided in the course of construction of transport terminals is exempt from the whole of service tax .

Answer

(b)State with reference to the provisions of the Finance Act, 1994 whether service tax is leviable in the following cases:

(i) Taxable services rendered in the Special Economical Zone and consumed therein.

(ii)Services provided in the Continental Shelf of India and Exclusive Economic Zone of India. (2x2 = 4marks)

:- (i)

Notification No. 15/2009 –ST provides- unconditional exemption from service tax to services consumed wholly within the SEZ.

Developer or unit of SEZ has to maintain proper accounts of receipt and utilisation of taxable services for which exemption is claimed.

It further provides that exemption can be claimed by way of refund of service tax paid on services consumed partially or wholly outside the SEZ.

(ii) Notification No. 14/2010 –ST dated 27-2-2010 provides that-

Construction and operation of installations, structures and vessels for purpose of prospecting or extraction/production of minerals oils and natural gas in the whole of exclusive economic zone and continental shelf of India,

any service provided or to be provided by or to such installations, structures and vessels and for supply of any goods connected with these activities would be within the purview of service tax under the Finance Act,1994

Answer:

(c)Write a brief note with reference to the Service Tax Rules, 1994 in the following cases:

(i)Advance payment of service tax

(ii)Time limit for preservation of records (4marks)

-(i) As per the provisions of rule 6(1A)of Service Tax Rules,1994 , a facility has been provided to the assesses to make advance payment of service tax on his own and adjust the amount so paid against the service tax which he is liable to pay for the subsequent period.

Provided that the assessee shall,-

Page 174: Indirect Taxation CA Workbook

CA. Raj Kumar Practice Booklet Intimate the details of the amount of service tax paid in advance, to the Superintendent within a period of 15

days from the date of such payment; and

Indicate the details of the advance payment made, and its adjustment, if any in the subsequent return to be filed

under section 70 of the Act.”

(ii) As per the provisions of rule 5(3) of the Service Tax Rules, 1994,

All records maintained by the assessee shall be preserved at least for a period of five years immediately after

the financial year to which such records pertain.

For the purposes of this rule, “registered premises “include all premises or offices from where an assessee is

providing taxable services.

Answer

(d) S Ltd provides management consultancy services that are subject to service tax under the Finance Act, 1994. In respect of the services rendered during the month of December, 2009, the service tax of 15 lakh was remitted to Government treasury on January 20, 2010.Arrive at the penalty under section 76 of the Finance Act, 1994 which is leviable in this case. (4 marks)

:- Section 76 of the Finance Act,1994 provides for penalty for failure to pay service tax, as under:

(i) Rs. 200 per day during which such failure continues

Or

(ii)Amount computed at the rate of 2% of such tax, per month.

Whichever is higher?

It is computed from the first day after the due date till the date of actual payment. Total amount of penalty under this section shall not be exceed the service tax payable.

Amount of penalty will be the higher of the following:

(i)15 days x 200 = Rs. 3,000 (Since the period of delay is 15 days)

(ii)2% of Rs. 15 lakh for 15 days

= Rs. 2/100 x 15, 00,000x15/31

=Rs. 14,516.13 and the amount of service tax ie Rs. 15 lakh (whichever is lower)

Thus, X & Co. will have to pay the penalty of Rs. 14,516.13

Question 5(a) Write a brief note on the deficiencies of the VAT system. (4 marks)

Answer:-

1) The other feature is that central sales tax (which is an origin based tax) has been allowed to coexist with VAT (which is essentially destination based tax) in a system where central sales tax payments are not eligible for set off in the receiving State.

Page 175: Indirect Taxation CA Workbook

CA. Raj Kumar Practice Booklet 2) One of the key deficiencies in the current system is that there is lack of uniformity in rates of VAT across the

states. Distortions also arise on account of differences in classification of the product and concessions or exemptions like composition schemes, exempted category of goods in certain States.

3) There is a burden of record keeping for small traders even though they could switch over to a simplified composition scheme without the benefit of input credit. This could add to the supply chain cost because of the non availability of input credit when the goods pass through a small trader under the composition scheme.

4) Another possible weak point in the introduction of VAT, which will have an adverse impact is that ,since the tax is to be imposed or paid at various stages and not on last stage, it would increase the working capital requirements and the interest burden on the same .In this way, it is considered to be non-beneficial as compared to the single stage-last point taxation system.

5) It is claimed that the tax is regressive i.e,. its burden falls disproportionately on the poor since the poor are likely to spend more of their income than the relatively rich person. There is merit in this argument, particularly if its attempts to replace direct or indirect taxes with steep, progressive rates.

Answer:

(b)A private agency has built a canal system on built-own-operate-transfer basis under a contract with the State Government. User charges at commercial rates are leived by the private agency and the revenue so generated is utilized for servicing and repayment of the capital investment.

Briefly write a note with reference to ‘commercial or industrial construction service’ under the Finance Act,1994 Whether the user charges collected by the private agency is liable to service tax. (4 marks)

- Circular No.116/09/2009 ST dated 15.09.2009 clarifies that as per section 65 of the Finance Act,1994,.

”Commercial or industrial construction service “is chargeable to service tax if it is used, occupied or engaged either wholly or primarily for the furtherance of commerce or industry.

As the canal system built by the Government or under Government projects is not falling under commercial activity, the canal system built by the Government will not be chargeable to service tax.

However, ,if the canal system is built by the private agencies an is developed as a revenue generating measure, then such construction should be charged to service tax.

In this case the canal system is built by a private agency and user charges are collected at commercial rates. Hence, the user charges collected by the private agency, in the given cases will be subject to service tax.

Answer

(c)Briefly explain the provisions under the Service Tax Rules, 1994 regarding late fee for delayed filing of returns. (4 marks)

:- Late fee for delayed return [Rule 7C of the Service Tax Rules,1994]

The prescribed late fee for furnishing a delayed return is given in the following table:

Period of delay Late fee 15 days from the date prescribed for submission of the return Rs. 500

Page 176: Indirect Taxation CA Workbook

CA. Raj Kumar Practice Booklet Beyond 15 days but not later than 30 days from the date prescribed for submission of the return

Rs. 1,000

Beyond 30 days from the date prescribed for submission of the return

An amount of Rs. 1,000 plus Rs. 100 for every day from the 31st day till the date of furnishing the said return

Maximum late fees

Total late fee for delayed submission of return shall not be exceeding Rs. 2,000.

Further, where the assessee has paid the prescribed late fee for delayed submission of return, the proceedings, if any, in respect of such delayed submission of return shall be deemed to be concluded.

Late fee may be reduced/ waived-

Where the gross amount of service tax payable is nil,

the Central Excise Officer may , on being satisfied that there is sufficient reason for not filling the return, reduce or waive the penalty (late fee)[Proviso to rule 7C]

Answer:

(d)State briefly with reference to section 66A of the Finance Act,1994 who is liable to pay service tax for services provided from outside India. (4 marks)

- Section 66A of the Finance Act,1994 provides a separate mechanism for levy of service tax on services received from outside India. It has to be read with the Taxation of Services (Provided from Outside India and Received in India) Rules, 2006

Section 66A provides that where any services specified under section 65(105) are rendered by a person who are established a business or has a fixed establishment from which the service is provided or has a permanent address or usual place of residence in a country other than India and such services are received by a person (recipient) who has his place of business, fixed establishment, permanent address or usual place of residence in India, then such service shall be a taxable service.

Such service shall be treated as if the recipient had himself provided the service in India.

However, the provisions of this section would not apply to a service received by an individual for a purpose other than for use in business or commerce. A person carrying on a business through a branch or agency in a country shall be treated as having a business establishment in that country.

Question 6. Answer any four of the following:

(a)Discuss with a brief note the provisions of section 129E of the Customs Act,1962 regarding deposit of duty and interest or penalty leived pending appeal. (4 marks)

Answer:- As per section 129E where, in any appeal ,

the decision or order appealed against relates to any duty and interest demanded in respect of goods which are not under the control of customs authorities or any penalty leived under this Act, the person desirous of appealing against such decision or order shall ,pending the appeal ,first deposit the duty and interest demanded or penalty leived with the proper officer.

However, such deposit may be waived by the Commissioner (Appeals)or the Appellate Tribunal if he or it is of the opinion that deposit of duty and interest demanded or penalty leived would cause undue hardship to such

Page 177: Indirect Taxation CA Workbook

CA. Raj Kumar Practice Booklet person. Such waiver may be allowed subject to conditions as he or it may deem fit to impose so as to safeguard the interest of revenue.

Answer

(b)Write a brief note on sanctioning of refund of customs duty under section 27(2) of the Customs Act, 1962 under which refunds will not be credited to the consumer welfare fund. (4 marks)

:- In view of the provisions of unjust enrichment in the Customs Act,1962, the amount found refundable has to be transferred to the Consumer Welfare Fund.

However, the amount of duty and interest found refundable instead of being credited to the Fund, is to be paid to the applicant if the amount is relatable to-

The duty and interest, if any on such duty paid by the importer, or the exporter as the case may be if he had not passed on the incidence of such duty and interest if any, paid on such duty to any other person.

The duty and interest, if any, on such duty on imports made by an individual for his personal use.

The duty and interest, if any, paid on such duty borne by the buyer, if he had not passed on the incidence of such duty and interest, if any ,paid on such duty to any other person.

the export duty as specified in section 26

Drawback of duty payable under sections 74 and 75.

The duty and interest, if any, paid on such duty borne by any other such class of applicants as the Central Government may, by notification in the Official Gazette, specify.

Answer

(c)Write a brief note with specific reference to rule 1 of the Rules of interpretation of the First Schedule to Customs Tariff Act, 1975 (4 marks)

:- Rule 1 of the general rules for interpretation states that- The titles of sections, chapters and sub-chapters in the First Schedule to the Customs Tariff Act, 1975 are

provided for ease of reference only. For legal purposes, classification shall be determined according to the terms of the headings and any relative

section or chapter notes and provided such headings or chapter notes do not otherwise require, according to the subsequent rules.

Answer

(d)Certain goods were brought to the export shed on 5.10.2009.The goods were examined and ‘let export order’ was issued on the same day by noting of the shipping bill. Computer processed shipping bill was issued on 6.10.2009.DEPB rate was lowered on 6.10.2009 and the Department allowed DEPB at the rate prevailing on 6.10.2009.The goods were permitted for clearance and loading on 5.10.2009.It is the assessee’s case that under the Customs Act,1962, they are entitled for the higher rate of DEPB prevailing on 5.10.2009.Write a brief note whether the assessee’s stand is correct in law. (4 marks)

:- In this case, the goods were brought to export shed on 5.10.09 and were examined on the same day when the noting of the shipping bill was also completed. The goods were permitted for clearance and loading on the same day

Section 16(1)(a)of the Customs Act,1962 deals with date for determination of rate of duty and tariff valuation of export goods.

Page 178: Indirect Taxation CA Workbook

CA. Raj Kumar Practice Booklet It states that in the case of goods entered for export under section 50,the rate of duty and tariff valuation, if

any, applicable to any export goods ,shall be the rate and valuation in force on the date on which the proper officer makes an order permitting clearance and loading of the goods for exportation under section 51.

Section 50 deals with entry of goods for exportation by presenting a shipping bill and section 51 deals with the permission to clear and load the goods.

Since, in the given case, both the events have been taken place on 5.10.09, the rate of duty for DEPB purposes shall be the rate, prevailing on 5.10.09 which is the higher rate. The assessee’s stand in this case is correct.

Answer:

(e) Briefly explain the provision relating to transshipment of goods without payment of duty under section 54 of the custom act, 1962.

- Provisions relating to transhipment of goods without payment of duty under section 54 of the Customs Act, 1962 are as follows:-

(1) Where any goods imported into a customs are intended for transhipment, a bill of transhipment shall be presented to the proper officer in the prescribed form. Where the goods are being transshipped under an international treaty or bilateral agreement between the government of India and Government of a foreign country, a declaration for transhipment instead of a bill of transhipment shall be presented to the proper officer in prescribed form.

(2)Subject to the provisions of section 11, where any goods imported into a customs station are mentioned in the import manifest or the import report, as the case may be as for transhipment to any place outside India, such goods may be allowed to be so transhipped without payment of duty.

(3)Where any goods imported into a customs station are mentioned in the import manifest or import report, as the case may be, as for transhipment

to any major port as defined in the Indian Ports Act,1908 (15 of 1908), or the customs airport at Mumbai,Calcutta,Delhi or Chennai or any other customs port or customs airport which the Board may, by notification in the Official Gazette specify in this behalf, or

to any other customs station and the proper officer is satisfied that the goods are bonafied intended for transhipment to such customs station.

The proper officer may allow the goods to be transhipped, without payment of duty, subject to such conditions as may be prescribed for the due arrival of such goods at the customs station to which transhipment is allowed.

Question 7(a) UPT Ltd,are manufactures of electrical transformers, switch gears falling under Chapter Heading 85042200 and Chapter Heading 85352121 of Central Excise Tariff. The manufacturers cleared excisable goods valued at 30 lakhs against Served from India Scheme Certificate (SFIS) at nil rate of duty availing the exemption by way of debit of SFIS under Notification No.34/2006-CE dated 14.6.2006.As separate accounts for purpose of sub-rule (3)of rule 6 of the Cenvat Credit Rules,2004 were not maintained by the assessee (M/s UPT Ltd.), they were required to pay 5% of the value of the exempted goods of 1.5 lakhs (30 lakhs @5%)on the basis that the goods cleared under SFIS scheme are to be treated as exempted goods. Discuss with reference to the definition of exempted goods under rule 2(d) of the Cenvat Credit Rules, whether the stand taken by the Department is correct. (Assume that debit to SFIS is a method of payment of appropriate duty) (4 marks)

Answer:- (a) Rule 3 ( 6) of the CENVAT Credit Rules,2004 provides that a manufacturer of goods opting not to maintain separate accounts shall have the option.

(i) to pay an amount equal to five percent of the value of the exempted goods or

Page 179: Indirect Taxation CA Workbook

CA. Raj Kumar Practice Booklet (ii)pay an amount equivalent to the CENVAT credit attributable to inputs used in or in relation to manufacture of exempted goods subject to the procedure in sub-rule 3A.

In this case, the Department has demanded the amount under the first option i.e. 5% of the value of the exempted goods. The demand raised by the Department is questionable. The issue which requires consideration is as to whether the goods cleared in this case against debit entries in the SFIS certificate could be treated as ‘exempted goods’

Rule 2(d)of the said rules defines exempted goods as excisable goods which are exempted from the whole of the duty of excise leviable thereon and includes goods which are chargeable to NIL rate of duty.

Considering the facts of the case, it may be inferred that both the said goods are not exempted goods.

Firstly, the transformers and switchgears are not exempt from the duty leviable because the relevant debit entries made in the SFIS are towards appropriate duty levied on the said goods.

Secondly, the goods in question are not eligible for NIL rate of duty and the debit entry in the SFISD certificate is in discharge of the appropriate duty liability on the said goods.

The aforesaid view has also been supported by the Tribunal in case of Universal Power Transformer Pvt Ltd. Banglore 2010 (Tri.-Bang.),

Wherein the Tribunal held that the duty liability debited in SFIS scrip amounts to discharge of liability and not amounts to exemption.

Thus, the stand taken by the Department to classify the said goods as ‘exempted goods’and demand 5% of the value of the same is not correct in law.

Answer

(b)Briefly state the provisions of the Central Excise Rules, 2002 relating to electronic payment of excise duty and electronic filing returns. (4 marks)

:-

With effect from 1.4.2010,CBEC has reduced the quantum of excise duty for electronic payment of duty from Rs.50 lakh to Rs.10 lakh .

It has amended the third proviso to rule 8(1) of the Central Excise Rules ,2002 to provide that where an assessee, who has paid duty of ten lakh rupees or more, including the amount of duty paid by utilization of CENVAT credit, in the preceding financial year

shall thereafter, deposit the duty electronically through internet banking.

Further, it has inserted a proviso after second proviso to rule 12(1) of the aforesaid rules to provide that

Where an assessee has paid total duty of rupees ten lakh or more including the amount of duty paid by utilization of CENVAT credit in the preceding financial year,

he shall file the monthly or quarterly return,as the case may be electronically.

(c)Hand blender manufactured by the assessee was cleared as gift along with each unit of juicer, mixer and grinder

(JMG).The MRP of 750 through printed on the ‘hand blender box’ contains clear indication that it is supplied free of

cost. The package of JMG also shows this. The hand blender is not sold separately. The Department has issued a show

Page 180: Indirect Taxation CA Workbook

CA. Raj Kumar Practice Booklet

Answer

cause notice that duty should be paid on the hand blender. Write a brief note with reference to section 4A of the Central

Excise Act,1994 whether the notice is substainable in law.(4 marks)

:- No, the notice issued by the department is not sustainable in law.

In this case, though the MRP of Rs. 750 is printed on the “hand blander box ‘there is a clear indication that it is supplied free of cost. This is merely a sale promotion measures in order to augment the demand

In Sony India Ltd. -2004-SC , Supreme Court held that-

The offer of gifts was only incidental benefit and not the part of consideration to be paid in regard to television sets as such.

It is also clear that hand blander would be given free with every purchase of JMG and there is no sale of the same.

Hence, it can be concluded that the duty under section 4A will have to be paid on the JMG and the hand blender provided as a gift will not be again subject to duty on the MRP because there is no sale of hand blender.

(d)State briefly with reference to the provisions of the Finance Act, 1994 whether the following services are liable to tax.

(i)Permanent transfer of intellectual property rights.

(ii)Services provided in relation to handling, storage and warehousing of empty containers under storage and warehousing service. (4 marks)

Answer:- The term ‘intellectual Property Service’, has been defined in section 65(55b)of the Finance Act,1994 as:-

(i)transferring temporarily; or

(ii)permitting the use or enjoyment of

An intellectual property right.

Hence, permanent transfer of intellectual property right does not attract service tax.

(ii)Master Circular No. 96/7/2007 ST dated 23.08.2007 has clarified that empty containers are covered within the meaning of ‘goods’ as per section 65(50).Thus, services provided in relation to storage and warehousing of empty containers is liable to service tax under ‘storage and warehousing service.’

Page 181: Indirect Taxation CA Workbook

CA. Raj Kumar Practice Booklet

SALIENT FEATURES OF "EXCISE AUDIT 2000"

INTRODUCTION

The economic liberalization initiated in 1991 by the Government carried a special focus on Tax Reform as itwas felt that complexities in levy and collection of tax could fetter economic activity.

The Tax Reforms Committee headed by Prof. Raja J. Chellaiah examined the Tax System and Administrationin the country.

The Committee concluded that a simple, credible and progressive system was necessary to improve Taxcompliance. The need for better relations between the Tax Collectors and Taxpayer was also emphasised. Theneed to achieve both equity and efficiency in Taxation was thought to be important.

Different steps in auditing are -

Preparatory or preliminary review: In this phase the audit party gathers as much relevant information aspossible about the assessee and its operations in the office. The details to be gathered are, reason for selection,result of last audit, profile of the assessee giving details of ownership, goods manufactured etc. private records,special procedures, revenue realised, details of anti-evasion action if any, income tax returns, sales tax returns,annual reports, Cost audit reports, changes in law during the audit period, classification and price declaration,etc.

Gathering and documenting the systems information: This phase starts as soon as the party reaches the unit. Theparty gathers and documents systems information by interviewing key personnel and tries to understand theorganisational pattern, obtains annual reports and reports submitted to other departments/institutions, studies taxaccounting systems. The auditors also find out about trading activity, captive consumption, exports, purchaseprocedure and policy, imports, job work, ISO reports and other revenues.

Page 182: Indirect Taxation CA Workbook

CA. Raj Kumar Practice Booklet Tour of the premises/plant: This is a very important element of the audit programme. The audit party gets anopportunity to physically verify many facts as understood during the first two phases and clarify doubts in thisphase. The tour should cover all areas and should be thorough.

Evaluation of internal controls: An understanding of the organisational chart of the unit, review of accounts andgeneral ledger and a general systems review enables the auditor to understand and study the impact of varioussubsystems that have an impact on indirect tax revenue. The evaluation of internal controls involves study oftax accounting, revenue accounting and expenditure accounting. ‘Walkthrough’ method can be a very effectivetool in evaluation of internal control

Risk loss analysis (Reasonableness test): Based on the studies made in the earlier steps, the audit party has toanalyse and work out what would be the risk to revenue because of deficiencies of accounting system and whatwould be reasonable. The analysis will be used for development of Audit plan.

Trends analysis: Study of trends in revenue, modvat availment, profit, prices and quantum of the itemsmanufactured over a period will help the audit party in identifying areas which are to be looked into.

Development of Audit plan: After completing all the steps detailed above, the auditors develop an audit plan ina narrative or list format which will be consistent with the complexity of the audit.

Verification in accordance with the plan: The Audit Programme has detailed guidelines for verification of eachand every aspect of the audit work and auditors are expected to use the guidelines and other techniqueseffectively to achieve best results.

Preparation of audit findings: After completion of the audit, audit findings are to be prepared. Every findingshould be substantiated with adequate evidence in the relevant Working Papers.

Review of results with the assessee: Findings with Working Papers to substantiate the findings especiallyrelating to short levies are to be handed over to the assessee and reviewed. Wherever the assessee agrees withthe findings, he may be persuaded to pay the amount and pay the correct duty in future. Where the findingsrelate to procedural deficiencies, an undertaking will be sought from the assessee about future compliance.

Review with the Range officer/Divisional Asstt. Commissioner: The audit party should also have a discussionwith the concerned RO/AC before preparing the final Audit report. This will ensure greater accuracy in thereport.

Finanlisation of Report: The final report will contain the details of findings of the audit party alongwith resultsof review with the assessee and the Range Officer/Divisional Asstt. Commissioner.

WORKING PAPERS

Working Papers are to be filled up by the Supervisor in the Audit team. He will detail documentation of workdone by the audit party in a given format. The papers will be used -

For substantiation of audit findings For evaluation of the performance of the auditors For use by succeeding auditors as a part of preliminary review

Page 183: Indirect Taxation CA Workbook

CA. Raj Kumar Practice Booklet There will be a set of Working Papers for each item of work/verification done by the auditors.

WHAT IS NEW IN THE NEW AUDIT MANUAL AND THE STANDARD AUDIT PROGRAMME

New way of selection of units for audit through ‘Risk Assessment’ concept. Before commencement of audit, Audit Plan will be approved by Addl. Commissioner/Joint

Commissioner (Audit). Emphasis on private records. System of discussion with key personnel from assessee’s side. Tour of factory.

Concept of Working papers being filled up by Auditors. This will provide transparency in the workdone.

Review of audit report with the assessee before it is filed. Objective is to measure degree of

compliance to law and procedure by the assessee.

System of formal review of audit and auditors’ work. by the ‘Audit Board’ headed by

Commissioner.

Greater transparency norms followed during conduct of audit.

Audit report to be prepared in prescribed format.

Codification of verification work to facilitate Computerisation.

Audit will be done by only trained officers. Senior officers will also get trained. Professionally

qualified Asstt. Directors (Cost) will also participate in audit in major units.

Training of Auditors will be done with the help of Instructor Guide and Participant Manual.

Clear signal that Audit should hand over to Anti-evasion where summon, search, seizure,

interrogation is called for.

NEW TECHNIQUES ADOPTED IN AUDIT

Selection of units based on Risk assessment (separate project).

Adoption of concepts of ‘mandatory’ and ‘discretionary’ workload and ‘target

penetration’ in audit selection.

Risk loss analysis (reasonableness test).

Trends analysis.

‘Walkthrough’ system.

Evaluation of Internal control system.

ABC analysis

SELECTION OF UNITS FOR AUDIT

Frequency of Audit Frequency of Audit Annual Total ED paid

Page 184: Indirect Taxation CA Workbook

CA. Raj Kumar Practice Booklet [ (PLA)+CCR]

1) Unit paying more than 3 crores Every year 2) Units Paying between Rs 1 crore and 3 Once every 2 years

Crores 3) Units paying between Rs. 50 Lakhs and Once every 5 years

1 crore 4) Units paying below Rs 50 Lakhs 10% of the units every year

Frequency of audit under Service Tax

Frequency of Audit Annual Total ED paid [(PLA)+CCR]

1) Unit paying more than 50 lakhs Every year 2) Units Paying between Rs 25 Lakhs and 50 Once every 2 years

Lakhs 3) Units paying between Rs. 10 Lakhs and Once every 5 years

25 Lakhs 4) Units paying below Rs 10 Lakhs 2% of the units every year

Advantages to be derived from the new Audit System

By the Department

The manuals give a message that audit is not a routine and mechanical exercise of review ofstatutory records and documents.

The new manuals shift the focus to preparatory work and verification of private records(envisage under Rule 173G).

Due importance has been given to returns and reports submitted to other law enforcingagencies. This will increase the information base for audit.

The new technique of selection of units for audit by Risk assessment will bring in optimal useof limited manpower.

Concept of evaluation of Internal Control system will help in determination of the departmentof scrutiny required. This will save time of assessees and auditors.

The manuals propose a regular system of review of the work of auditors. This will make auditmore result oriented.

The Standard Audit Programme would be the companion of auditors and it would help them toensure that they have not missed out any aspect.

Concepts like ABC analysis and ‘Walkthrough’ will help auditors to perform their functionsmuch better.

Training manuals will help in achieving minimum levels of uniform training in techniques ofauditing.

The concept of Working papers would ensure that auditors record their work in detail. The provision of evaluation of audits undertaken through the Monitoring Cell headed by

Commissioner will ensure the quality of audit. The new system of audit will bring ‘Audit wing’ to the forefront of Excise administration.

Page 185: Indirect Taxation CA Workbook

CA. Raj Kumar Practice Booklet It will create a more compelling environment of voluntary compliance.

By assessees :

Preparatory work done by audit parties in the office in advance would reduce the time requiredto be spent in the unit.

Department will be moving towards ‘Real Time Audit’ which will benefit assessees as therewill be less demand for old records.

Audit will be done in a far more transparent manner. Evaluation of Internal control systems by the audit parties will help the assessee to strengthen

their system to ensure that the legal requirements are fulfilled. Before end of the audit assessee will come to know the objections.

The assessee will get an opportunity to explain his views on the objections to be raised and as a resultsome of these will get settled. They will get a copy of the objection.

The stress in the new Manual is on ensuring that the audit does not cause inconvenience to theassessee. There will be adequate notice of the audit to the assessee (at least 15 days).

Department’s attempt will not be to reopen old assessments, which are settled withoutconcurrence of senior officers from the department.

There will not be more than one audit by the Excise department in a year. Since audit of more than Rs. 1 crore/5 crores units will be supervised by AC/DC/JC/ADC

(Audit), and the report signed by them, there will be less scope for frivolous audit. ‘Audit Protocol’ is an innovation from which assessees (who volunteer for this scheme) can

benefit immensely (eight Corporates have opted for this scheme; total number of units to becovered under this scheme is only 31, as of now).

There may not be any increase in the number of days taken for conducting the new audit. Forvery large units more number of auditors may be engaged to finish the work within areasonable time.

There will be no formal audit of units operating under Section 3A (capacity based assessment).

Page 186: Indirect Taxation CA Workbook

CA. Raj Kumar Practice Booklet

Examination Tips SOME SUGGESTIONS TO STUDENTS

Before the examination:

Understand the subject thoroughly. Do not memories answers without understanding. Improve English, as poor English creates bad impression on the examiner. Avoid long and unwieldy sentences. Practice to solve example. Mere reading example does not help. Slow and steady wins the race- regular study is much better than the last month’s rush. Solve as many old question papers as you can. makes your base of mathematics strong-if it is weak Quoting section numbers and important case law certainly makes better impression on examiner.

At the examination:

General

First solve question which you know best. Allocate time for answering each question and stop writing after time allotted by you to that question is over.

Keep margin for checking. Keep cool and do not panic.

Specific

Haw to read question properly AND How to make answer

WORDS USED IN THE QUESTION HOW TO MAKE ANSWER

List

State

Define

Make a list of…

Express, fully or clearly, the detail/facts of ...

Give the exact meaning of…

Page 187: Indirect Taxation CA Workbook

CA. Raj Kumar Practice Booklet Describe

Distinguish

Explain

Communicate the key feature of…

Highlight the differences between …

Make clear of intelligible; state the meaning of…

Identify

Illustrate

Recognize, establish or select after consideration

Use an example to describe or explain something

Calculate / compute

Prepare

Reconcile

Solve

Ascertain or reckon mathematically

Make or get ready for use

Make or prove consistent / compatible

Find an answer to

Analyse

Compare and contrast

Discuss

Interpret

Examine in detail the structure of…

Show the similarities and / or differences between.

Examine in detail by arguments...

Translate into intelligible or familiar terms

Advise

Evaluate

Recommend

Counsel, inform or notify

Appraise or assess the value of

Advise on a course of action

Solving practical problems –

Sometimes, some data is not given or question is vague. In such case, student should clearly state the assumptions made by him, so that examiner knows that you are aware of the legal provision.

If there are two alternate assumptions, student may solve the example in two different ways (if time permits), stating clearly assumptions made for each answer

Answering question on case study:

Usually, two questions are asked on recent case law, usually of Supreme Court. The answer should be presented in following sequence-

(a) Facts of the case – As stated in question, but in your own words

Page 188: Indirect Taxation CA Workbook

CA. Raj Kumar Practice Booklet (b) Issue for consideration – What issue is to be decided

[Starting lines may be as follows: The issue for consideration in the given question is………….]

(c) Legal position _Quote SECTIONS, RULES, CASE LAWS CIRCULARS, NOTIFICATION etc., regarding the ISSUE.

(d) Conclusion _ Give your conclusion in the light of Legal position.

[Ending lines may be as follows:

(1) In the light of what is stated above.......

(2) IN nutshell…….

(3) In the backdrop of the undisputed factual potion, the plea of department/ assessee is contrary to the facts……

(4) Summing up we hold that…..

(5) From the perusal of facts given in the question……….

(6) For all the aforesaid reasons……

(7) On careful consideration of stated facts and circumstances, our opinion on the various issues is as under…….

NOTE: If you remember the case law, state that facts of the case are similar to so and so case. If you do not remember citation, but at least remember that it is a Supreme Court case, you can state that way.

Note: Use your own reasoning. Whatever you write, write in a clear manner. Answer should be specific & throughout the answer only one view should be stated.

Attractive Paper Presentation: Answer paper should be made attractive and impressive by

Writing, point –wise and precise answers. Drawing graphs and charts Making tabular statements wherever required Writing question number and sub- number and head line of the question asked which will facilitate the

examiner in quickly assessing your paper.

After the examination

Do not discuss your answers with immediately after the examination as it will upset your mood for next paper.

Page 189: Indirect Taxation CA Workbook

CA. Raj Kumar Practice Booklet However, after the whole examination is over, review your answers.

……………………………………………………………………………………………………………………………………………………………………

FOR SUCCESSFUL PEOPLE THERE IS ONLY ONE SECOND OF TENTION AND ALL, ALL THE REMAINING SECONDS OF WORK.

REMEMBER PLAYER IS THAT WHO PLAY IN CRITICAL SITUATION. SPECIFIC NOTE: Take proper sleep.

With Best wishes

CA. Raj Kumar


Recommended